[d | an-b-bro-fr-gf-hr-l-m-maid-med-mi-mu-ne-o-old_o-p-ph-r-s-sci-sp-t-tran-tv-w-x | bg-vg | au-mo-tr | a-aa-abe-azu-c-dn-fi-hau-jp-ls-ma-me-rm-sos-tan-to-vn | misc-tenma-vndev | dev-stat]
[Burichan] [Futaba] [Gurochan] [Tomorrow] [Архив-Каталог-RSS] [Главная]

Файл: kuri.jpeg -(11 KB, 698x406, kuri.jpeg)
11 No.4863704  

Можно тут математика-тред устроить? Люблю поболтать о математике.

>> No.4863708  

Совсем невмоготу? До 1-го апреля-то всего ничего ж осталось!

>> No.4863710  

>>4863704

>Можно тут математика-тред устроить?

Если только это будет /b/акотред.

>Люблю поболтать о математике.

/sci/

>> No.4863715  
Файл: kuri91.jpg -(153 KB, 700x639, kuri91.jpg)
153
>Если только это будет /b/акотред.

На такой и рассчитываю!

>/sci/

Пускай будет филиалом!

>> No.4863720  

Образ гомомрфизма изоморфен факторгруппе по ядру гомомрфизма!

>> No.4863721  

>>4863715

> Пускай будет филиалом!

Ещё не закрыли, чтобы филиалы открывать.

>> No.4863723  
Файл: kuri92.jpg -(274 KB, 1332x850, kuri92.jpg)
274

>>4863720
Говоря проще, кообраз изоморфен образу!

>> No.4863725  
Файл: kuri92.jpg -(265 KB, 700x1246, kuri92.jpg)
265

>>4863721
Разве филиалы это нечто, что открывается, когда закрывается то, по отношению к чему филиал является филиалом?

>> No.4863734  

Какая вам больше нравится математика: непрерывная или дискретная?

>> No.4863740  
Файл: kuri93.jpg -(16 KB, 236x407, kuri93.jpg)
16

>>4863734
Та, которую сложно поместить в одну из этих двух категорий.

>> No.4863758  

Мне арифметика больше по душе. Так что наверное дискретная. Хотя и у непрерывной математики есть своя романтика. Нечто неуловимо напоминающее космологию и квантовую физику.
А вам какая арифметика больше нравится, теоретическая или формальная?

>> No.4863762  
Файл: kuri94.jpg -(255 KB, 551x900, kuri94.jpg)
255

>>4863758
Квантовые штуки это как раз нечто между дискретным и непрерывным мне кажется! А арифметика мне никакая не нравится, она вгоняет меня в тоску!

>> No.4863764  

>>4863762
На этом тред можно закрывать но перед этим скинь пак на Курису, если не бака

>> No.4863768  
Файл: kuri96.jpg -(8 KB, 188x268, kuri96.jpg)
8

>>4863764
На этом нужно закрывать обсуждение того, что тред нужно закрывать! Пак собираю по ходу дела сам, если будешь в треде активным, то скину!

>> No.4863775  
Файл: clr8faf2tuy01.jpg -(36 KB, 640x501, clr8faf2tuy01.jpg)
36
>Гуманитарии беснуюстя
>> No.4863780  

>>4863775
Кого цитируешь?

>> No.4863781  
Файл: kuri96.jpeg -(12 KB, 267x189, kuri96.jpeg)
12

>>4863775
Слово "гуманитарий" воспринимаю исключительно как комплимент, а не как пейоративность! Тем не менее, мог бы задать какие-нибудь вопросы, чтобы поднять беснование на тот нужный дискурсивный уровень, который не вызывал бы у тебя отторжения, заодно бы и другим пример подал!

>> No.4863782  

>>4863781
Не вижу смысла, математика для меня - ещё та бесовщина. И поднимать уровень дискуссии - только раздражать себя же самого. Я лучше выпью чаю да съем бутерброд с арахисовым маслом.

>>4863780
Себя из другого треда с другой борды.

>> No.4863785  
Файл: kuri97.jpg -(26 KB, 405x720, kuri97.jpg)
26

>>4863782
Что по-твоему тебя должно раздражить? Сама математика или проявление чьей-то в ней некомпетентности? В любом случае, слишком уж легко ты раздражаешься по слишком невинным поводам, раздражение слишком ценный ресурс, чтобы вот так им туда-сюда разбрасываться!

>> No.4863790  
Файл: Zq4fhsK6GQI.jpg -(38 KB, 604x549, Zq4fhsK6GQI.jpg)
38

>>4863723
Образ-кообраз.

>> No.4863791  
Файл: kuri98.jpeg -(6 KB, 189x267, kuri98.jpeg)
6

>>4863790
Ядро-коядро
Алгебра-коалгебра
Модуль-комодуль
Предел-копредел
Декартво квадрат-кодекартов квадрат
Кофе-фе
Коко-кококо

>> No.4863796  

>>4863762

> А арифметика мне никакая не нравится, она вгоняет меня в тоску!

Даже теория чисел?

>> No.4863798  
Файл: kuri99.jpeg -(7 KB, 172x294, kuri99.jpeg)
7

>>4863796
А кроме теории чисел бывает какая-то ещё?

>> No.4863800  
Файл: Pixiv 7035311 70136685 10.jpg -(51 KB, 600x800, Pixiv 7035311 70136685 10.jpg)
51

Предлагаю заставить ОПа считать в слух до числа Грэма.

>> No.4863801  
Файл: kuri100.jpeg -(3 KB, 225x225, kuri100.jpeg)
3

>>4863800
Хорошо, что не до числа райо!

>> No.4863803  

>>4863785
Сама математика как объект интереса.

>> No.4863804  
Файл: Pixiv 23623617 62436570.jpg -(43 KB, 384x384, Pixiv 23623617 62436570.jpg)
43

>>4863801
Куда хоть более полезный разговор и, наверное, хоть кто-то пошёл гуглить, что это за числа такие. Хотя, первой ссылкой будет Википедия, где он вообще ничего не поймёт.

>> No.4863814  
Файл: kuri101.png -(469 KB, 649x1183, kuri101.png)
469

>>4863804
Рад, что твоё желание того, чтобы наш тред выполнял образовательную функцию удовлетворено, хотя оно мне и кажется странным!
>>4863803
Тебе важно, чтобы мы придерживались некоторых рамок приличия в отношении своего интереса математики потому же почему и, скажем, стыдно взрослым состоятельным мужчинам увлекаться комиксами или по какой-то другой причине?

>> No.4863817  
Файл: kepchka.gif -(1 KB, 35x18, kepchka.gif)
1

>>4863814
Причина сугубо религиозная. Даже капча указывает на это.

>> No.4863825  
Файл: kuri102.jpeg -(6 KB, 288x175, kuri102.jpeg)
6

>>4863817
Может это обозначения синуса и она наоборот указывает на то, что тебе нужно написать в кои-то веки нечто математичное!

>> No.4863832  

>>4863825
Вычислять синусы - грешно

>> No.4863834  
Файл: kuri103.jpg -(465 KB, 715x1000, kuri103.jpg)
465

>>4863832
Тогда не стоит их вычислять!

>> No.4863837  

>>4863834
Однозначно. Матетматика - величайший грех. Я очищу этот тред!

>> No.4863839  
Файл: Pixiv 371157 22606624 01.png -(471 KB, 900x800, Pixiv 371157 22606624 01.png)
471

>>4863834
Мне вот тоже тригонометрия не очень нравилась. Скорее всего, из-за того, что решения простейших уравнений превращались в призыв Сатаны на целую страницу в тетрадке.

>> No.4863840  

Вообще странно что тригонометрию начинают учить до комплексных чисел и формулы Эйлера. Это как учить квадратные уравнения до отрицательных чисел.

>> No.4863849  
Файл: kuri104.jpg -(98 KB, 759x1017, kuri104.jpg)
98

>>4863837
Это хорошо, люди для которых математика невыносима, определенно добавят интересности!
>>4863839>>4863840
Это да, школьная математика вообще странное образование.

>> No.4863852  

>>4863840
Э? Но ведь тригонометрические функции от действительных чисел всегда действительные. И эти функции в действительных числах имеют простое и понятное приложение в геометрии. Какой смысл менять их на комплексную экспоненту? Конечно, она полезна, когда у тебя там частота и фаза, но всё же.

>> No.4863853  
Файл: 15337848855260.jpg -(8 KB, 300x222, 15337848855260.jpg)
8

>>4863849

>математика
>> No.4863855  

Теория типов — это математика?

>> No.4863856  

>>4863855
А каких типов, стрёмных?

>> No.4863859  

>>4863852
Так ведь и квадратные уравнения можно решать только в положительных числах, а для отрицательных объявлять что решения нету (для комплексных так тоже делают). Вроде бы до изобретения отрицательных было примерно так. Но при этом придется держать несколько подформул и специальных случаев, а если увеличить набор чисел, то всё становится легче и естественней.

>> No.4863862  
Файл: Pixiv 79961 63890498 06.jpg -(43 KB, 695x855, Pixiv 79961 63890498 06.jpg)
43

>>4863849
Ага, а последовавшие за ними пределы, логарифмы, матрицы воспринимались только как «извините, я вернулся в первый класс?», очень странное ощущение.

>> No.4863864  

С чего начинать изучать фило^Wматематику?

>> No.4863865  
Файл: kuri105.jpeg -(6 KB, 235x195, kuri105.jpeg)
6

>>4863855
Думаю всё же компьютер саенс, чисто математических вопросов там нету.
>>4863852
По модулю всего сказанного не очень даже понятно, зачем тригонометрические неравенства нужно решать даже с точки зрения математического образования, не говоря уже о практике!
>>4863862
Это да!

>> No.4863873  
Файл: _proofstep.gif -(12 KB, 592x369, _proofstep.gif)
12

А что думаете по поводу Metamath Proof Explorer
http://us.metamath.org/
http://us.metamath.org/mpegif/mmset.html
http://us.metamath.org/downloads/metamath.pdf
Система для формализации и верификации математических доказательств, работающая через систему переписывающих правил

>> No.4863875  

>>4863865
А какие вопросы являются чисто математическими?

>> No.4863876  

>>4863873
Я думаю, что Пролог сделали лет тридцать назад.

>> No.4863878  
Файл: kuri106.jpeg -(7 KB, 189x267, kuri106.jpeg)
7

>>4863864
Мне кажется Гельфанд Шень "Алгебра" хороший старт. Можешь ещё попробовать разные системы типа khan academy и brilliant org, с ними не очень знаком, но вроде беглым взглядом показалось, что для новичков норм.
>>4863873
Гильберт стайл пруф чекеры, да ещё и которые кодируют всё в ZFC это, мягко говоря, прошлый век.
>>4863875
В этом контексте: вопросы о всяких выразительных и доказательных силах, о семантиках и моделях и такое всё, а вопросы об эффективных программных реализациях о том, как что-то конкретное закодировать и вопросы типа "можно ли придумать какой-то ещё синтаксис, чтобы кодировать стало ещё удобнее" мне кажется всё же больше по части CS

>> No.4863879  
Файл: Axiome_du_choix.png -(39 KB, 466x273, Axiome_du_choix.png)
39

>>4863878

> Гильберт стайл пруф чекеры, да ещё и которые кодируют всё в ZFC это, мягко говоря, прошлый век.

Что значит "Гильберт стайл"?
Там нет встроенного обязательного ZFC. Ты вместо ZFC можешь свою аксиоматику придумать, описать как набор переписывающих правил, и из нее выводить следствия (леммы, теоремы).

>мягко говоря, прошлый век.

А что не прошлый век?

>> No.4863882  

>>4863878
Но ведь как минимум гомотопическая теория типов уж точно не про реализацию, а как раз таки про доказательства. Изоморфизм Карри–Ховарда и вот это вот всё.

>> No.4863886  
Файл: kuri107.jpg -(14 KB, 236x377, kuri107.jpg)
14

>>4863879
Гильберт-стайл значит отсутствие типов, малое количество правил вывода, схемы аксиом. Это всё конечно да, но основная база знаний в которой наиболее нетривиальные теоремы доказаны у них на ZFC, а обязательна она или нет - какая, в общем-то, разница. По-поводу ноу-хау в пруф-чекинге я точно не знаю, но думаю всякие HoTT и её вычислительные интерпретации, вроде кубических типов от того ученика Воеводского, которого я забыл как звать.

>> No.4863887  
Файл: _takagaki_kaede_idolmaster_drawn_by_sera(...).png -(1465 KB, 783x1200, _takagaki_kaede_idolmaster_drawn_by_sera(...).png)
1465

>>4863864
С Демидовича и Финкельштейна.

>> No.4863889  
Файл: kuri108.jpg -(17 KB, 236x334, kuri108.jpg)
17

>>4863882
А почему ты противопоставил доказательства и реализацию? Как раз задача вида "закодируйте доказательство чего-то там" она очень реализационная, если такое слово есть. Не знаю ни одной чётко сформулированной проблемы касательно HoTT, даже тот самый список на нлабе меня не устраивает, потому что все формулировки там очень wishy-washy, что среди математиков неприемлимо, а в среде CS - как раз.

>> No.4863891  

>>4863889
Ты явно в этом больше разбираешься, но насколько я знаю, HoTT по крайней мере кто-то планировал использовать в качестве фундамента для всей математики (вместо теории множеств сто лет назад и чего там ещё). Но при этом сам он математикой не является?..

>> No.4863892  
Файл: kuri109.jpeg -(4 KB, 300x168, kuri109.jpeg)
4

>>4863891
Ну ты как-то слишком серьезно к слову "математика" относишься, я лишь говорил, что оно для меня не ощущается как математика и люди, которые занимаются HOTT тусят среди CS и публикуются чаще всего в СS журналах, но наверняка этим кто-то и на математических факультетах тоже занимается, поэтому пусть будет математика, мне не жалко!

>> No.4863894  
Файл: Confluence_example_expression.png -(36 KB, 1000x779, Confluence_example_expression.png)
36

>>4863886

>Гильберт-стайл значит отсутствие типов, малое количество правил вывода, схемы аксиом.

Что значит "отсутствие типов"? Через системы переписывающих правил ты можешь себе сделать любые мыслимые типы. Знаешь что такое "система переписывающих правил?"

Что значит "малое количество правил вывода"? Каждая лемма/теорема это новое правило вывода, т.е. новый способ переписывания, т.е. новый способ замены X на Y. Если например надо доказать что 1 + 1 = 2 и у тебя где-то есть доказанная теорема вида
Если мы видим X + 1 Где X - один единственный символ из множества от 0 до 8 включительно, то вот набор правил замены
0 + 1 -> 1
1 + 1 -> 2
2 + 1 -> 3
...
8 + 1 -> 9
Но если X это 9 то замена будет
9 + 1 -> 10

Применяем это правило (теорему) к выражению 1 + 1 = 2 - получаем 2 = 2
А потом находим правило, что если слева и справа у нас два одинаковых числа, то это правда. Все, мы доказали что 1 + 1 = 2

>схемы аксиом

А что и как ты собираешься доказывать (выводить) без аксиом?

>> No.4863895  

>>4863892
Просто я со стороны думал, что вся современная математика скорее такая. А какая на самом деле современная математика?

>> No.4863901  
Файл: kuri110.jpeg -(4 KB, 300x168, kuri110.jpeg)
4

>>4863894

>Что значит "отсутствие типов"? Через системы переписывающих правил ты можешь себе сделать любые мыслимые типы.

Значит, что типы не вшиты в тот первоначальный синтаксис, который "дан из коробки", как например это делается в MLTT, а то, что их можно сэмулировать тут роли не играют - любая тьюринг-полная синтаксическая машинка эмулирует любую другую.

>Что значит "малое количество правил вывода"?

Значит то, что они же сами пишут в строчке " the only rules of inference are the (infinite) specific instances of the modus ponens and generalization schemes", то есть у них только 2 правила вывода, в противовес системам с натуральным выводом, где наоборот правил вывода много, а схем аксиом мало.

>А что и как ты собираешься доказывать (выводить) без аксиом?

Я не про аксиомы, а про схемы аксиом. Почитай на их же сайте абзац "axioms vs axiom schemes".

Вообще ты странный мод для разговора включил: будто я тебя сейчас в чём-то убеждать буду. Запомни: тут никто ни на что не претендует!

>> No.4863903  

>>4863892
Ты так это сказал, будто бы мне жалко, и будто бы это я захотел тебе непременно доказать, что это математика. Мне как бы гораздо более всё равно.

>> No.4863904  

>>4863901

>Вообще ты странный мод для разговора включил: будто я тебя сейчас в чём-то убеждать буду. Запомни: тут никто ни на что не претендует!

Почему тогда у тебя аватарка такое серьёзное лицо делает?

>> No.4863913  
Файл: kuri111.jpeg -(10 KB, 284x177, kuri111.jpeg)
10

>>4863895
Слишком широкий вопрос, но вот мне кажется что вопросы кодирования чего-то там в каком-то там синтаксисе - это точно мимо.
>>4863903
Мне показалось, что тебе показалось, будто бы я HoTT чем-то обидеть хотел когда сказал, что это CS. Но если нет, то и хорошо?
>>4863904
Разве же серьезное? Мне кажется что это лицо в смятении. А вообще я никогда не подбираю аватарку специально под сообщение.

>> No.4863921  

>>4863913

> Слишком широкий вопрос, но вот мне кажется что вопросы кодирования чего-то там в каком-то там синтаксисе - это точно мимо.

А если хотя бы кусочек?

>> No.4863932  
Файл: kuri112.jpeg -(9 KB, 225x225, kuri112.jpeg)
9

>>4863921
Есть большое количество "программ" или "направлений исследований", в каждом из которых есть свои объекты изучения, свои магистральные задачи, свои элементарные факты, свои звёзды и свое понимание того, какая задача интересна, а какая нет. Некоторые такие направления очень маленькие, другие большие. Некоторые направления типа "улучшить оценку на какие-нибудь числа/функции", некоторые типа "два числа/объекта какие-то совпали, вот бы построить теорию вокруг этого, которая объясняла бы почему, заодно может и что-то вокруг решим", некоторые занимаются классификациями алгебраических и геометрических объектов, некоторые наоборот изучают контрпримеры и исключительные объекты, то, что из классификаций выбивается. Вообще никакого общего проекта у математиков, в отличии от фундаментальных физиков, нету.

>> No.4863936  
Файл: 12-Figure1-1.png -(21 KB, 1246x816, 12-Figure1-1.png)
21

>>4863901

> Значит, что типы не вшиты в тот первоначальный синтаксис, который "дан из коробки", как например это делается в MLTT, а то, что их можно сэмулировать тут роли не играют - любая тьюринг-полная синтаксическая машинка эмулирует любую другую.

А есть ли смысл их встраивать, и какая в этом выгода? Может быть правильней будет иметь как можно более простую реализацию самой синтаксической машины, без всех этих типов, и все подобные вещи уже описывать через правила переписывания для нее? Так можно сократить места, где можно совершить ошибку. Еще можно сделать математически доказанный способ выполнить трансляцию безтипового первоначального синтаксиса в типовый, оттранслировав при этом и все имеющиеся аксиомы, теоремы и их доказательства, и вообще супер будет.

>Значит то, что они же сами пишут в строчке " the only rules of inference are the (infinite) specific instances of the modus ponens and generalization schemes", то есть у них только 2 правила вывода, в противовес системам с натуральным выводом, где наоборот правил вывода много, а схем аксиом мало.

Почему это плохо? В чем системы с натуральным выводом лучше этого? Там есть сквозное доказательство всего и вся, начиная с самых основ?

>Вобще ты странный мод для разговора включил: будто я тебя сейчас в чём-то убеждать буду. Запомни: тут никто ни на что не претендует!

Ты неправильно интерпретируешь мое поведение. Я просто хочу разобраться, что тут лучше и что тут хуже. Например, нужны ли все эти типизации, или они только мешают и усложняют реализацию? А что насчет всяких SMT решателей, Coq, Isabelle/HOL, Idris, Agda? Почему их так много, и насколько им можно доверять? https://github.com/clarus/falso вот например в Coq смогли доказать ложь из-за бага в реализации самого Coq, ну куда это годится?

>> No.4863938  

>>4863932
Ну а какой всё-таки раздел математики сейчас генерирует больше всего инноваций?

>> No.4863942  
Файл: kuri113.jpg -(12 KB, 235x344, kuri113.jpg)
12

>>4863936
Не знаю есть ли в этом выгода, ну видимо раз Воеводский этот проект начал, то есть, он всё же кое-что и в математике и в её основаниях понимал. Вероятно типы более естественны для математики, так как это то, как математики в принципе работают: имея два пространства можно взять их произведение, или копроизведение, или ещё какую-нибудь операцию над ними проделать, а такие объекты как спектр кольца, расслоение, пучок - они вообще зависимые типы естественным образом. А гомотопический тип (из математики) естественным образом является гомотопическим типом (из HoTT), что должно облегчать кодирование доказательств с их участием.

Насколько можно доверять - вопрос философский, мне вот большей уверенности в верности доказательства не придаёт то, что его закодировали программисты. Потому что, в конце концов, кто будет проверять то, что они закодировали именно то, что мне нужно?

За ссылку спасибо, не знал, будет чем подколоть программистов.

>>4863938
Ну давай я скажу "программа ленглендса" (алгебра + геометрия), если тебе совсем-совсем конкретика нужна.

>> No.4863944  

Test

>> No.4863946  

Слишком серьёзно. А говорили что баковать будете.

>> No.4863948  
Файл: kuri114.jpeg -(6 KB, 234x215, kuri114.jpeg)
6

>>4863946
Так а ты побакуй, а я поддержу!

>> No.4863959  
Файл: Lambda_cube.png -(6 KB, 321x294, Lambda_cube.png)
6

>>4863942

> Насколько можно доверять - вопрос философский, мне вот большей уверенности в верности доказательства не придаёт то, что его закодировали программисты. Потому что, в конце концов, кто будет проверять то, что они закодировали именно то, что мне нужно?

Доказательство необязательно кодируются программистами. Можно просто брать чужие готовые доказательства, доказанные математиками, и переписывать в свою систему проверки доказательств. И таким образом доказывая корректность всех отдельных шагов в этом доказательстве. Но это все сработает только если сама система переписывающих правил не имеет ошибок, но ее достаточно легко сделать без ошибок (с тем же coq в этом плане сложнее будет).

>За ссылку спасибо, не знал, будет чем подколоть программистов.

Ну тут ничего особенного нет, людям свойственно ошибаться. И математики в своих теоремах допускают ошибки. И поэтому есть смысл делать программы, которые бы проверяли корректность доказательства. Но сами программы проверки желательно делать настолько простыми, насколько это вообще возможно, дабы избежать ошибок в самой реализации. И мне кажется что всякие типизированные лямбда-исчисления, если они встроены в само ядро пруфчекера, а не выводятся каким-либо образом из базовых аксиом - это плохо, т.к. мы не можем проверить что эти типизированные лямбда-исчисления реализованы правильно.

Кстати, что вообще лучше почитать про основания математике, чтоб прям на пальцах всё объяснялось?

>> No.4863984  
Файл: kuri116.jpeg -(10 KB, 166x303, kuri116.jpeg)
10

>>4863959

>Доказательство необязательно кодируются программистами. Можно просто брать чужие готовые доказательства, доказанные математиками, и переписывать в свою систему проверки доказательств. И таким образом доказывая корректность всех отдельных шагов в этом доказательстве. Но это все сработает только если сама система переписывающих правил не имеет ошибок, но ее достаточно легко сделать без ошибок (с тем же coq в этом плане сложнее будет).

Ну так этим как раз программисты и занимаются в основном! А как оно работает я знаю, но за ревью спасибо всё равно!

>Ну тут ничего особенного нет, людям свойственно ошибаться. И математики в своих теоремах допускают ошибки. И поэтому есть смысл делать программы, которые бы проверяли корректность доказательства. Но сами программы проверки желательно делать настолько простыми, насколько это вообще возможно, дабы избежать ошибок в самой реализации. И мне кажется что всякие типизированные лямбда-исчисления, если они встроены в само ядро пруфчекера, а не выводятся каким-либо образом из базовых аксиом - это плохо, т.к. мы не можем проверить что эти типизированные лямбда-исчисления реализованы правильно.

Естественно ничего такого нет, естественно все могут ошибаться, обещаю, что мои подколки если и не будут не злые, то по крайней мере в них тоже не будет ничего такого! Но программы проверки нужно делать такими, чтобы в них удобно было, собственно, кодировать сложные доказательства, а аккуратную реализацию оставлять на откуп хорошим CSникам. В конце концов, 100%ой уверенности не достигнуть никогда, потому что вдруг в ядре процессора ошибка (которое устроено ой как сложно, по крайней мере документации современных процессоров часто занимают несколько тысяч страниц)?

>Кстати, что вообще лучше почитать про основания математике, чтоб прям на пальцах всё объяснялось?

Это хороший вопрос, есть область знаний связанная с метаматематикой, то есть с метаматематическими свойствами теорий, вроде теорем Гёделя, Лёба, Тарского, Тенненбаума и тд, это лучше в Cori Lascar "Mathematical Logic course with exercises" мне кажется очень хороший учебник, но в сети вроде нету, есть ещё пост Тао https://terrytao.wordpress.com/2009/04/10/the-completeness-and-compactness-theorems-of-first-order-logic/ очень хорошее ревью на основые теоремы мат.логики. А если ты конкретно об синтаксисе оснований, кодировании и всём таком, то не знаю даже, читай оригинальные работы мартина лёфа и после этого HoTT book (она очень хорошо написана и легко читается).

>> No.4863991  

>>4863855
Да. Любая работа с/в формальные языки - математика.

>> No.4863994  
Файл: Pixiv 38326 56176448.png -(322 KB, 646x710, Pixiv 38326 56176448.png)
322

>>4863991
Любой формальный язык, есть алгоритм. Алгоритм, это программа для некоторой машины Тьюринга. Любую машину Тьюринга можно привести к другой, в том числе, построенной на чистой математике.

Всё, что можно представить формальным языком, есть математика.

Видел где-то такое.

>> No.4864000  
Файл: kuri117.jpeg -(12 KB, 183x275, kuri117.jpeg)
12

>>4863991
Надеюсь хоть, что не любая математика - это работа, а то работать не люблю очень!
>>4863994
Машинна Тьюринга - это и есть программа, а не приёмник программ, и это и есть математическая абстракция программы на языке программирования, а не так, что некоторые машины тьюринга "внутри математики", а некоторые "вне её". А про "всё что можно представить формальным языком - математика" мне кажется что нет! Думаю математика находится на уровне той истории, для которой нужно придумать формальный язык, чтобы её красиво оформить, а не на уровне самого формального языка, поэтому матматики основания и не очень чтут.

>> No.4864007  

>>4863984

>Но программы проверки нужно делать такими, чтобы в них удобно было, собственно, кодировать сложные доказательства, а аккуратную реализацию оставлять на откуп хорошим CSникам.

Нет, я тут не согласен.
Процитирую фрагмент с сайта Metamath http://us.metamath.org/

> Unlike most other systems, Metamath attempts to use the minimum possible framework needed to express mathematics and its proofs. Other systems do not consider that aspect necessarily important, and their underlying computer programs can be large and complex in order to perform mathematical reasoning at a higher level. Metamath's proofs are often quite long compared to those of other systems, but they are completely transparent with nothing hidden from the user. All reasoning is done directly in the proof itself rather than by algorithms embedded in the verification program. Metamath is unique in this sense, offering an alternative approach for those attracted to its philosophy of simplicity.

Т.е. очень важно иметь минимальную простую и корректную реализацию прувчекера, в которой ошибиться очень трудно. Для облегчения кодирования сложных доказательств лучше использовать что-то вроде препроцессора, который более удобный для человека синтаксис превращает в менее удобный синтаксис для программы и потом передает ей для проверки. Чтоб если в этом препроцессоре будет бага, то теорем чекер просто не смог бы проверить доказательство из-за этого бага.

>В конце концов, 100%ой уверенности не достигнуть никогда, потому что вдруг в ядре процессора ошибка (которое устроено ой как сложно, по крайней мере документации современных процессоров часто занимают несколько тысяч страниц)?

Ядра процессоров кстати доказывают с помощью формальных методов, тех же SMT решателей. Можно спроектировать очень простой процессор, доказать его корректность (т.е. сооветствие некоторой спецификации) и потом на нем же запустить пруфчекер, проверить доказательство корректности его самого. Если так сделать, то уверенность будет хоть и не 100%, но очень большая.

>> No.4864009  
Файл: 7.png -(99 KB, 2200x700, 7.png)
99

Кстати, вопрос еще про аксиоматику теории множеств. Вот континуум-гипотеза, которая не доказывается ни в ZF ни в ZFC аксиоматике, и требует введения аксиомы детерминированности. Каким образом (из-за чего) вообще придумываются аксиомы? Они придумываются при появляении некоторой гипотезы, которую без этой аксиомы не доказать? Или есть какие-то правила построения системы аксиом?

>> No.4864011  
Файл: kuri119.jpeg -(5 KB, 189x267, kuri119.jpeg)
5
>Т.е. очень важно иметь минимальную простую и корректную реализацию прувчекера, в которой ошибиться очень трудно. Для облегчения кодирования сложных доказательств лучше использовать что-то вроде препроцессора, который более удобный для человека синтаксис превращает в менее удобный синтаксис для программы и потом передает ей для проверки. Чтоб если в этом препроцессоре будет бага, то теорем чекер просто не смог бы проверить доказательство из-за этого бага.

Не вижу большого отличия от ситуации, когда более сложный и удобный для человека синтаксис реализуется напрямую в машинный код. Как я узнаю, что промежуточные препроцессорно-сгенерированные теоремы на метамате логически соответствуют той теореме, которую я доказываю и в более сложном синтаксисе?

Грубо говоря, утрируя ситуацию в бесконечность раз, представим глупый синтаксис следующего рода: он любой текст переводит в тождественную истину, или любую другую тождественно чекаемую теорему на МЛТТ, и вот я пишу на этом более сложном синтаксисе "гипотеза Римана верна, мамой клянус" а оно хоп и скомплировалось мне, уверенность какого толка это должно мне дать?

>>4864009
Только из идеологии, идеология введения аксиомы выбора основана на идее, которая называется "унивёрсум фон-Неймана", фактически это нечто неопределяемое (ну почти, определяемая только в теориях, которые ещё страшнее чем этот унивёрсум), это просто картинка, мысль, концепт, вот в унивёрсуме фон Неймана аксиома выбора верна, ровно как верно и бесконечное количество других аксиом, которые теоретико-множественники готовы присоеденить к ZFC и называемые "аксиомами больших кардиналов", проблема в том, что никаким рекурсивно-перечислимым множеством аксиом идею этого самого "унивёрсума фон-Неймана" не схватить, теорема Гёделя. В унивёрсуме фон-Неймана континуум-гипотеза не верна.

Аксиома детерменированности взялась из желания сделать так, чтобы все множества на прямой были измеримы и построить тем самым более простой и красивый анализ, и хоть эту программу можно даже докрутить до логического конца, ущерба от того, что ломаются некоторые базовые интуиции касательно мощностей и того, что можно делать с множествами, а что нельзя гораздо больше, чем от приписки "пусть f измеримая функция" при любой рассуждении в вещественном анализе.

Это всё только моё мнение, конечно.

>> No.4864013  

>>4864011

>на МЛТТ

не на млтт, а на метамате, ну тут неважно конечно

>> No.4864015  

>>4863704
Математика — царица полей.

>> No.4864017  

>>4864011

>Не вижу большого отличия от ситуации, когда более сложный и удобный для человека синтаксис реализуется напрямую в машинный код.

Разница в том, что правильно реализовать трансляцию более сложного синтаксиса напрямую в машинный код - сложнее. Больше мест, где можно ошибку допустить. Надо доказывать корректность этой самой трансляции.

>Как я узнаю, что промежуточные препроцессорно-сгенерированные теоремы на метамате логически соответствуют той теореме, которую я доказываю и в более сложном синтаксисе?

Если у тебя есть формальное определение этого более сложного синтаксиса, можно опять таки доказать соответствие одного синтаксиса другому. Т.е. можно доказать, что любое утверждение в более сложном синтаксисе может быть переписано в более простой синтаксис метамата, и это будет по смыслу в точности то же самое.

>> No.4864018  

>>4864011

>Грубо говоря, утрируя ситуацию в бесконечность раз, представим глупый синтаксис следующего рода: он любой текст переводит в тождественную истину, или любую другую тождественно чекаемую теорему на МЛТТ, и вот я пишу на этом более сложном синтаксисе "гипотеза Римана верна, мамой клянус" а оно хоп и скомплировалось мне, уверенность какого толка это должно мне дать?

Ну очень просто. Если у тебя доказано что "он любой текст переводит в тождественную истину, или любую другую тождественно чекаемую теорему на МЛТТ" то все супер, и гипотеза Римана верна.

>> No.4864020  
Файл: kuri121.jpeg -(8 KB, 189x267, kuri121.jpeg)
8

>>4864017

>Т.е. можно доказать, что любое утверждение в более сложном синтаксисе может быть переписано в более простой синтаксис метамата, и это будет по смыслу в точности то же самое.
>Надо доказывать корректность этой самой трансляции.

И как ты собираешься не доказать, но хотя бы даже сформулировать утверждение об этом? Чтобы хотя бы сформулировать "это было по смыслу в точности то же самое", то наверное у них должен быть один смысл слова "смысл" иначе говоря, у них должна быть одна и та же семантика, но если у них одна и та же семантика, то наверное "более сложный синтаксис" не такой уж более и сложный? Ведь более сложный синтаксис заводят, чтобы думать о более сложных объектах, я работаю в HoTT, чтобы думать о его термах как об именах объектов и морфизмов некоторого специфического (\infty,1)-топоса, каким образом я хотя бы сформулирую "такая-то интерпретация HoTT в metamath сохраняет смысл", если я даже не могу насытить термы metamath тем же смыслом, что и термы HoTT?

Даже не утрируя ситуацию, представим что у нас есть язык metamath и машинный код, все валидные термы которого, - это просто конечные строки из 0 и 1. Каким образом по твоему можно доказать, что отображение некоторого доказательства того, что 2+2=4 cо всеми шагами в строку вида "0010111010100111...00101" сохраняет смысл? И вообще, какой самостоятельный смысл у строки "0010111010100111...00101" может быть в отрыве от всякого кодирования? Или доказательством будет ссылка на факт, что "0010111010100111...00101" это код соответствующего доказательства в metamath, при чём тут смысл тогда?

>Ну очень просто. Если у тебя доказано что "он любой текст переводит в тождественную истину, или любую другую тождественно чекаемую теорему на МЛТТ" то все супер, и гипотеза Римана верна.

Доказзывать это не надо, потому что это определение моей интерпретации одного языка в другой, то есть доказательством тут будет являться ссылка на определение. Если ты намекаешь на то, что она плохая и бесполезная, то тогда скажи как концептуально отличить плохую интерпретацию между языками разной природы от хорошей. Думаю, это ничего не значит.

>> No.4864054  
Файл: 6-Figure3-1.png -(5 KB, 512x112, 6-Figure3-1.png)
5

>>4864020

>каким образом я хотя бы сформулирую "такая-то интерпретация HoTT в metamath сохраняет смысл", если я даже не могу насытить термы metamath тем же смыслом, что и термы HoTT?

Это про проекции Футамуры-Турчина, суперкомпиляция, преобразование программ
https://ru.wikibooks.org/wiki/Основы_функционального_программирования/Трансформация_программ
https://youtu.be/UWgBqQvj1gc https://habr.com/ru/post/47418/
Теория компиляции... У меня от нее голова кружится. Но я попробую сформулировать. Не уверен что всё напишу правильно, если что - переспроси.
Вот допустим HoTT - некий язык. Есть интерпретатор HoTT которому если скармливаем доказательства на языке HoTT и он говорит "True" если доказательство верно или "False" если нет. Пусть он будет вообще мамой клянус 100% корректный (ЭТО ВАЖНО. Будем считать что реализация интерпретатора это его спецификация и есть. Иначе надо тогда понять, в какой теории описана спецификация самого языка, т.е. описано то, как или каким образом формулируется ответ "True" или "False" на ту или иную входную строку, и это надо перевести в интерпретатор)
То же самое, есть у нас интерпретатор Metamath которому скармливаем доказательства на соотв. языке и он говорит: "True" если доказательство верно или "False" если нет. Он тоже 100% корректный. Транслятор из HoTT языка в Metamath язык - какими он свойствами должен обладать? Очень просто - он должен в ответ на любую корректную с т.з. языка HoTT строку выдавать корректную с т.з. языка Metamath строку такую, которая будучи переданная на вход интерпретатору Metamath, то он будет выдавать "True" если "True" выдаст на исходную строку сам интерпретатор языка HoTT, и выдаст "False" если в точности то же самое выдаст HoTT интерпретатор в ответ на исходную строку HoTT. Это свойство транслятора и надо доказывать.
И получается, что если например в HoTT языке ты докажешь гипотезу Римана, и если даже при трансляции из HoTT в Metamath ты получаешь какую-то ерунду, типа доказательства для равенства 2+2=4 то это даже и неважно, т.к. у тебя есть доказательство для транслятора, что если там правда, то и там правда. Этого уже будет достаточно. Хотя это и нафиг не надо, раз у тебя есть по условию 100% гарантия корректности интерпретатора HoTT. Но ты спрашивал как это сформулировать - я ответил. Надеюсь, не слишком всё запутано.

>Даже не утрируя ситуацию, представим что у нас есть язык metamath и машинный код, все валидные термы которого, - это просто конечные строки из 0 и 1. Каким образом по твоему можно доказать, что отображение некоторого доказательства того, что 2+2=4 cо всеми шагами в строку вида "0010111010100111...00101" сохраняет смысл?

Ну ок, попробуем. Первый вопрос - откуда взялась строка вида "0010111010100111...00101"?
Допустим, существует некая функция f(x) написанная в тех же машинных кодах, которая принимает строку на языке Metamath, и возвращает "00100..." который делает то же самое, что и Metamath, которому скормили такую же точно строку на языке Metamath. Вопрос - как это доказать?
Можно действовать следующим образом. У нас есть документация на машинный код. Описываем его в metamath т.е. пишем интерпретатор машинного кода внутри Metamath на языке Metamath. Потом цепляем нашу функцию f(x) к этому интерпретатору, чтобы она им исполнялась. Дальше пишем теорему, что для любой строки справедливо, что если ее подставить в функцию f(x) (которая интерпретируется интерпретатором машинного кода) то она сгенерирует такую строку "00100..." которая будучи проинтерпретированной интерпретатором машинного кода, выдаст тот же результат, что и выдаст сам метамат, который, если надо, тоже можно в самом метамате описать.
Вообще, Машину Тьюринга (МТ) вполне можно описать на метамате. В МТ можно написать реализацию точно такой же МТ и потом можно даже доказать, что та МТ запущенная внутри МТ в ответ на любой код будет такая же по поведению, как и МТ сама по себе, лол.

>> No.4864125  
Файл: kuri122.jpg -(16 KB, 236x448, kuri122.jpg)
16

>>4864054
Ещё раз, если он 100% корректный, то зачем мне metamath в качестве предыдущего шага? Если он не 100% корректный и мне нужен metamath чтобы повысить свою уверенность в том, что он корректный, то какого сорта уверенность я получу от того, что гипотеза Римана на HoTT транслируется в доказательство "2+2=4" на metamath?

>> No.4864130  

А давайте вместо "metamath" говорить "me tomato"?

>> No.4864133  
Файл: kuri123.png -(126 KB, 690x399, kuri123.png)
126

>>4864130
Хорошая идея!

>> No.4864138  
Файл: fuura_kafuka__sayonara_zetsubou_sensei__(...).jpg -(23 KB, 1024x576, fuura_kafuka__sayonara_zetsubou_sensei__(...).jpg)
23

Какие вы тут все умные. В школе я постоянно занимал призовые места на области по математике почему отказался ехать на всерос - отдельная история, но сейчас могу лишь в матан или в задачки, не требующие глубоких знаний.

>> No.4864140  
Файл: kuri124.jpg -(10 KB, 236x471, kuri124.jpg)
10

>>4864138
Ты молодец всё равно отдельную историю можешь рассказать, если хочешь! Но ещё никто вроде не умничал!

>> No.4864155  
Файл: 6748e08379b274041c2afc28c43eba8427fe91fa.jpg -(57 KB, 620x717, 6748e08379b274041c2afc28c43eba8427fe91fa.jpg)
57

>>4864015
Математика — царица лолей.

>> No.4864160  

>>4864155
Царица с интегралом!

>> No.4864163  
Файл: fallacy.png -(17 KB, 251x196, fallacy.png)
17

>>4864125

>Ещё раз, если он 100% корректный, то зачем мне metamath в качестве предыдущего шага? Если он не 100% корректный и мне нужен metamath чтобы повысить свою уверенность в том, что он корректный, то какого сорта уверенность я получу от того, что гипотеза Римана на HoTT транслируется в доказательство "2+2=4" на metamath?

Есть более глубокий вопрос. Достаточно ли ты четко сам понимаешь, что именно представляет из себя HoTT, чтобы на основании спецификации HoTT и реализации самого проверятеля-решений-через HoTT убедиться в ее безошибочности? https://github.com/HoTT/HoTT вот у нас этот самый HoTT описан на языке Coq. Как я уже показывал, в Coq иногда бывают баги, из-за которых можно доказать ложь. Откуда мне знать, что в процессе описания HoTT теории там нигде никто не доказал ложь? В корректность самого Coq можно только уверовать, а сам Coq запрограммирован на языке OCaml в который тоже нет 100% уверенности...
Попробую через аналогию. Намного проще написать 100% корректный интерпретатор языка Brainfuck, чем написать 100% корректный интерпретатор машинного кода для некоторого нетривиального процессора. Так вот, интерпретатор языка Brainfuck это считай что Metamath, а HoTT (который описывается в Coq теории) это как интерпретатор машинного кода для процессора. И там и там у нас будет тьюринг-полнота, т.е. не будет вообще ничего, что можно выразить там, но нельзя там.
Если у тебя есть некая теория (язык) HoTT и существует способ на машине Тьюринга за конечное число шагов для синтаксически корректного (well formed formula) доказальства на HoTT точно понять, правильное это док-во или нет, то этот процесс проверки корректности доказательства вполне может быть переписан в Metamath, и у тебя будет больше уверенности, чем от Coq + OCaml в котором этот HoTT сейчас реализован. Надеюсь, так более понятно...

>> No.4864165  

>>4864160
Собственным или несобственным?

>> No.4864166  

>>4864165
Неопределенным!

>> No.4864170  
Файл: kuri125.jpg -(231 KB, 480x800, kuri125.jpg)
231

>>4864163
Вопрос о том, насколько я убеждён в безошибочности HoTT мне не кажется интересным, потому что для меня кодирование доказательства на любом пруфере не делает его более убедительным. А то, что в какой-то уровень всё равно придётся поверить без всяких аргументов и так понятно, просто для меня это уровень "доказательства написанного на английском языке, и проверенным сообществом профессиональных математиков".

Даже в этой твоей аналогии (хоть она и не совсем корректна, потому что у языков программирования общая семантика более-менее есть, в отличии от пруверов) до сих пор не понимаю, почему интерпретатор языка OCaml написанный на бреинфаке должен меня убеждать в корректности языка OCaml больше или меньше, чем если бы этот интерпретатор был бы реализован на аппаратном уровне.

>>4864165
Тот, который спаривание цепи с коцепью!

>> No.4864176  
Файл: Turing_machine_1.JPG -(149 KB, 1358x830, Turing_machine_1.JPG)
149

>>4864170

>Вопрос о том, насколько я убеждён в безошибочности HoTT мне не кажется интересным, потому что для меня кодирование доказательства на любом пруфере не делает его более убедительным.

А что делает?

>А то, что в какой-то уровень всё равно придётся поверить без всяких аргументов и так понятно, просто для меня это уровень "доказательства написанного на английском языке, и проверенным сообществом профессиональных математиков".

Да. И желательно чтоб тот уровень, в который надо верить, был как можно более маленьким. И Metamath в этом плане лучше, чем HoTT работающий через Coq.

>(хоть она и не совсем корректна, потому что у языков программирования общая семантика более-менее есть, в отличии от пруверов)

Если функция проверки корректности (которая обязательно должна завершаться за конечное число шагов) для прувера №1 может быть закодирована на брейнфаке, и если брейнфак-интерпретатор может быть закодирован в прувере №2 и в этом прувере можно доказать, что именно выдаст этот интерпретатор интерпретатор с подставленным в него кодом проверки корректности для любого доказательства, переданного в него, то у нас уже прувер №1 сводим к пруверу №2.

>почему интерпретатор языка OCaml написанный на бреинфаке должен меня убеждать в корректности языка OCaml больше или меньше, чем если бы этот интерпретатор был бы реализован на аппаратном уровне.

Ну хотя бы потому, что сам брейнфак намного проще, чем аппаратный уровень. В него как-то проще поверить.

>> No.4864185  
Файл: kuri126.jpeg -(7 KB, 207x243, kuri126.jpeg)
7
>Да. И желательно чтоб тот уровень, в который надо верить, был как можно более маленьким. И Metamath в этом плане лучше, чем HoTT работающий через Coq.

Меня мелкость уровня ни в чём не убеждает, скорее наоборот, чем более короткое и высокоуровневое доказательство - тем более скорее, что оно правильное.

>Если функция проверки корректности (которая обязательно должна завершаться за конечное число шагов) для прувера №1 может быть закодирована на брейнфаке, и если брейнфак-интерпретатор может быть закодирован в прувере №2 и в этом прувере можно доказать, что именно выдаст этот интерпретатор интерпретатор с подставленным в него кодом проверки корректности для любого доказательства, переданного в него, то у нас уже прувер №1 сводим к пруверу №2.

Осталось только найти оракула, который скажет, что функция проверки корректности на бреинфаке - это действительно функция проверки корректности на бреинфаке, а не что либо ещё. А лучше не искать а реализовывать функцию проверки корректности на том, что удобно и что лежит под рукой, а не на Coq.

>> No.4864194  

>>4864185

>Меня мелкость уровня ни в чём не убеждает, скорее наоборот, чем более короткое и высокоуровневое доказательство - тем более скорее, что оно правильное.

Лол, вспомнилась цитата:
«Есть два подхода к программированию. Первый — сделать программу настолько простой, чтобы в ней очевидно не было ошибок. А второй — сделать её настолько сложной, чтобы в ней не было очевидных ошибок.»
Tony Hoare. Профессор, занимался реализацией Алгол 60, сейчас исследователь в Microsoft Research.

Так что или у нас сложный проверятель доказательств, в котором могут быть ошибки (но нет очевидных ошибок), но зато там можно записать короткие высокоуровневые доказательства (в которые проще верить, хотя зато сложнее поверить в безошибочность самого проверятеля).
Или у нас простой проверятель доказательств, в котором очевидно нет ошибок, но доказательства какие-то длинные. Но он их проверяет, и делает это правильно.

Второй вариант мне кажется лучше первого

>> No.4864198  
Файл: detail_8f5de01b30a5f8a7a94689d1d5a14b9f.jpg -(206 KB, 600x400, detail_8f5de01b30a5f8a7a94689d1d5a14b9f.jpg)
206

>>4864170

> для меня это уровень "доказательства написанного на английском языке, и проверенным сообществом профессиональных математиков".

Слышал про замощение плоскости пятиугольным паркетом? Давно было "доказано", что вот только такими вариантами выпуклыми прямоугольниками можно замостить плоскость, потом какая-то домохозяйка прочитала об этом в каком-то журнале и придумала контрпример. Сейчас вроде бы появилось окончательное доказательство, что типов замощений ровно 15
https://youtu.be/hkhaipY3JmU?t=5064 https://ru.wikipedia.org/wiki/Пятиугольный_паркет

>> No.4864200  

Никогда не любил математику в школе. Сейчас бы с каким-нибудь преподавателем с радостью разбирал задачки с самого 4-го класса и выше.
Детство, всё-таки, странная вещь. Вроде говорят, что в детстве проще всего учиться, больше всего информации впитываешь, но даже если вспомнить предметы, которые мне нравились, я точно помню, что как-то осознанно не учился. А сейчас вот с радостью. И во сколько же лучше всего учиться? Это ещё вопрос Во какой! Его надо задать, а потом на него ответить. Ну, собственно, я его уже задал.
Такая вот математика.

>> No.4864201  

>>4864185

>Осталось только найти оракула, который скажет, что функция проверки корректности на бреинфаке - это действительно функция проверки корректности на бреинфаке, а не что либо ещё.

Что-то мне подсказывает, что оракул тут не требуется. Это можно попробовать доказать теми же самыми теорем чекерами, и это должно быть доказуемым утверждением.

>> No.4864202  
Файл: kuri127.jpg -(24 KB, 320x452, kuri127.jpg)
24

>>4864194
Имеешь право, но если что я не верю интуиции программистов в вопросах математических доказательств.

>>4864198
Слышал, я кстати сейчас заглянул на энвики, и чего-то не увидел этой самой истории с несколькими итерациями доказательств опубликованных в журналах и последующих контрпримеров предъявленных домохозяйкой, о которой так любят все рассказывать. Но в любом случае, даже если она и была, то мне не кажется, будто бы если заставить каждого математика кодировать свои доказательства в пруфчекерах таких историй станет меньше, просто ошибки переместятся с уровня фактиеских на уровень "закодировано было совсем не то, что заявлялось", потому что я вот сейчас зашёл на метамас посмотреть fundamental theorem of algebra и вообще не понял, как в тех 20 строчках метамасного кода (пусть даже хорошо откомментированного) её там можно разглядеть, так что эти пруверы просто увеличат количество рабочих мест для умных программистов (что хорошо) и количество бюрократии (что плохо).

>> No.4864210  
Файл: 536746021111e22f1cd500134b0d1da683c37ec1.png -(7 KB, 579x58, 536746021111e22f1cd500134b0d1da683c37ec1.png)
7

>>4864202

>Имеешь право, но если что я не верю интуиции программистов в вопросах математических доказательств.

Но в корректность самого Coq ты почему-то веришь.

>> No.4864212  
Файл: kuri128.gif -(2033 KB, 500x450, kuri128.gif)
2033

>>4864200
Думаю в том возрасте, когда чувствуешь любовь к знаниям и не чувствуешь отторжения от процесса обучения!
>>4864201
А мне кажется, что это принципиально неформулируемое утверждение, потому что то, что семантика для языка является семантикой ты можешь только объявить и постулировать.

>> No.4864213  
Файл: kuri129.jpg -(267 KB, 700x1005, kuri129.jpg)
267

>>4864210
Не верю и не не верю, вообще плевать на всё это мероприятие с пруферами если честно!

>> No.4864227  
Файл: 0010-010-Zakon-iskljuchenija-tretego.jpg -(59 KB, 960x720, 0010-010-Zakon-iskljuchenija-tretego.jpg)
59

>>4864212

>А мне кажется, что это принципиально неформулируемое утверждение, потому что то, что семантика для языка является семантикой ты можешь только объявить и постулировать.

Что тогда эта семантика значит, если из нее нельзя вывести правила того, как пруфчекнуть (или иными словами, как проинтерпретировать) то, что в ней описано? Зачем и о чем она?
>>4864213

>Не верю и не не верю

Интуиционистская логика какая-то.

>> No.4864237  
Файл: kuri130.jpg -(265 KB, 800x600, kuri130.jpg)
265

>>4864227
Не очень понял вопроса, семантика - это всегда некоторые абстрактные сущности мира идей, именами которых термы твоего языка эти сущности помечают, есть терм "2+2" как трёхсимвольная строка, а есть та самая четвёрека из стандартной модели натуральных чисел, которая и помечается этим термом, как именем. (Конечно можно рассмотреть всё с тз метаязыка в котором и семантика и синтаксис будут в одной вселенной, но это тут не имеет значения). Семантику никак не пруфчекнуть, иначе она была бы синтаксисом, ты можешь придумать некоторый синтаксис, который бы твою семантику описывал и пруфчекнуть его. Но если два синтаксиса придумывались с тем, чтобы описывать разные семантики, то у тебя нету никакого способа проверить, хороша ли твоя интерпретация одного языка в другой или плоха, кроме того, чтобы пользуясь своим чутьем построить некоторую интерпретацию и постулировать, что она хороша и при всех последующих рассуждениях нужно сверяться именно с ней.

>Интуиционистская логика какая-то.

Да, сходство есть! Тут она очень естественно вылезла и понятна даже на бытовом уровне, действительно, почему я должен решать, верно ли в моём мире А или верно не А, если меня это не очень-то и волнует?

>> No.4864238  

>>4864227

> пик

Очень нехорошая формулировка, даже если это рассчитано школьников.

>> No.4864240  
Файл: kuri131.jpeg -(8 KB, 221x228, kuri131.jpeg)
8

>>4864238
А какая формулировка правильная?

>> No.4864245  
Файл: naushniki-sayonara-zetsubou-sensej-fuura(...).png -(294 KB, 1920x1080, naushniki-sayonara-zetsubou-sensej-fuura(...).png)
294

>>4864240
"Из высказывания и отрицания этого высказывания...", например.

Утверждения "У кафки рыжие волосы" и "У Кафки голубые волосы" противоречат друг другу, но оба ложны, потому что волосы Кафки -- черные.

В то же время, из утверждений "У Кафки рыжие волосы" и "У кафки не рыжие волосы" как раз верно ровно одно.

>> No.4864246  
Файл: kuri132.jpg -(20 KB, 236x344, kuri132.jpg)
20

>>4864245
Теперь ясно, постараюсь так и запомнить!

>> No.4864247  

>>4864245
Или у него нет волос!
так что нуже квалификатор - "у кафки есть волосы и ..."

>> No.4864251  
Файл: kuri133.jpg -(107 KB, 1280x880, kuri133.jpg)
107

>>4864247
А вдруг волосы не имеют цвета? Нужен ещё один квалификатор, вроде "у кафки есть волосы и они имеют цвет и ..."!

>> No.4864252  

>>4864251

И вообще может и сам Кафка не существует

>> No.4864254  
Файл: kuri134.jpeg -(7 KB, 189x267, kuri134.jpeg)
7

>>4864252
А обязательно ли нужно существовать, чтобы иметь волосы черного цвета? Что на этот счёт говорит логика?

>> No.4864255  

>>4864254

Наверно да. если предполагается что кафка может или не может существовать, значит надо писать утверждение что он существует

>> No.4864257  

>>4864254
https://math.stackexchange.com/questions/202452/why-is-predicate-all-as-in-allset-true-if-the-set-is-empty

>> No.4864259  
Файл: kuri135.jpg -(254 KB, 1280x720, kuri135.jpg)
254

>>4864255
Тогда лучше не предполагать, а то не люблю когда писать надо много!

>> No.4864260  
Файл: kuri136.jpeg -(5 KB, 301x167, kuri136.jpeg)
5

>>4864257
Интересный факт, постараюсь так и запомнить!

>> No.4864261  

>>4864257

здесь не all. если бы было "у всех кафок" тогда да, all.

>> No.4864262  
Файл: HG6cP0X.png -(100 KB, 443x336, HG6cP0X.png)
100

>>4864247
Существуют волосы такие, что они растут из головы Кафки, при этом они рыжие!

А отрицанием тогда будет:

Не существует волос таких, что они растут из головы Кафки, или же эти волосы не рыжие!

>> No.4864267  
Файл: kuri137.jpg -(574 KB, 700x951, kuri137.jpg)
574

>>4864262
Думаю нет, но спорить не буду!

>> No.4864274  
Файл: t1r7NFK[1].png -(273 KB, 495x522, t1r7NFK[1].png)
273

>>4864261

>"у всех кафок"
>> No.4864280  
Файл: Sayonara.Zetsubou.Sensei.full.113635.jpg -(42 KB, 1280x720, Sayonara.Zetsubou.Sensei.full.113635.jpg)
42

>>4864267
Я взял на себя труд развернуть отрицание конъюнкции двух утверждений в дизъюнкцию их отрицаний. Какие умные слова-то!

Не знаю, как это делают настоящие математики, но зато я точно знаю, что not (a and b) == not a or not b

А спорить очень весело, просто не все это понимают!

>> No.4864283  
Файл: slide_2.jpg -(67 KB, 960x720, slide_2.jpg)
67

>>4864237

>Не очень понял вопроса, семантика - это всегда некоторые абстрактные сущности мира идей, именами которых термы твоего языка эти сущности помечают, есть терм "2+2" как трёхсимвольная строка, а есть та самая четвёрека из стандартной модели натуральных чисел, которая и помечается этим термом, как именем.

Ну мы должны понимать, что именно закодировано в строке "2+2=4". Если семантика нам этого понять не позволяет, то непонятно вообще, что с этим делать. Вот например возьмем человека из прошлого, знающего допустим только римские цифры, но не знающего десятичной системы счисления с арабскими цифрами 0123456789, не знающего смыслов + и =. Что он сможет понять из строки "2+2=4"? Для него она не будет иметь вообще никакого смысла. Но если мы на понятном для него языке опишем ему смысл этого, то это тогда будет ему понятно, и он тогда сможет сказать, что это выражение, при таком смысле отдельных символов, является справедливым. Нужны синтаксис и семантика.
Вообще, что ты понимаешь под семантикой? Возьмем определение из вики https://ru.wikipedia.org/wiki/Семантика_(программирование)

>Семантика языка — это смысловое значение слов. В программировании — начальное смысловое значение операторов, основных конструкций языка и т. п.

Если твое определение таковым и является, то да, из знания синтаксиса и семантики будут следовать некие правила преобразования (переписывания), которыми можно доказать корректность выражения "2+2=4"
Например, путем последовательной трансформации "2+2=4" -> "3+1=4" -> "4=4" -> "4-1=4-1" -> "3=3" ... = "0=0" -> "TRUE"
Или я где-то ошибаюсь?

>> No.4864285  

>>4864261
Если множество всех Кафок пустое то это то же самое по смыслу, что и "Кафки не существовует".

>> No.4864297  
Файл: kuri138.jpeg -(172 KB, 700x1082, kuri138.jpeg)
172

>>4864280
Возьми отрицание от \exists x (P(x) /\ Q(x) /\ R(x)) где P(x) = "x - волосы", Q(x) = "x - растут из головы Кафки", R(x) = "x - рыжие" тогда! Мне наоборот кажется, что немногие понимают, что спорить - не весело!
>>4864283
Позже напишу, но если коротко, определение на вики мне не нравится. Под семантикой, как я говорил, понимают ситуацию, когда некоторые абстрактные объекты именуются термами некоторого языка, сами эти абстрактные объекты частью языка не являются. Есть "идея числа 4" - это семантика, а есть термы "2+2" или "IIII" или "IV" - это синтаксис. В некотором смысле это понятие метаматематическое, но его можно сделать математическим поднявшись на уровень метаязыка и тогда можно работать с языком как с объектом и, скажем, можно в ZFC записать утверждение в духе "натуральные числа N являются моделью для языка PA", впрочем если мы так сделаем, семантика самой ZFC, которая на уровне метаязыка всё равно останется понятием метаматематическим.

>> No.4864300  

>>4864262

>Не существует волос таких, что они растут из головы Кафки, или же эти волосы не рыжие!

Как-то слишком сложно. Лучше так:
Волосы на голове Кафки если и есть (их может и не быть!), то они точно не рыжие.
Вообще, вот интересный тест на логическое мышление: https://hr-portal.ru/pages/hu/logika.php

>> No.4864306  
Файл: kuri139.jpeg -(7 KB, 173x291, kuri139.jpeg)
7

>>4864300
Думаю неправильно, либо "не существует рыжих волос на голове кафки" либо "любой объект или находится не на голове кафки, или не рыжий, или не является волосами", но спорить не буду!

>> No.4864318  
Файл: images.jpg -(5 KB, 284x177, images.jpg)
5

>>4864297

> \exists x (P(x) /\ Q(x) /\ R(x)) где P(x) = "x - волосы", Q(x) = "x - растут из головы Кафки", R(x) = "x - рыжие"

"Для любого х: отрицание (P(x) /\ Q(x) /\ R(x))"
или же:
"Для любого х: отрицание (P(x) /\ Q(x)) \/ отрицание R(x)"

Но я не уверен, что предложенная тобой модель подходит. Например, тут не учтено, что волосы-то у Кафки на голове если и есть, то их не может быть несколько.

Попробуем так: "если множество волос, крепящихся к голове Кафки непустое, то это рыжее множество волос". Рыжее множество волос мы можем определить, например, по средней длине отраженных волн солнечного света, но это мелочи.

>> No.4864319  
Файл: kuri140.png -(125 KB, 474x600, kuri140.png)
125

>>4864318
Это эквивалентно тому, что "все волосы на голове кафки рыжие", потому что, как скидыввали тут пытливые умы, для пустого множества любое утверждение верно. В моих обозначениях это \forall x ((P(x) /\ Q(x)) -> R(x))

>Но я не уверен, что предложенная тобой модель подходит.

А подходит для чего? Мы какую-то цель преследуем? Я просто не в курсе!

>> No.4864333  
Файл: images (1).jpg -(8 KB, 180x280, images (1).jpg)
8

>>4864319
Так ты же придумал большую формулу с функциями и уголочками как аналог утверждения про рыжесть Кафки!

> Это эквивалентно тому, что

Что -- "это"? Я не понимат. Видишь как весело!

>>4864300
Да, скорее всего это подходит. А этот тест мне уже кидали, он показался мне скучным.

>> No.4864339  
Файл: kurisu141.jpeg -(6 KB, 212x238, kurisu141.jpeg)
6

>>4864333
Действительно я придумал!

>> No.4864368  

>>4864333
Тебе вообще надо более четко определить, что конкретно значит твое утверждение.
Ну вот допустим: "Существуют волосы такие, что они растут из головы Кафки, при этом они рыжие!"
Волосы целиком все одного цвета могут быть, или впринципе возможно такое, что часть волос будет рыжими, а другая часть будет иметь другой цвет?
Ну т.е. допустим у нас есть множество всех рыжих волос. Среди этого множества есть непустое подмножество, для которого справедливо, что эти волосы растут на голове кафки. Но из этого вообще не следует, что у Кафки на голове есть только рыжие волосы, волос другого цвета нет.

Если мы хотим получить отрицание, можно сказать так: "подмножество из множества всех рыжих волос, для которого было бы верно, что они растут из головы Кафки - пустое!". Или "нет таких рыжих волос, которые б росли из головы Кафки"

>> No.4864376  

>>4864237

> Но если два синтаксиса придумывались с тем, чтобы описывать разные семантики, то у тебя нету никакого способа проверить, хороша ли твоя интерпретация одного языка в другой или плоха, кроме того, чтобы пользуясь своим чутьем построить некоторую интерпретацию и постулировать, что она хороша и при всех последующих рассуждениях нужно сверяться именно с ней.

В >>4864054 предлагают не интерпретацию одного языка в другой, сохраняющую семантику, а преобразование синтаксиса, сохраняющее не имеющее отношения к семантике свойство корректности доказательств. То есть чисто формальную доказанную сводимость удобного языка к простому языку, позволяющую использовать удобный язык и иметь столько же уверенности (если строить уверенность на простоте), как если бы это был простой язык (если, конечно, само доказательство сохранения корректности достаточно простое и хорошо поддаётся проверке вручную).

>> No.4864377  
Файл: kuri143.jpg -(8 KB, 236x236, kuri143.jpg)
8

>>4864376
Не знаю что такое "свойство корректности доказательств", хотт не о доказательствах, а о высших индуктивных типах, а в metamath естественным образом никаких высших индуктивных типов нету.

>> No.4864378  
Файл: yande.re 502269 animal_ears fate_grand_o(...).png -(1951 KB, 1280x2150, yande.re 502269 animal_ears fate_grand_o(...).png)
1951

>>4863704
А по каким материалам можно начать изучение математики, если отрывками знания есть и со школы, и с университета, но много пробелов и хотелось бы получить хороший уровень знаний математики и понимания тоже?

>> No.4864383  
Файл: kuri144.jpg -(29 KB, 200x398, kuri144.jpg)
29

>>4864378
Гельфанд Шень "Алгебра" для вката, если всё оттуда знаешь, то наверное какой-то хороший учебник по алгебре типа aluffi algebra: chapter 0. Я не знаю, на твоём месте я бы больше кусочно развивался, читай блоги, матоверфлоу, гугли какие-нибудь обзоры по темам, о которых слышал краем уха, смотри лекции на лекториуме и тд и тд

>> No.4864384  

>>4864383
Просто мне бы желательно для машин-лёнинга всякого, нейронок и т.д...
Если начну учить мат.статистику, то могу чего-то не понять из-за того, что не понимаю каких-то базовых вещей.
Если начну читать какой-нибудь геометрический смысл какой-то темы (для лучшего понимания со всем сторон), могу не понять из-за того, что не знаю геометрию почти совсем. Тригонометрию тоже, хотя всякие тригонометрические функции всегда можно увидть в различных теориях (и уже не в математике, а в темах, где она применяется).

>> No.4864391  
Файл: kuri145.jpeg -(6 KB, 168x300, kuri145.jpeg)
6

>>4864384
Я бы советовал решать проблемы по мере их возникновения и подучивать какие-то темы когда ты обнаружишь, что твоего знания фатально не хватает. Посмотри khan academy или brilliant org, там весь энтрилевел довольно неплохо собран. А в учебниках по математике не для математиков плохо разбираюсь, к сожалению.

>> No.4864396  

>>4864391
Спасибо за советы.

>> No.4864433  
Файл: kurisu146.jpeg -(14 KB, 168x300, kurisu146.jpeg)
14

Напишите вопросы какие-нибудь, а я утром посмотрю и мне будет очень приятно!

>> No.4864464  
Файл: DzMcqUXWsAAWKcN.png -(322 KB, 750x421, DzMcqUXWsAAWKcN.png)

>>4864377

>Не знаю что такое "свойство корректности доказательств", хотт не о доказательствах, а о высших индуктивных типах, а в metamath естественным образом никаких высших индуктивных типов нету.

В metamath естественным образом вообще почти ничего нет, кроме переписывания.
The complete specification of the Metamath language is only four pages long (Section 4.1, p. 106). http://us.metamath.org/downloads/metamath.pdf#page.106
Все эти высшие индуктивные типы там тоже вполне представимы. Теория зависимых типов для Metamath описана: https://github.com/digama0/dtt.mm . HoTT этот тоже можно описать, тут нет никакой проблемы. Зависимые типы и этот HoTT могут добавить выразительности, но разве они позволяют доказать что-то, что без них доказать никак нельзя (не считая доказательств, связанных с самой теорией зависимых типов и HoTT) ? Т.е. например если доказывать теорему Пифагора, то это можно доказать хоть с HoTT, хоть без HoTT, просто с этой теорией типов это может быть удобнее, и можно например проще обобщить теорему пифагора на более общий случай (например пространственная теорема пифагора, можно и для 4-5-6 мерного пространства что-то такое описать, пользуясь зависимыми типами для координат точек в n-мерном пространстве, которые специализируются (зависимы от) "мерностью" пространства, точки в котором они описывают).

>> No.4864549  
Файл: IMG_20190126_115717.jpg -(85 KB, 1280x632, IMG_20190126_115717.jpg)
85

>>4864433
Можешь объяснить значение теорем Гёделя о неполноте? Я правильно понимаю, что теоретически внезапно может оказаться, что большая часть знаний о математике не верна? И есть ли такие направления в математике, на которые эти теоремы не влияют?

>> No.4864599  
Файл: fuura_kafuka___vector_by_raver_mew2007.png -(103 KB, 622x350, fuura_kafuka___vector_by_raver_mew2007.png)
103

>>4864368

> Волосы целиком все одного цвета могут быть, или впринципе возможно такое, что часть волос будет рыжими, а другая часть будет иметь другой цвет?

Мне кажется, что необходимо рассмотреть множество всех волос на голове, а остальное -- вопрос технический.

Например, мы можем считать рыжим такое множество волос, доля рыжих волосков в которых превышает определенный порог. Или же усреднить спектр излучения по каждому волосу и найти результирующий цвет.

Или вообще, определять цвет волос по паспорту. Как пол или год рождения, только цвет волос. И заморачиваться не придется!

>>4864384
Верь в себя и решай проблемы по мере их возникновения! Я бы еще посоветовал тебе освежить знания в линейной алгебре, чтобы работать с векторами и матрицами.

А так, в ML есть готовые библиотеки, порой позволяющие решать задачи, вообще не зная, как это внутри работает.

>> No.4864641  
Файл: kuri147.jpg -(70 KB, 706x800, kuri147.jpg)
70

>>4864464
Конечно всё можно сэмулировать на всём, мне просто не кажется это интересным и не кажется что эмуляция на метамасе должна добавлять мне больше уверенности, чем на С. Про теорему пифагора - принципиально наверное можно, но будет довольно сложно доказывать вещи такого толка, по крайней мере я с наскоку не знаю как, HoTT она больше под всякую топологию заточена скорее.
>>4864549
Теоремы Гёделя говорят, что истинность не тождественна доказуемости, и что в принципе есть недоказуемые утверждения, которые тем не менее истинны. В принципе то, что найдётся противоречие в арифметике Пеано, скажем так, ничему не противоречит, то есть ничему не противоречит что используя всякие простые рассуждения про натуральные числа кто-то получит, что 0=1. Но на такое надеется было бы слишком оптимистично, всё-таки метаматематические интересности такого большого размаха - это не нечто, что находится каждый день. Нет, теорема Гёделя сугубо метаматематический эффект, свойственный любому достаточно выразительному синтаксису, поэтому просто так от неё отмахнуться не получится.

>> No.4864719  
Файл: 1547496768711.jpg -(304 KB, 1943x1619, 1547496768711.jpg)
304

>>4863804

>полезный

Практически бесполезный для человека, который не занимается этим профессионально каждый день.
Все расчудесные знания выветрятся без упорной практики.
>>4863803
Смысл типа "интересоваться", если

>Все расчудесные знания выветрятся без упорной практики.

?

>> No.4864725  
Файл: kuri148.jpg -(36 KB, 200x410, kuri148.jpg)
36

>>4864719
А почему у тебя такое сильное желание удерживать всё то, что ты однажды узнал?

>> No.4864731  
Файл: 1548666283652.png -(694 KB, 720x720, 1548666283652.png)
694

>>4864725
Зачем изучать особо изощренную математику, если ты не хочешь внести свой личный вклад либо искусно приложить математику к науке?

>удерживать всё то, что ты однажды узнал

Тот же вопрос. Зачем? Надеюсь, ты не будешь оправдывать бесплодное умственное напряжение примитивным гедонизмом.

>> No.4864734  
Файл: kuri141.jpg -(1186 KB, 1250x2000, kuri141.jpg)
1186

>>4864731
За тем же, зачем и некоторые люди годами изучают лор каких-нибудь звёздных воин или властелина колец, думаю.

>> No.4864738  

>>4864731

>>удерживать всё то, что ты однажды узнал
>Тот же вопрос. Зачем?

Стоп, что, это же ты выражал непонимание по отношению к людям, которые могут интересоваться чем-то даже осознавая, что у них это что-то может выветрится, так что вопрос "зачем" нужно адресовать тебе. Почему тебе так важно, чтобы знание которое ты узнаешь не выветривалась, и ещё более важно, чтобы ты при помощи этого знания мог выдать некоторый продукт? Продуктов, нынче, по пять копеек за кило на рынке, кому они нужны.

>> No.4864756  

>>4864738
Глупости. Во-первых, у тебя не настроен детектор. Во-вторых, "продукты по пять кило" это дикая редукция, практически нигилизм по отношению к интеллектуальным ценностям. Предположу, что ты это понаписал лишь бы вставить свои пять копеек дешёвого развесного мнения Сырны, но эти мнения может высказать любой гимназист, а вот решение актуальной задачи либо какое-то другое принципиально новое знание под ногами не валяется. Вот и назревает вопрос: зачем интересоваться просто так, не пытаясь ничего достичь и не ставя целей, упиваясь посредственным удовлетворением праздного любопытства? Это уровень NPC, а не человека мыслящего.

>> No.4864763  
Файл: kuri149.jpeg -(11 KB, 199x253, kuri149.jpeg)
11

>>4864756
Терпеть не могу интеллектуальные ценности, человеков мыслящих и актуальные задачи. Глупости так глупости, что же поделать, если ты не хочешь меня понимать!

>> No.4864769  
Файл: 15425627537690.jpg -(34 KB, 304x429, 15425627537690.jpg)
34

>>4864763
Хм, думаю здесь ты прав. Извини, что побеспокоил.

>> No.4864772  
Файл: kuri150.jpeg -(8 KB, 300x168, kuri150.jpeg)
8

>>4864769
Можешь беспокоить и дальше, только не требуя от меня при этом, чтобы я тебя в чём-то убеждал! Хотя можешь и требовать, просто боюсь что отказ ты воспримешь на свой счёт и обидешься, а я этого не хочу!

>> No.4864778  

У меня в вузе есть матан и дискретка до нее была линейка, и если с матаном все понятно, в любом учебнике все рассказано и доказано, то со вторыми двумя предметами я сладить не могу. Читаю и просто не понимаю что происходит. Может посоветуете какой нибудь хороший и понятный учебник?

>> No.4864792  
Файл: kuri152.jpeg -(3 KB, 251x201, kuri152.jpeg)
3

>>4864778
Ландо: Введение в дискретную математику и Axler: Linear Algebra Done Right, не знаю насколько они помогут или не помогут тебе вкатиться, но учебники хорошие. Посмотри ещё видосы 3blue1brown по линейной алгебре, там анимации красивые и несколько важных интуиций объяснено.

>> No.4864802  
Файл: hello_html_m668420c.jpg -(34 KB, 500x229, hello_html_m668420c.jpg)
34

>>4864641

>HoTT она больше под всякую топологию заточена скорее.

А что именно ты доказываешь в рамках этой HoTT и зачем это вообще нужно? Иными словами, какие именно практически полезные вещи мы от этого получаем, как это можно применить?
Вот например есть криптография на эллиптических кривых, это что-то практически применимое, т.е. этой криптографией можно шифровать файлы. А что с HoTT и топологией?

>> No.4864819  
Файл: kuri154.jpg -(107 KB, 1280x720, kuri154.jpg)
107

>>4864802
В HoTT можно доказывать всё то, что и в любой другой математике, просто ощутимый профит от неё появляется в мягких вещах, связанных с рассмотрением разного типа структур с точностью до изоморфизма или гомотопии или какого-то такого мягкого отношения эквивалентности. HoTT и топологию через запятую не очень правильно ставить, потому что они о разном, первое - это о программной проверке мат. доказательств, а второе - это математика как она есть.

Топология редко нужна для прикладных вещей, она, по отношению к прикладным вещам, играет более описательную роль, чем вычислительную. Но с этой задачей она справляется неплохо, даже вон нобелевку по физике в 16 дали за формализм описывающий всякие toplogical phases of metter, не совсем понял о чём это, но выглядело не оч интересно, поэтому я не учил. Кроме фундаментальной физики приложений не знаю. (Если только всякие доказательство существования и единственности всяких диффуров через какие-то техники типа h-принципа или лефшеца не считать за приложения, но я в доказательствах существования ничего прикладного не вижу)

>> No.4864913  
Файл: kuri155.jpeg -(11 KB, 189x267, kuri155.jpeg)
11

Сегодня всю ночь не сплю, поэтому можем болтать!

>> No.4864930  

>>4864913
Ну ок, раз тут затрагивалась тема с доказывателями теорем, то какой софт для математиков еще нужен? Чем ты сам лично пользуешься, и что можешь посоветовать для своей области? Есть например Wolfram Mathematica, Maple, MATLAB, Maxima, Sage. Ты занимаешься исключительно проблемами, связанными с топологией, или есть что-то еще?

>> No.4864936  
Файл: kuri155.png -(1086 KB, 1500x835, kuri155.png)
1086

>>4864930
Я не занимаюсь проблемами связанными с топологией, я начал говорить о ХоТТ потому что так разговор зашёл, я занимаюсь абстрактной алг.геометрией и теорией представлений. Я никакой софт не использую, так что ничего не могу посоветовать, пару раз использовал вольфрам математику чтобы домашки делать на каких-то младших курсах и всё.

>> No.4864950  

>>4864936

>я занимаюсь абстрактной алг.геометрией и теорией представлений.

А где на практике применима алг.геометрия и теория представлений? Я сходу могу назвать только криптографию на эллиптических кривых. Понимаешь ли ты, каким образом работает эта криптография, или ты занимаешься чем-то другим, не связанным с этим?
Как ты осознал, что хочешь по жизни заниматься именно математикой?

>> No.4864951  

>>4864936

> Я никакой софт не использую

Даже LaTeX? Или таки Word? (Понятно, что это не совсем тот софт, о котором спрашивали, но всё-таки в чём пишешь?)

>> No.4864958  
Файл: kuri156.jpg -(14 KB, 236x419, kuri156.jpg)
14

>>4864950
Понимаю, но занимаюсь чем-то другим, не связанным с этим, криптография на эл.кривых использует алг.геометрию довольно примитивного уровня. Не знаю, в какой-то момент решил, что люблю её и стал ею заниматься. Про приложения - да, наверное только во всяких кодированиях и криптографии.
>>4864951
Латех на оверлифе.

>> No.4864966  
Файл: map of mathematics.jpg -(521 KB, 1024x721, map of mathematics.jpg)
521

Що у меня нашлось одначе.

>> No.4864968  
Файл: kuri157.jpg -(15 KB, 236x324, kuri157.jpg)
15

>>4864966
Что кто-то жив я рад, но карта глупая!

>> No.4864979  

>>4864966
Я бы поделил на Bullshit и Usable instrument.

мимохейтер

>> No.4864981  
Файл: turing1.jpg -(7 KB, 280x144, turing1.jpg)
7

>>4864641

>Теоремы Гёделя говорят, что истинность не тождественна доказуемости, и что в принципе есть недоказуемые утверждения, которые тем не менее истинны.

Мне кажется, ты сильно упрощаешь. Например, имеют ли какое-то отношения теоремы Гёделя к парадоксу Рассела, канторову диагональному процессу и проблеме останова для машины Тьюринга? Мне кажется, имеют. https://web.archive.org/web/20170215130846/https://inf.1september.ru/2000/5/art/turing.htm
Только связь не очень очевидна. Походу, неразрешимость проблемы останова как раз следствие этой теоремы Гёделя о неполноте, и наоборот! А понять неразрешимость проблемы останова куда проще

>> No.4864982  

>>4864981
Знаешь, что делают, когда кажется?

>> No.4864983  
Файл: inverted_halting_machine.jpg -(24 KB, 573x249, inverted_halting_machine.jpg)
24

>>4864982
Зачем же что-то делать, если можно погуглить? https://cs.stackexchange.com/q/419 https://www.scottaaronson.com/blog/?p=710

>> No.4864985  

>>4864983
Во-первых, вопрос переадресовывается >>4864981. Во-вторых, это более слабое утверждение:

>suppose we had a sound and complete (and recursively-axiomatizable, yadda yadda yadda) formal system F, which was powerful enough to reason about Turing machines
>> No.4865008  

>>4864979
Забавляют такие ребята. Особенно когда они не разбираются в предмете. Нет, я тоже не разбираюсь, но ведь я и не делю. Зато я в курсе, что многие современные технологии ещё век назад были "ненужным теоретическим булщитом без применения", а полвека назад - "мечтами романтиков без возможности практической реализации".

>> No.4865015  

>>4864966
Это что-то похлеще китайской классификации животных от Борхеса.

>> No.4865018  

>>4865015
Очень хорошая классификация, кстати.

>> No.4865053  

>>4864966

>Оптимизация и теория меры в разных сторонах рисунка
>Множество мандельбродта рядом с топологией
> Откуда-то на рисунке математическая физика, химия и биология.

Могли бы и экономику включить, почему нет?

Уноси где взял.

>> No.4865068  

>>4865053

>Могли бы и экономику включить, почему нет?

А она там есть

>> No.4865070  

>>4865068
Тогда надо аниме добавить.

>> No.4865101  
Файл: mapofmath.jpg -(268 KB, 1247x1346, mapofmath.jpg)
268

>>4864979
Я бы делил на derived categories и bullshit. :з
>>4864981
Да конечно оно связано, вообще есть такая шутка, что самореферентность - это единственный инструмент для генерирования теорем в мат.логике. Но гёделя из проблемы останова действительно не очень тяжело вывести: достаточно запустить программу, которая запускает текст программы и одновременно ищет доказательство того, что эта самая программа не остановится в (скажем) формальной арифметике, если арифметика полна, то либо когда-то программа остановится из-за того, что мы её запустили, либо когда-нибудь будет найдено доказательство того, что она не остановится, то есть мы решили проблему останова, но мы же знаем что она не решается, значит арифметика неполна. С высокой точки зрения это всё инстанции изоморфизма карри-говарда, который говорит, что мат.логика и теория вычислимости это примерно о том же самом. Рассел это тоже по структуре аргумент в духе "самоприменимость всё ломает", поэтому ощущается похожим образом, но как его чисто формально свести к проблеме останова или гёделю не очень представляю.
>>4864985
Прям уж более слабое!

>>4865053>>4864966
Вот карта получше, только курису из-за неё не влезла.

>> No.4865155  
Файл: kuri158.jpeg -(6 KB, 270x187, kuri158.jpeg)
6

Я бампну тред пусть тут так и не принято!

>> No.4865158  

Я учился в ВУЗе по специальности "информатика и вычислительная техника", там было очень немало фундаментальных математических курсов, но нигде даже не упоминали про теорию категорий. И, насколько я знаю, на математическом факультете там же тоже.
Не знаю, везде ли так, но почему?

>> No.4865160  
Файл: kuri160.jpg -(25 KB, 362x512, kuri160.jpg)
25

>>4865158
На хороших факультетах вроде рассказывают. На самом деле если совсем фундаментальных курсов не было, а были стандартные матан линал тфкп функан дискртека, и не была курса по функ. программированию, то она не очень и нужна.

>> No.4865161  

>>4865160
Функциональное программирование было по выбору (на Лиспе и на F#). Была математическая логика, логические основы программирования, рассказывали про лямбда-исчисление, про частично рекурсивные функции (и про машину Тьюринга само собой). Но про категории умолчали.

>> No.4865164  
Файл: kuri160.jpeg -(6 KB, 225x225, kuri160.jpeg)
6

>>4865161
Плохо сделали, выходит!

>> No.4865168  

>>4865160
А сам ты на чем-нибудь умеешь программировать?

>> No.4865169  

>>4865164
На самом деле у меня этот вопрос всплыл, потому что я случайно наткнулся (не спрашивайте как) на печатную статью из журнала 1983 года под заголовком "О природе программирования" (хотя я её целиком не прочитал). Так там уже упоминается теория категорий. Значит нельзя сослаться на то, что курсы устаревшие, потому что даже в устаревших уже могло бы это быть. Вот мне интересно, почему тогда.

>> No.4865172  
Файл: kuri161.jpg -(196 KB, 745x887, kuri161.jpg)
196

>>4865168
Да, умею, на плюсах, питоне, джаваскрипте, хаскелле и немного на агде и лине - немного пробовал их реализацию hott, но я скорее на upper-любительском уровне, а не профессиональном. Немного интересовался спортивным программированием и всякими алгоритмическими головоломками, поэтому даже знаю стандартный набор алгоритмов для этих самых соревнований.

>>4865169
Не знаю даже! Выучи её не в университете, если тебе интересно!

>> No.4865173  

>>4865169
Наверно потому, что большинству программистов никогда эта теория категорий не понадобится. Я вот программирую микроконтроллеры на Си, и отлично себя чувствую без знаний всех этих теорий категорий.

>> No.4865176  

>>4865172

> Выучи её не в университете, если тебе интересно!

Ну, мемы про зигохистоморфные препроморфизмы уже выучил, этого достаточно, когда-то мне объяснили, что это за морфизмы такие, но я не смог запомнить.

>> No.4865177  

>>4865173
Если бы ты писал, например, на Rust, то мог бы использовать тип Result, который удовлетворяет монадическим законам, для более удобной, чем в C, обработки ошибок. Хотя для этого, в общем-то, не нужно знать, что он им удовлетворяет. И тем более не нужно про то, что монада это моноид в категории эндофункторов.

>> No.4865178  
Файл: kuri164.jpg -(440 KB, 1494x2094, kuri164.jpg)
440

>>4865173
Это да, обидно наверное, когда учишь нечто, а потом оно тебе не понадобляется.
>>4865176
Ну это какие-то хитрые технические изыски, а если хочешь основной панчлайн, то почитай про syntax-sematics duality где-нибудь, и про категорные семантики каждой вершины лямбда-куба.

>> No.4865185  

>>4865172

>Да, умею, на плюсах, питоне, джаваскрипте, хаскелле и немного на агде и лине - немного пробовал их реализацию hott, но я скорее на upper-любительском уровне, а не профессиональном

"лине" - это Gallina из Coq? Как соотносится программирования и математика, если говорить о языках неспециализированных (не Agda, не Gallina, а просто обычные языки для обычных программистов, вроде той же джавы)? Помогают ли они понимать математику, и помогает ли математика понимать их?

>> No.4865187  
Файл: kuri165.jpg -(30 KB, 512x407, kuri165.jpg)
30

>>4865185
Не, майкрософтовский Lean https://leanprover.github.io/about/

Ну сорт оф, есть мат. доказательства выполненные при помощи пруф ассистантов (в том числе и довольно важные, вроде классификации простых конечных рупп), есть вычисления, которые дают важные интуиции; с другой стороны, есть теория сложности вычислений и теория вычислимости + постоянно ведутся разработки всяких хитрых теорий типов то есть, де факто, довольно высокоуровневых языков программирования, ну и есть деятельность напрямую связанная с придумыванием всяких алгоритмов для всяких шифрований, есть всякие подходы с использованием какой-никакой теории вероятностей для анализа и реконструкции данных и тд и тд

>> No.4865218  
Файл: recursive_sequence_2.jpg -(10 KB, 189x84, recursive_sequence_2.jpg)
10

>>4865187
А есть какие-нибудь книжки для тех, кто хорошо разбирается в программировании, но плохо разбирается в математике, чтоб в них описывалась математика языком компьютерных программ? Чтоб например вместо пикрилейтеда, числа фибоначчи описывались как-нибудь так
unsigned int fib(unsigned int n)
{

return (n == 0 || n == 1) ? 1 : fib(n - 1) + fib(n - 2);

}
или:
unsigned int numbers[10] = {1,1};
for (size_t i = 2; i < 10; ++i)
{

numbers[i] = numbers[i-1] + numbers[i-2];

}
Потому что многие программисты не понимаю весь этот математический синтаксис. Где вообще пожно про него почитать? Вот например кто-то пишет ab(b+c) и это надо понимать как a×b×(b+c), а когда мы пишем cos(x) то мы уже не понимаем это как c×o×s×(x) и даже не понимаем это как некую cos×(x). А когда проходят интегралы ∫x²dx (в TeX это будет \int_{}^{}{x^2}dx ) то вообще неясно, можно подумать что мы берем интеграл от некоторого выражения "x²×d×x" хотя на самом-то деле dx это элемент интегрирования. Почему так запутано?

>> No.4865223  

>>4863840
Алло, а как юзать тригоном формулы в комплексных, без тригонома?

>> No.4865228  

>>4863878

>о том, как что-то конкретное закодировать и вопросы типа "можно ли придумать какой-то ещё синтаксис, чтобы кодировать стало ещё удобнее" мне кажется всё же больше по части CS

А разве cs не о алгоритмах и структурах данных, есле да, то при чем тут кодирование

>> No.4865230  
Файл: kuri166.jpeg -(9 KB, 183x275, kuri166.jpeg)
9

>>4865218
Первый раз такой странный реквест слышу, синтаксис абузят в целях удобночитаемости. В твоём примере с интегралом мы и так берём интеграл от выражения (диф.формы) x^2 × d(x), диф.формы и функции ведут себя существенно разным образом при замене переменных, поэтому приписка dx тут существенна.

Единственное что по сути реквеста помню, была такая книжка Contemporary Abstract Algebra от Gallion (или как-то так автора звали, думаю найдешь), там было некоторое количество упражнений, делать которые нужно было на компьютере. Ещё на курсере был курс по лин.алу для программистов тоже почти весь на компьютере, но все объяснения всё равно делались в привычном для всех синтаксисе, думаю, тебе придётся к нему привыкнуть поверив в то, что как привыкнешь - станут очевидны его профиты.
>>4865223
Не понял о чём ты!
>>4865228
Думаю, что есть CS об алгоритмах и структурах данных, а есть CS о кодировании мат.доказательств в термы какого-то синтаксиса, а есть CS о чём-то ещё - наверняка!

>> No.4865231  

>>4865230
Как ты собираешься считать геометрию в комплексных без тригонома?

>> No.4865232  
Файл: kuri167.jpg -(190 KB, 2400x1348, kuri167.jpg)
190

>>4865231
Выведу более коротким способом через комплексные все нужные мне соотношения и буду считать, как считал и раньше, когда выводил их более длинным способом, думаю.

>> No.4865235  

>>4865218
Лучше предложи сразу всю математику на Лисп заменить.

>> No.4865237  

>>4865218
Часто всё-таки cos пишут другим шрифтом, чтобы было понятно, что это один токен.

>> No.4865241  

>>4865218
Лучше бы постыдился такой код показывать!

>> No.4865273  

>>4865235
Зачем ограничиваться математикой? Все инструкции писать кодом.

>> No.4865277  
Файл: sicp anime.jpg -(99 KB, 600x338, sicp anime.jpg)
99

>>4865218

>Потому что многие программисты не понимаю весь этот математический синтаксис

Программисты прекрасно понимают весь этот синтаксис, поскольку кончали и старшую школу и университет, не надо тут наговаривать.

>Где вообще пожно про него почитать?

В школьных учебниках по математике. Ты как с луны свалился.

>> No.4865294  

>>4865237

>Часто всё-таки cos пишут другим шрифтом, чтобы было понятно, что это один токен.

Хорошо, что мне ни в языках программирования не приходится писать cos другим шрифтом, чтобы было ясно, что это не c×o×s а именно функция cos
>>4865277

>Программисты прекрасно понимают весь этот синтаксис, поскольку кончали и старшую школу и университет, не надо тут наговаривать.

Есть те, которые не понимают.
Язык математики, в отличии от языка программирования, допускает неоднозначность интерпретации написанного на нем. Вот я приводил простой пример с косинусом. Если мне надо записать выражение "косинус от a плюс b" то я напишу cos(a+b) и всё будет понято правильно, а если мне надо написать "c умножить на o умножить на s и умножить на выражение a плюс b" то я не могу это записать как cos(a+b) потому что меня могут неправильно понять, и мне надо записывать cos другим шрифтом или придется записывать как-нибудь по-другому, явно обозначая умножения т.е. c×o×s×(a+b). В языках программирования не возникает подобных неприятных неоднозначностей, там не надо что-то писать другими шрифтами, чтобы мою запись можно было правильно проинтерпретировать.

>В школьных учебниках по математике. Ты как с луны свалился.

Я не помню чтоб в учебниках математики учили, что чтоб было понятно, что последовательность латинских букв "cos" это именно косинус, то надо записывать это каким-то особым шрифтом в тетрадке, там видимо просто расчитывают на то, что у тебя переменные не будут называться c o s и ты не будешь их перемножение записывать как "cos". Там вообще нет формального определения синтаксиса и семантики математических выражений и точного, однозначного способа интерпретировать их!

>> No.4865295  

>>4865294
Эту проблему с неоднозначностью можно было бы решить, если б математики все внезапно договорились бы, что теперь давайте писать cos(x) как {cos}(x), log(x) как {log}(x) и так далее, и тогда было бы понятно, что выражение в {} скобках это название функции, которую применяем к выражению в () скобках, но вот по-моему такого никогда не произойдет.

>> No.4865296  
Файл: fuck that cat.gif -(2083 KB, 480x270, fuck that cat.gif)
2083

>>4865294

>Есть те, которые не понимают.

Нет. Те кто не понимают и не в силах понять - кодомакаки, а не программисты.

>Язык математики, в отличии от языка программирования, допускает неоднозначность интерпретации написанного на нем.

Что ты несешь?! Дальще даже и читать не буду.

>> No.4865298  

>>4865294

>в отличии от языка программирования, допускает неоднозначность интерпретации написанного на нем.

Что такое undefined behavior?

>> No.4865299  

>>4865296

>Что ты несешь?! Дальще даже и читать не буду.

А что не так? Каким образом мне понять, что если кто-то на бумажке рукой написал выражение cos(x+y), то cos это именно косинус, а не c умножить на o умножить на s?

>> No.4865300  

>>4865298

  1. Есть языки, в которых UB нет
  2. Если UB все же есть, то ситуации с UB определены в стандарте соответствующего языка. Где мне почитать официальный стандарт на язык математики?
>> No.4865307  

>>4865299
Таким же образом, как и все остальные надписи -- из контекста.
Ты решил покормиться на тупости?

>> No.4865309  

>>4865300

Попробуй взглянуть на своё существование более философски: отвлечься от истинности/ложности или важности/неважности своего мнения и задуматься, что вообще происходит. Человек, видящий перед собой только математические закорючки, при виде человеческого языка превратится в формалиста (ведь там нет закорючек, которые он видит 99% времени; видящий идеи за закорючками может не превратиться). В этом поколении уже программисты пытаются "взгромоздиться" на математиков: прямо N десятков минут назад видел, как кто-то заметил что в математике ещё и не должно быть "неопределяемых понятий", а должно быть всё как в программировании. Суть в том что это не процесс мышления, а процесс подобный схождению оползней грязи после дождя. Перекладывание слов изо рта в рот, где участники не субъекты а объекты процесса

(Хотя ты вроде просто спросил что-то, но тогда сам виноват что начал говорить так будто что-то утверждаешь)

>> No.4865310  

>>4865300
Ну то есть в реальности таки порядочно ситуаций где написано так, а что выполнится зависит от компилятора, системы или вообще рандома.

>> No.4865311  

>>4865300

>Где мне почитать официальный стандарт на язык математики?

Попробуй с Бурбаки начать.

>> No.4865421  
Файл: kuri168.jpg -(21 KB, 235x364, kuri168.jpg)
21

Вверх!

>> No.4865426  
Файл: MivJadc.png -(149 KB, 402x501, MivJadc.png)
149

>>4865421

>> No.4865523  
Файл: kuri169.jpeg -(13 KB, 162x310, kuri169.jpeg)
13

Сегодня снова не сплю, походу

>> No.4865526  
Файл: question_sophie_3.jpg -(122 KB, 1280x720, question_sophie_3.jpg)
122

Какие есть полностью асимметричные выпуклые правильносторонние четырехмерные многоячеечники?

>> No.4865537  
Файл: kuri170.jpeg -(9 KB, 200x251, kuri170.jpeg)
9

>>4865526
А что значит "полностью асимметричный"? Просто группа симметрий тривиальна (как у множества) или что-то посильнее? Я думал что все правильносторонние политопы ещё в начале XX века классифицировали.

>> No.4865543  

>>4865537
Ну в трехмерном случае это будут многогранники Джонсона. И среди них нету такого чтобы группа симметрий была тривиальна, хоть какое-нибудь отражение или поворот найдётся. Не совсем очевидно почему, но так получилось.
А вот если обобщить на четырехмерные многогранники с правильными сторонами, то найдётся ли асимметричный? Там вариантов сильно больше, поэтому почему бы нет?

>> No.4865550  

>>4865546
Осталось их всех перерисовать в анимешных девочек. Наращенных и усеченных!

>> No.4865554  
Файл: kuri171.jpg -(80 KB, 1500x2200, kuri171.jpg)
80

>>4865543
http://mathieudutour.altervista.org/Regular/index.html так вот классификация всех правильносторонних политопов во всех размерностях нужно аккуратно чекать, но вроде 0_21+Pyr(октаэдр) удовлетворяет твоему реквесту, так как 0_21 не имеет осевых симметрий, а после того как мы одну из граней видоизменили, то каких-то других симметрий, которые меняют грани местами остаться не должно. Там же z-структура выписана полностью - сядь да посчитай группу симметрий, что мешает!

>> No.4865572  

>>4865554
Интересно, спасибо за наводку. Вообще это получается аналогично полностью усеченному пятиячеечнику у которого одна октаэдральная грань остается на месте. Сейчас прикину, можно ли ее вертеть на месте не мешая остальному.

>> No.4865620  
Файл: kuri172.jpg -(15 KB, 236x333, kuri172.jpg)
15

>>4865613
Режим сбит и работать надо.
Никак, это открытая проблема, такое свойство (ну, почти такое) называется "нормальность числа в десятичной системе", на сегодняшний момент нету никаких подходов доказывать нормальность каких-нибудь не искуственно придуманных чисел, насколько мне известно.

>> No.4865637  

>>4865620
А в чем заключается твоя работа как математика? Ты доказываешь какие-то теоремы, или выводишь какие-то свойства, или что?

>> No.4865643  
Файл: kuri173.jpeg -(7 KB, 184x274, kuri173.jpeg)
7

>>4865637
Доказываю какие-то теоремы и вывожу какие-то свойства.

>> No.4865655  

>>4865643
Ну а можешь поподробнее объяснить, что означает быть математиком? Надо регулярно какие-то научные журналы по математике читать, читать какие-то чужие бумаги, писать и отправлять свои бумаги на проверку, и кто-то потом говорит что у тебя вот там и там ошибка, и ты потом это исправляешь? Или как это всё происходит примерно?

>> No.4865657  
Файл: kuri174.jpg -(18 KB, 225x350, kuri174.jpg)
18

>>4865655
Пишу бумагу, показываю научному руководителю, если он говорит что всё в порядке, то отправляю на пир-ревью в тот журнал, который он мне советует, если там говорят что всё в порядке, то через некоторое время меня публикуют и я получаю небольшое количество символического капитала.

>> No.4865665  

>>4865657
А если кто-то занимается математикой на любительском уровне и у него нет никаких научных руководителей, но он совершил (или думает что совершил) какое-то серьезное математическое открытие, например доказал гипотезу Римана, то куда ему лучше всего обращаться? В https://arxiv.org/ ?

>> No.4865681  
Файл: kuri176.jpg -(17 KB, 236x401, kuri176.jpg)
17

>>4865665
На архив постить можно только по инвайтам что-то закидывать, инвайт может дать только человек, который туда что-то закидывал, поэтому нужно найти того, кто тебя туда заинвайтит предварительно доказав ему, что ты адекватный. Можно написать электронное письмо каким-нибудь профессорам которые занимаются схожной тематикой с просьбой проверить доказательство и описать, что ты любитель и тд и тд и если он увидит что всё адекватно, то сам предложит тебе выложить на архив и опубликоваться.

Конечно есть паранойя, что доказательство украдут, в таком случае можешь начать с каких-то более мелких проблем (ведь если RH решил, то проблемы помельче - и подавно), а когда получишь инвайт на архив взять и внезапно для всех выложить туда решение RH! Но я не помню случаев, когда в современной математике любителями было доказано что-то действительно весомое, обычно любители способны придумать только какой-то пример в виде комбинаторной конфигурации, который будет лучше всех остальных придуманных примеров, но чтобы сложное рассуждение придумать - не помню такого.

>> No.4865800  
Файл: kuri177.png -(247 KB, 700x443, kuri177.png)
247

Проснулся - можно и дальше в треде сидеть.

>> No.4865814  

>>4865311

>Попробуй с Бурбаки начать.
>An extreme example is Bourbaki’s language for set theory, which requires 4,523,659,424,929 symbols plus 1,179,618,517,981 disambiguatory links (lines connecting symbol pairs,usually drawn below or above the formula) to express the number “one”

Не, спасибо

>> No.4865821  
Файл: kuri178.jpeg -(5 KB, 183x275, kuri178.jpeg)
5

>>4865814
Да к этому эпсилон-калькулусу Гильберта всё равно не прибегают нигде, кроме первых глав первой книги. Гораздо страшнее то, что они стилистически устарели, мне кажется. Хотя как справочник по диаонали сёрфить - сойдёт.

>> No.4865823  

>>4865307

>Таким же образом, как и все остальные надписи -- из контекста.

В языках программирования контекст можно однозначно понять из кода.

>Ты решил покормиться на тупости?

Нет, не собираюсь я ни на чем кормиться, и нет у меня никакого желания кому-то что-то доказывать, я просто указал на некоторые на мой взгляд проблемные места с синтаксисом этих математических записей.

>>4865298

>Что такое undefined behavior?

Это неопределенное поведение. Есть такие языки программирования, которые в отличии от языка для записи математических формул, строго формализованы и не допускают неоднозначной интерпретации того, что на них написано. Например всё тот же брейнфак, в котором UB нет и быть не может.

Мне неинтересно кому-то что-то доказывать и убеждать в чем-то. Если кто-то не видит никакой проблемы с недостаточно четким определением математического синтаксиса и семантикой и думает что я решил покормиться тупостью, ну что ж, это его право.

>> No.4865824  
Файл: kuri179.jpeg -(9 KB, 189x266, kuri179.jpeg)
9

>>4865823
Есть пруфчекеры и теории типов вроде упомянутого metamath или библиотек Lean, и есть целая каста людей, которые кодируют математические пруфы на языке этих пруфчекеров, там неоднозначности нету (правда интересных теорем тоже нету).

Неоднозначность - это некоторая сознательная цена за удобночитаемость, код на языках программирования, даже очень хорошо структурированный от условных "мастеров своего дела", читать всё же не так уж и приятно даже самим программистам.

>> No.4865826  

>>4865823

>Если кто-то не видит никакой проблемы с недостаточно четким определением математического синтаксиса и семантикой

В математике конечно есть известные проблемы, но это явно не "cos можно прочитать как c o s".

>> No.4865830  

>>4865824

>Есть пруфчекеры и теории типов вроде упомянутого metamath или библиотек Lean, и есть целая каста людей, которые кодируют математические пруфы на языке этих пруфчекеров, там неоднозначности нету (правда интересных теорем тоже нету).

А с SMT решателями ты работал? Например есть SMT-решатель Z3 от майкрософта, которым можно доказать, что x != x * x для любых x не считая 0, 1
https://rise4fun.com/Z3/iMVP
Я еще читал, что выводимые там доказательства можно оттранслировать в Coq и проверить еще там. Или просто использовать это как оралкул.
https://yurichev.com/writings/SAT_SMT_by_example.pdf

>> No.4865832  
Файл: kuri180.gif -(1661 KB, 500x281, kuri180.gif)
1661

>>4865830
Да не, времени нету всё пробовать, к тому же я бы не сказал, что я питаю какую-то особую страсть к пруфчекерам, так, трогал для общего развития когда-то.

>> No.4865884  
Файл: adm0qrsl28b21.png -(17 KB, 507x231, adm0qrsl28b21.png)
17

>>4865826

>> No.4865886  
Файл: kuri181.jpeg -(8 KB, 166x304, kuri181.jpeg)
8

>>4865884
Картинка смешная, а вот ситуация страшная?

>> No.4865938  
Файл: kuri182.jpeg -(10 KB, 252x200, kuri182.jpeg)
10

Пойду приготовлю еду, а вы пока напишите чего-нибудь, негоже когда тред мёртвый!

>> No.4865942  

>>4865620
А какие в последнее время были случаи что какой-то факт считался очевидным, но труднодоказуемым, а потом бах! и доказывали что верно как раз противоположное?

>> No.4865947  
Файл: Сірники.jpg -(126 KB, 653x490, Сірники.jpg)
126

>>4863704
С чего начать изучение математики? Люблю физику, чтобы понять многие определения из физики нужно знать математику. Я вообще не понимаю где начало у математики, откуда начинать её изучать?

>> No.4865952  
Файл: kuri183.jpg -(76 KB, 605x380, kuri183.jpg)
76

>>4865942
Неплохой вопрос! Вспомнилось несколько примеров, во-первых "комплексная гипотеза якобиана", о том, что если якобиан голоморфного отображение C^n->C^n в каждой точке не вырожден, то существует обратное голоморфное отображение, придумали контрпример, что поламало много доказательств, которые к тому моменту были (эта же гипотеза открыта, если ослабить условие голоморфности до регулярности в смысле Зарисского). Второй известный про гипотезу о групповых кольцах, что если G и H конечные, то ZG = ZH титтк G = H, доказали для многих частных случаев и сильно верили в общий, но потом (аспирант, кстати) нашёл контрпример. Ещё, мне кажется, долго верили в то, что проверка нету детерменированного полиномиального алгоритма проверки на простоту, но потом 3 индуса на какой-то мат.школе AKS-алгоритм придумали. Ещё верили, и, более того, было доказано, что на lim^1 = 0 на системах миттага-леффлера в 60х, и этим даже пользовались, а потому тоже контрпример нашли. Ну и нашли более эффективные примеры (структуры веари-фелмана) к задаче Кеплера о "наиболее эффективной мыльной пене", хотя интуитивно кажется, что конструкция самого Кеплера c биусечёнными кубами оптимальна.
>>4865947
Гельфанд Шень "Алгебра", khan academy, brilliant.org

>> No.4865954  

>>4865952
Спасибо. С меня конечно же ничего.

>> No.4865958  
Файл: kuri184.jpeg -(8 KB, 208x242, kuri184.jpeg)
8

>>4865954
От тебя ничего и не нужно!

>> No.4866010  
Файл: kuri185.jpeg -(9 KB, 300x168, kuri185.jpeg)
9

Заснул случайно! Сейчас точно начну готовить!

>> No.4866034  
Файл: topol2.png -(235 KB, 832x552, topol2.png)
235

Вопрос такой. В pdf-ке http://kpfu.ru/docs/F565128140/nasyrov_klein_but.pdf написано

> Поверхность называется неориентируемой, если она содержит в себе какой-то лист Мебиуса. В противном случае поверхность

называется ориентируемой.

> У ориентируемой поверхности есть две стороны, у неориентируемой - одна.

Так вот, если взять ленту из бумаги (это НЕ лист Мёбиуса, у него две поверхности), сделать из этой штуки фигуру вращения то на выходе получится уже поверхность (ну это как тор, в котором при разрезании на две части вместо "oo" будет "∞" т.е. бублик без дырки с самопересечением по центру, как в закрытом торе). Как эту поверхность описать, есть ли у нее название? Это известная штука вообще? Точка, в которой внешняя поверхность этого какбы тора переходит во внутреннюю там будет только одна - это центр закрытого тора. Гомеоморфно ли это бутылке Клейна?

>> No.4866036  
Файл: 03588982ec17.jpg -(17 KB, 640x480, 03588982ec17.jpg)
17

Еще картинка для пояснения

>> No.4866047  

>>4865681

>Но я не помню случаев, когда в современной математике любителями было доказано что-то действительно весомое, обычно любители способны придумать только какой-то пример в виде комбинаторной конфигурации, который будет лучше всех остальных придуманных примеров, но чтобы сложное рассуждение придумать - не помню такого.

https://www.theverge.com/2018/10/24/18019464/4chan-anon-anime-haruhi-math-mystery
A 4chan poster may have solved part of a very tricky math problem that mathematicians have been working on for at least 25 years. The user was just trying to figure out the most efficient way to watch episodes of a nonlinear anime series, but the result has generated considerable interest from mathematicians around the world who have no way to identify the anonymous user.

>> No.4866055  
Файл: kuri186.jpg -(22 KB, 500x274, kuri186.jpg)
22

>>4866034
Это horn torus, это не гладкая поверхность (есть особенность в центре) для таких понятие (не)ориентируемости не рассматривают, ну или рассматривают но я не в курсе. Негомеоморфна бутылке клейна.
>>4866047
Во-первых не факт, что это был любитель, а во-вторых "найти более оптимальную перестановку которая удовлетворяет каким-то там свойствам" подходит под моё видение "примера в виде комбинаторной конфигурации". Неужто ты думаешь, что я не вспомнил эту историю когда писал тот свой пост?

>> No.4866079  

>>4865952
Интересные примеры, спасибо! Некоторые весьма красивые и наглядные даже если в математике не очень разбираться.

>> No.4866082  

>>4866055
Вот этот пример наверное хорошо подходит под "найти более оптимальную перестановку", но всё же: он насколько замечателен? Для непрофессионального математика круто?
https://www.quantamagazine.org/decades-old-graph-problem-yields-to-amateur-mathematician-20180417/

>> No.4866094  
Файл: kuri187.jpg -(56 KB, 596x380, kuri187.jpg)
56

>>4866082
Для непрофессионального математика любое продвижение в любой открытой проблеме - это круто, но всё же мне кажется что конкретные комбинаторные конфигурации и примеры менее ценны, чем какие-то универсальные рассуждения.
>>4866079
Без проблем, если что ещё вспомню - скажу!

>> No.4866105  
Файл: StandardTori_701.png -(37 KB, 508x338, StandardTori_701.png)
37

>>4866055

>Это horn torus, это не гладкая поверхность (есть особенность в центре) для таких понятие (не)ориентируемости не рассматривают

Я почитал про этот horn torus и не очень понял, что они там имеют ввиду. Если мы идем от наружной поверности такого тора к его центру, то мы так можем попасть наизнанку (на внутреннюю поверхность) такого тора? Может под этим horn torus понимают немного не то, что я имею ввиду?

>> No.4866108  
Файл: tori_question.png -(35 KB, 508x338, tori_question.png)
35

>>4866105
Вот попробовал проиллюстрировать свой вопрос

>> No.4866171  
Файл: kuri188.jpg -(35 KB, 300x188, kuri188.jpg)
35

>>4866105
Возможно я неправильно тебя понял, посмотри ещё klein eight figure возможно то, что ты ищешь.

Как раз в том-то и дело, что так как нельзя сказать особая точка "внутри" или "снаружи" то нельзя и как-то естественно "продолжить" кривую, которая через эту особую точку проходит. Это я и имел в виду под словами, что для таких поверхностей вопросы ориентируемости не рассматривают.

>> No.4866186  

>>4866171

>Возможно я неправильно тебя понял, посмотри ещё klein eight figure возможно то, что ты ищешь.

Нет, klein eight figure это не то. Я думаю что ты меня понял правильно, просто я не понимаю, почему нельзя четко определить то, куда мы попадем, если пройдем через эту особую точку (выйдем ли опять наружу этого "бублика с нулевой дыркой" или войдем внутрь)? Я не думаю что это прямо так сложно, указать свойства этой точки при прохождении через нее.

>> No.4866198  
Файл: kuri189.jpeg -(6 KB, 189x267, kuri189.jpeg)
6

>>4866186
Можно реализовать оба сеттинга в зависимости от того, какую историю ты про этот бублик хочешь рассказывать, никакого выделенного среди них нету, тут дело не в технической сложности, а в том, что обе тобою описаные ситуации на картинке равноправны и по желанию для своих целей использовать можно как первую, так и вторую, предварительно это оговаривая, а ты как-будто хочешь, чтобы какой-то математический бог выделил среди них одну.

>> No.4866210  
Файл: kuri190.jpeg -(6 KB, 225x225, kuri190.jpeg)
6

>>4866186
Кстати говоря, твоя конструкция с тем, чтобы рассмотреть такой путь на хорн торе, тем не менее, довольно остроумная, не могу не похвалить!

>> No.4866226  
Файл: Nh1c.jpg -(26 KB, 383x424, Nh1c.jpg)
26

>>4866198
Вот тут такую картинку нашел
http://www.meru.org/Advisors/klein.html

>Note the double-cone at the Klein bottle's center:
>Any Klein bottle in 3-dimensional space must pass through itself somewhere. The Klein bottle comes in many topologically equivalent forms.

Вот эта вот прокрученная вокруг центра ∞ которую назвали "frisbee" - это и есть то, что я имел ввиду. И походу это просто еще один способ нарисовать бутылку Клейна.

>> No.4866295  

>>4866226
Хорошо тогда

>> No.4866379  
Файл: kuri192.jpeg -(9 KB, 269x188, kuri192.jpeg)
9

Бездарный день.

>> No.4866381  

>>4866379
Это посольство Курисача? Что-то слишком прилипла ассоциация персонажа и борды, даже сильнее, чем Хоро<->Доброчан.

>> No.4866383  
Файл: kuri193.jpeg -(9 KB, 189x267, kuri193.jpeg)
9

>>4866381
Нет, нужна была аниме-девочка связанная с математикой и я ничего лучше, чем Курису, не придумал.

>> No.4866461  

>>4866383
А в каком вообще аниме есть математика?

>> No.4866464  
Файл: kuri194.jpeg -(5 KB, 195x259, kuri194.jpeg)
5

>>4866461
Вроде бы ни в каком нету, ну или я не в курсе.

>> No.4866488  
Файл: Pixiv 4303033 68883476 01.png -(398 KB, 800x1012, Pixiv 4303033 68883476 01.png)
398

>>4866464
Надо было бы Сырну брать.
https://www.youtube.com/watch?v=5wFDWP5JwSM

>> No.4866490  
Файл: kur197.jpeg -(4 KB, 299x168, kur197.jpeg)
4

Надо было, но уже взял кого взял.

>> No.4866504  

>>4866490
Связывать девочек никогда не поздно.

>> No.4866507  

>>4866488
Сырна на ычане это слишком банально.

>> No.4866508  
Файл: kuri196.jpeg -(10 KB, 188x268, kuri196.jpeg)
10

>>4866504
Быть может в следующем треде, если мне это всё не надоест, тем более уникальные картинки с Курису искать всё сложнее, а писать какие-то скрипты которые бы меняли картинки старые или вручную ставить точку в пэинте - лень.

>> No.4866511  
Файл: 500px-Knot_table.svg.png -(38 KB, 500x372, 500px-Knot_table.svg.png)
38

ОП, а ты знаешь что-нибудь про теорию узлов? Она где-нибудь применяется(например чтоб через нее придумать узел, который трудно развязать), или просто математикам хочется побольше всяких вещей формализовать своими теориями?

>> No.4866514  
Файл: kuri197.jpeg -(10 KB, 195x258, kuri197.jpeg)
10

>>4866511
Кое-что знаю. Не, большая часть науки про узлы касается свойств всяких алгебраических и топологических инвариантов узлов (типа гомологий флоера, хованова и фундаментальной группы дополнения), а также гиперболических структур на дополнении к узлу. Вообще узлы - это такая игрушечная моделька в алг. топологии, на которой большая часть алг.топологической машинерии может быть протестирована, прежде чем переходить к каким-то более сложным случаям. Помнится даже кто-то из серьезных людей хотел написать книжку в духе "knots as toy model in alg.topology", но так и не написал. Вычислительная теория узлов связанная со всякими алгоритмами развязывания и завязывания есть, но там, вроде, пока что без сногсшибающих результатов, вроде ничего лучше чем алгоритма из класса co-NP нету. А, ещё Концевич свой интеграл придумал интегрируя какие-то бандлы по каким-то узлам, но про это я точно не в курсе деталей.

>> No.4866516  
Файл: kuri199.jpg -(81 KB, 811x951, kuri199.jpg)
81

А, вот что ещё вспомнил, есть арифметическая топология и cловарь Мазура-Моришиты-Капранова-Резникова, про то что S^3 соответствует Spec(Z), простые числа - узлам, индекс зацепления - символу Лежандра, факт неимения нетривиальный неразветвлённых накрытий у S^3 транслируется в факт неимения нетривиальных алг.расширений у Z, квадраатичный закон взаимности Гаусса транслируется в аналогичный (но тривиальный) закон для зацеплений и тд и тд. Насколько я понимаю пока что какой-то серьезной причины такой сильной аналогии не найдено, но вот это действительно очень неожиданная и будоражащая фантазию штука, за которые я математику и люблю.

>> No.4866554  
Файл: kuri200.jpg -(45 KB, 200x300, kuri200.jpg)
45

>>4866516
Неразветвлённых расширений нету, конечно, разветвлённых сколько угодно. как-будто кто-то проверять будет

>> No.4866686  
Файл: pomme.jpg -(271 KB, 2518x3200, pomme.jpg)
271

Начал читать тред, а там "семантики", "смыслы", "проверяемость"... Философия больше чем математика. Это не яблоко.

>> No.4866696  
Файл: makise_kurisu_by_theboredgnome-d6c1n10.p(...).png -(61 KB, 233x300, makise_kurisu_by_theboredgnome-d6c1n10.p(...).png)
61

>>4866686
Семантика действительно слегка метаматематический концепт, хоть с ним и можно в рамках математики работать, если закрывать глаза на кое-какие неудобства. А что философия больше - так и пускай, математике не нужно быть больше кого-то, чтобы я её любил!

>> No.4866709  
Файл: LvpSxy3iPA8.jpg -(195 KB, 1201x1500, LvpSxy3iPA8.jpg)
195

>>4866696
Просто с философской точки зрения, ваш спор о доверии к компиляции очень просто разрешить.

1) Истины не существует, есть только "достаточно точные в контексте задачи" приближения. Тогда исчезает требование к этой самой стопоцентной доказуемости. И ничто тогда не запрещает мне какой-нибудь заковыристый интеграл взять в том же вольфраме, даже зная, что он неправильно считает интегралы.

2) Смысл связан с семантикой, а значит переход от одного языка к другому в корне меняет саму суть поставленной задачи. И когда мы доказываем что "гипотеза Римана верна", то мы можем сделать это только на русском языке. А "Riemann's hypothesis is correct" - это уже другая задача, решающаяся в других категориях. Пример не совсем контрастный, потому что человеческие языки (почти все) обладают эквивалентными категориальными базами, но суть, надеюсь, понятна. А все сопутствующие вычисления на других языках возможны тогда и только тогда, когда мы имеем четкий словарь: "это значит это, а то значит то". Если же мы не понимаем кусок кода, то ничего доказать с помощью него мы не можем.

3) Логическое соответствие в разных логиках разное, невозможно установить логическое соответствие в суждениях машины и человека, обезьяны и человека, а зачастую и человека и человека.

Итого: При переходе из семантики в семантику, мы меняем условия задачи, и от этого никуда не денешься. Но так как условия задачи меняем мы , то мы осознаем с какими приближениями и допущениями наша задача решается в другой логике и семантике.

>> No.4866714  

>>4866709

>Истины не существует, есть только "достаточно точные в контексте задачи" приближения.

Не согласен с этим утверждением. Типа кто-то скажет, что 1+1=2 а кто-то такой "ну неее, истины не существует, 1+1 это может не точно 2, а допустим 2.00001 или там 1.9999" - но это же бред, 1+1 точно равно 2 если мы таким-то конкретным образом понимаем выражение "1+1", истина у нас будет одна и только одна, без вариантов.

>Смысл связан с семантикой, а значит переход от одного языка к другому в корне меняет саму суть поставленной задачи.

С этим тоже не согласен. Если я напишу "1+1=2" римскими цифрами "I+I=II" то никакой сути поставленной задачи не меняется.

>Логическое соответствие в разных логиках разное

Тут я вообще не понял. Что ты имеешь в виду?

>Итого: При переходе из семантики в семантику, мы меняем условия задачи, и от этого никуда не денешься.

Не согласен

>> No.4866721  
Файл: 625347.jpg -(225 KB, 1165x999, 625347.jpg)
225

>>4866714

>1+1=2

В каком контексте? Один математик плюс один студент равно полтора математика.

>I+I=II

Это не так считается. Возьмем менее тривиальный пример: 81*19 и LXXXI помножить на XIX. Все еще разницы не чувствуешь? Это почти все равно что написать "1" и "0,(9)". С точки зрения математики - одно и то же, а вот с точки зрения гносеологии совершенно разное. Хотя бы потому что получается из разных процессов. А машина вообще писать не умеет, только током биться.

>Логическое соответствие

Галилеева и Лагранджева механики, например. Минимальное отличие в логике приводит к максимальным отличиям в результатах. Причем нарушений логики нет ни там ни там. А на уровне, когда отличаются базовые концепции, все еще интереснее. Объясни от рождения слепому что "У Кафки рыжжие волосы - ложь", а "У Кафки черные волосы - истина". Потом то же самое объясни обезьяне.

>Не согласен

Посчитай сначала 81*19, а потом LXXXI помножить на XIX. Второе я не уверен, что сможешь, но все-таки.

>> No.4866733  

>>4866721
Опять же, более красиво: 1+1=2+dx, где dx - бесконечно малая величина. Интегрируем по x, и внезапно не x+x=2x, а x+x=2x+x^2. И в случае комплексных чисел, совсем не обязательно x=0

>> No.4866773  

>>4866733
где мои C, aaа?!

>> No.4866905  
Файл: kuri201.png -(571 KB, 861x929, kuri201.png)
571

Ужасный день, но активности в треде рад!

>>4866709
Я прокомментирую, но если что я ни с кем не спорил и ни в чём никого убедить не пытался! Кому нравятся пруверы и у кого они повышают уверенность в чём-то - пусть пишут свои термы дальше, просто мне они не нравятся!

1) Сильное заявление, но его можно придерживаться если разогнать фантазию в достаточной степени (и так всякие \Pi_0^1-формалисты и делают), для меня вот арифметическая истинность уж точно существует, да и в теоретико-множественную истинность, вообще говоря, верю, для меня унивёрсум-фон-неймана объект той же сложности, что и стандартная модель натуральных чисел и оба объекта мне кажутся предельно ясными.
2) Ну примерно о том я и говорил, совсем не обязательно всё редуцировать до какого-то одного метаязыка, так как математики рассказывают историю в пределах одной статьи, значит, грубо, можно считать что в пределах каждой статьи свой маленький синтаксис и своя маленькая семантика. Иногда можно построить интерпретацию результатов из одной статьи в другую - иногда нет.
3) Да, конечно, примерно об этом я и говорил когда говорил, что пруверы не уменьшат количество ошибок, а просто перенесут ошибки на уровень "закодировано было не то, что заявлялось".
>>4866721
Uuggh, ну ты прямо \Delta_0-формалист, как-то недоверие к истине у тебя не из того источника идёт, из какого мне хотелось бы чтобы она шла. Хотелось бы, чтобы дело у тебя не сводилось к всяким штукам вроде приколов с квалиа, когда непонятно то ли человек имеет в виду под "1+1=2", что и ты, сколько бы вы детали контекста между собой не обсуждали (можно сказать, что это семантичесская формулировка теоремы гёделя). И уж точно не хотелось бы чтобы дело сводилось к "природу можно формализовать по разному", всё-таки следует как-то отличать арифметическую и теоретико-множественную истинность от хорошей мат.модели какого-то природного явления.
>>4866733
Идея понятно, но, FGJ, в комплексных верно x^2 = 0 -> x = 0, говори тогда уже "дуальные числа" или, ещё понтовее "неверно в любом кольце с нетривиальными нильпотентами".

>> No.4866928  

Как вообще может такое быть, что математика абсолютна, что в любой вселенной с любыми физическими законами мы можем посчитать, к пример, число Пи с помощью простейшей бинарной логики и оно будет равно всегда одному и тому же? Почему невозможно даже вообразить себе такое, что математика будет вести себя совсем по-другому? Но значит ли то, что мы не можем это вообразить, то, что это не может существовать?

Помните "Контакт" Карла Сагана? В котором в числе Пи в определённой системе счисления нашли подпись бога? Но ведь даже бог не может изменить число Пи. Если бог может думать, то он уже может вычислить число Пи, значит он не может создать его и оставить там что-то, так как оно уже есть.

Как так-то?

>> No.4866932  

>>4866928

>и оно будет равно всегда одному и тому же

Искривления пространства при околосветовых скоростях и зависимость вообще почти всего от максимальной скорости света вроде говорит об обратном.

>> No.4866944  
Файл: kuri202.jpg -(76 KB, 1200x628, kuri202.jpg)
76

>>4866928
Сам диву даюсь что как-то так и выходит! Надеюсь когда-нибудь арифметика расщепится на \Sigma_0^1 уровне, чтобы этот мой невроз унять!
>>4866932
Думаю что он имел в виду то pi, которое через формулы Валли вычисляется, то есть имел в виду некоторую последовательность цифр которую выдаёт вполне конкретная программа, а не про измерения на околосветовых скоростях, в экзотических областях пространства-времени на первых наносекундах после большого взрыва и тд и тд!

>> No.4867095  
Файл: i_043.jpg -(57 KB, 485x528, i_043.jpg)
57

>>4866928

>Но ведь даже бог не может изменить число Пи.

Всему-то вас, молодежь, учить надо.
Значит так. Начинаешь с определения круга. Кругом называешь хреновину, у которой отношение периметра к диаметру равна Пи. Пи выбираешь на свой вкус.
И от этого начинаешь плясать. Типа, аксиоматика. И, соответственно, сотворяешь весь остальной мир.

>> No.4867097  

>>4866932
Кривизна кривизной, но Пи всё равно участвует во всём этом безобразии. И мы можем измерить кривизну с помощью, например, суммы углов треугольника, и потом из кривизны и нарисованной окружности посчитать Пи (если кривизна непостоянная, то будет сложнее, но это не важно). А затем сравнить с тем Пи, что посчитали на компьютере. И они внезапно будут равны. И мы не можем себе представить так, что не равны. Потому сама по себе геометрия абстрактна и применима к любому пространству, которое мы только можем придумать. И внезапно при этом имеет отношения с числами, которые можно взять и вычислить негеометрическим способом.
>>4866944

> \Sigma_0^1

Это вот это вот https://en.wikipedia.org/wiki/Arithmetical_hierarchy ? Можешь пояснить простыми словами? В частности, что именно ты имел в виду.

>> No.4867099  

>>4867095
Круг — это в первую очередь хреновина, у которой точки равноудалены от другой точки. И ты можешь, конечно, манипулировать метрикой сколько угодно, но ты не можешь сделать так, чтобы при эвклидовой метрике (корень из x²+y²) соотношение длины окружности к диаметру не было равно Пи. В определении эвклидовой метрики никакого Пи при этом не было.
То есть даже если ты живёшь в пространстве с абсолютно упоротой метрикой и топологией ты всегда сможешь на компьютере запрограммировать эвклидову плоскость. И там будет Пи. У тебя никак не получится сделать так, чтобы не было.

>> No.4867105  

>>4867097
Ну, да, но отношение длины окружности к радиусу в разных геометриях разное и может быть непостоянным. И в реальной жизни у нас, на секундочку, неЕвклидова геометрия.

>> No.4867108  

>>4866928
Может тебя вообще глючит, а на самом деле дважды два - пять, и противоположное можно только в бреду представить?

>> No.4867111  

>>4867105
Но только одно из них равно Пи. И у нас есть способ определить, какое.
>>4867108
Если глючит одного только меня, то я довольно быстро столкнусь с проблемами.

>> No.4867113  

>>4867108
И даже если всех. То если на самом деле равно 5, то мы сможем понять это.

А вот придумать логически непротиворечивый мир, где и правда 5 (понятное дело, что мы не об именах цифр и не о системах счисления говорим, а об абстрактных понятиях), мы не можем.

>> No.4867115  

>>4867113

>А вот придумать логически непротиворечивый мир, где и правда 5 (понятное дело, что мы не об именах цифр и не о системах счисления говорим, а об абстрактных понятиях), мы не можем.

Это, опять же, исходит из преположения что логика как и арифметика важнее и независимее ощущений. Однако вполне можно представить бред, в котором тебе будет казаться что 2х2=17 и обратное противоречит логике. Значит, может случиться и обратное - в бреду дважды два - четыре, а на самом деле - нечто другое.

>> No.4867116  

>>4867099
Окей, раз для тебя ключевыми являются именно эти свойства, то определяем понятия равноудаленности, метрики, сложения и возведения в степень таким образом, что это всё по опредлению выполнялось для нашего значения Пи.

А всё остальное уже будем из этого набора аксиом выводить. Разницы нет. Просто ты привык к тем базовым понятиям, которые справедливы для нашего мира. И других представить не можешь.

>> No.4867122  
Файл: kuri204.jpg -(151 KB, 1600x1090, kuri204.jpg)
151

>>4867097
Ну про \Sigma_1 уровень я загнул - низковато всё же. Но чтобы было некоторое недоказуемое и неопровержимое арифметическое утверждение первого порядка (на языке L^1) такого сорта, в которое были бы резонные причины как верить, так и не верить, это бы действительно пошатнуло картинку того, что мы понимаем, что такое "стандартная модель натуральных чисел" (то есть натуральные числа as it is) На языке L^3 такое утверждение есть - континуум-гипотеза называется, но L^3 это слишком абстрактно, а вот если бы было расщепление на L^1 - то это действительно была бы ситуация, которую не стыдно было бы назвать постмодерном (sic!) и посягательством на арифметическую истинность. А пока не вижу причин не пользоваться "арифметической истинностью" как вменяемым концептом.

Под "пространством", кстати, в математике зачастую понимают нечто намного более абстрактное, чем то "физическое пространство", которое трёхмерное, плоское, однородное, изотропное и с трансляционно-поворотными симметриями и которое обычно люди под этим словом представляют. Ничего трансгрессивного в этом нет, конечно, просто так мат. терминология устоялась, щито поделать.

>> No.4867166  

>>4867122
Ты не мог бы понятным для простого обывателя языком объяснить, что означают все эти сигма-уровни и языки L^1 L^2 и так далее?

>> No.4867185  
Файл: kuri204.jpeg -(5 KB, 273x185, kuri204.jpeg)
5

>>4867166
L1,L2,L3 языки логик первого, второго и третьего порядка, в контексте арифметики позволяют делать утверждения о натуральных числах, множествах натуральных чисел и множествах множеств натуральных чисел соответственно. Завтра напишу про арифметическую иерархию, устал очень сейчас. Если коротко: это некоторая классификация утверждений о натуральных числах по уровню абстрактности, на самом нижнем \Delta_0-уровне находятся утверждения которые требуют конечного (хоть иногда и очень большого) числа синтаксических преобразований для проверки их истинности (вроде 2+2=4, в шахматах всегда будет ничья, и тд итд)

>> No.4867189  
Файл: images (6).jpeg -(7 KB, 284x177, images (6).jpeg)
7

>>4867185

> в шахматах всегда будет ничья

Кажется очевидным, что в подобного рода играх должна обязательно существовать либо выигрышная стратегия для одной стороны, либо ничейная для обеих. Но как это доказать?

>> No.4867190  
Файл: 740a6ff3360c525881623d04b47328e6-700.jpg -(20 KB, 1120x700, 740a6ff3360c525881623d04b47328e6-700.jpg)
20

>>4867189
Ого, я только что взял и доказал. Видимо, в постели под одеялком думается намного лучше.

>> No.4867293  
Файл: kuri205.jpg -(24 KB, 236x419, kuri205.jpg)
24

>>4867189
>>4867190
Это открытая проблема, но тут это не важно, а важно то, что она сводима к конечному перебору.

>> No.4867302  
Файл: beavis_and_butt_head_by_frow7-d4h6s7s.png -(278 KB, 1024x774, beavis_and_butt_head_by_frow7-d4h6s7s.png)
278

Косинус
Синус
Глобус
Анус

Школа - отстой

>> No.4867654  

>>4867185

>L1,L2,L3 языки логик первого, второго и третьего порядка, в контексте арифметики позволяют делать утверждения о натуральных числах, множествах натуральных чисел и множествах множеств натуральных чисел соответственно.

Я кажется понял что тут имеется ввиду, правда это было сделано нестандартным способом (я вообще программист). Есть CS такая доказано нерешаемая проблема - проблема останова. Доказательство можно найти много где, можно почитать про доказательство через Канторов диагональный процесс тут https://web.archive.org/web/20170215130846/https://inf.1september.ru/2000/5/art/turing.htm (мне оно сильнее нравится, чем через рекурсию, но не суть). Взять например Великую Теорему Ферма (которую уже доказали). Если б существовал некий физический принцип, на основе которого можно было б дать заключение, завершается ли некая компьютерная программа на брейнфаке, то прибор, работающий на этом физическом принципе, мог бы легко доказать для теоремы Ферма, что она верна. Т.е. мы б просто закодировали на брейнфаке компьютерную программу, которая просто перебирает все счетное бесконечное множество чисел, удовлетворяющих условию, пытаясь найти решение, и спросили бы у прибора "зависает ли наш перебор теоремы Ферма?" - и на основе ответа мы б сделали соответствующее заключение. Проблему останова для машины Тьюринга можно решать через https://ru.wikipedia.org/wiki/Машина_Зенона если б ее можно было сконструировать. Т.е. будь у нас эта машина Зенона, то она за конечное время перебрала б все счетно-бесконечные варианты теоремы Ферма и загорелась бы лампочка "ЗАВИСАНИЕ".
Теперь давайте рассмотрим https://ru.wikipedia.org/wiki/Теорема_Гудстейна
Тут уже так не выйдет! У нас есть счетное бесконечное множество целых положительных чисел, и нам надо для него доказать, что некоторое преобразование для каждого такого числа всегда завершается чем-то там за конечное число шагов. Это тут просто неформулируемо, я не нашел способа это записать в терминах зависаемости одной конкретной МТ, тут нужен иной подход, тут нужно запустить счетно-бесконечное множество компьютерных программ на брейнфаке и проверить, что все из них всегда завершимы за конечное число шагов. Т.е. если у нас есть волшебный процессор с бесконечным числом волшебных процессорных ядер, каждое из которых за конечное время может решить проблему останова для МТ (как машина Зенона), то вот на такой штуковине эта проблема может быть решена - мы одновременно для каджого из ядер даем по одному из чисел, запускаем там эту проверку теоремы Гудстейна, и потом смотрим на бесконечный ответ от всех ядер - если хоть один из ответов будет "ЗАВИСАЕТ!" то теорема Гудстейна будет опровергнута. Теорема Гудстейна доказывается в арифметике второго порядка - это видимо и есть то, что имеется ввиду под L2. В общем между этими языками логик и способом перебора есть прямое соответствие

>> No.4867706  

А чем арифметика Пеано лучше или хуже арифметики Пресбургера?

>> No.4867786  
Файл: kuri207.jpg -(9 KB, 184x184, kuri207.jpg)
9

>>4867706
Это разрешимая теория, а значит - слабая, а значит почти ничего на ней сформулировать нельзя (например, нельзя выразить предикат "х - простое число").
>>4867654
Понял почти правильно, теорема гудстейна это Pi_2^0 утверждение на L1 языке а именно утверждение уровня "для любого n существует такое k т.ч. "k-ый член последовательности гудстейна начатой с n будет равен 0"

>> No.4867875  
Файл: 3170.jpg -(63 KB, 225x350, 3170.jpg)
63

>>4867293
Бака, я не про конкретно шахматы, а про любую игру, где два игрока могут менять текущую позицию по очереди по определенным правилам и рано или поздно можно придти к позиции, считающейся выигрышем (или проигрышем) (а ещё ничьей!) для того, кто ее получил!

И вот то, что любая такая игра всегда имеет выигрышную стратегию для одного из игроков, легко доказывается при помощи индукции.

>> No.4867880  
Файл: kuri208.jpeg -(10 KB, 188x267, kuri208.jpeg)
10

>>4867875
Да, конечно, неправильно прочитал твоё сообщение, думал ты спрашиваешь о ничейности шахмат.

>> No.4867882  
Файл: thumb-91934.jpg -(14 KB, 350x219, thumb-91934.jpg)
14

>>4867875
по определенным правилам, зависящим только от текущей позиции и, быть может, от предыдущих сделанных ходов

Так, наверное, будет точнее.

Хотя, в таком случае, нам вообще не нужно придумывать такую штуку как позиция, а просто считать, что ходы, доступные первому или второму игроку зависят только от предыдущих сделанных ходов, и что исход игры определяется при получении одной из заранее известных последовательностей ходов. Ха!

>> No.4867888  
Файл: kuri210.jpg -(116 KB, 706x1001, kuri210.jpg)
116

>>4867882
Да лучше сразу определять "комбинаторную игру для двух игроков с полной информацией" как следующую: есть орграф G со стартовой (отмеченной вершиной) s на которую ставится фишка, игроки ходят по очереди и за ход должны двинуть это фишку вдоль ребра, кто не может сделать ход - проиграл, а всякие шахматы-шашки-хексы-нимы-крестикинолики уже сводить к игре такого рода, как это и стандартно делается во всяких штуках типа теории Шпрага-Гранди.

>> No.4867895  
Файл: диван.png -(56 KB, 2000x753, диван.png)
56

Как продвигается решение задачи о передвижении дивана? Вообще она сложная потому что ей не занимаются или потому что она сводится к каким-то плохо поддающимся исследованию областям математики? Ну и ещё, для каких-нибудь похожих задач нашлись 100% доказанные оптимумы?

>> No.4867908  
Файл: kuri211.jpeg -(10 KB, 179x281, kuri211.jpeg)
10

>>4867895
Вроде никак, никаких громких заявлений о верхних оценках не слышал, а нижние, мне кажется, не очень интересны, не занимаются потому что никто не придумал какого-то продуктивного подхода, мне кажется. Да куча таких есть, проблема Ньютона о минимальном сопротивлении, проблема кривой кратчайшего спуска, да и вообще эволюция любой механической системы - это поиск решения которое минимизирует некоторыый функционал энергии, т.к. что вычисление траектории падения камня, орбиты планет, волновой функции атома гидрогена - это всё примеры такого рода, думаю если бы я в этом больше варился, то ещё бы какие-то аналитически вычеслинные оптимальные управления тебе бы в пример привёл, но мне ничего на ум не приходит.

>> No.4867917  
Файл: Fuura.Kafuka.600.437756.jpg -(47 KB, 600x375, Fuura.Kafuka.600.437756.jpg)
47

>>4867888
Не совсем. При таком определении оба игрока в случае попадания в одну и туже вершину имеют доступ к одним и тем же ходам, в то время как в тех же шахматах доступные ходы у каждого игрока -- свои.

>> No.4867924  
Файл: Fuura.Kafuka.600.437712.jpg -(21 KB, 600x375, Fuura.Kafuka.600.437712.jpg)
21

>>4867917
Впрочем, нет, всё в порядке. Мы же можем создать такой граф, в котором каждая вершина обозначает не просто позицию на доске, но позицию в сочетании с очередностью хода. Блин, как это интересно.

>> No.4867925  
Файл: kuri212.jpeg -(2 KB, 300x168, kuri212.jpeg)
2

>>4867917
Значит, что граф шахматных состояний устроен таким образом, что второй игрок никогда не попадёт фишкой в вершины, которые соответствуют ходам белых фигур, а первых не попадёт в вершины которым соответствуют ходы черных фигур. В случае шахмат граф понятно как устроен: вершинам соответствуют состояния доски + очерёдность хода (+ можно пару битов выделить на то, была ли рокировка и какие-то такие тонкости), а стрелкам, соответственно, допустимые ходы.

>> No.4868165  
Файл: kuri214.jpeg -(8 KB, 160x316, kuri214.jpeg)
8

Есть настроение посидеть ночью, потому - пишите.

>> No.4868171  

>>4867185

>Завтра напишу про арифметическую иерархию, устал очень сейчас.

Напиши. Или подскажи, где это можно почитать?

>> No.4868172  

>>4867908
Ясно, спасибо. Вообще может быть если хитро подобрать форму коридора, то можно будет свести эту задачу к каким-нибудь совсем непохожим.

>> No.4868180  
Файл: kuri215.jpg -(31 KB, 500x281, kuri215.jpg)
31

>>4868171
А, забыл совсем, ну как я и говорил это классификация утверждений по уровню абстрактности. На Delta_0 уровне утверждения в духе "2+2=4", "2^2^37+1 - простое число" и прочие в некотором смысле конечные утверждения, если проводить аналогию с естествознанием, то в соответствие Pi_0 утверждениям можно поставить конкретные события типа "7 марта в 13:00 солнце находилось в точке (13,27) небесной сферы", если с программированием, то Pi_0 уровню соответствуют программы без циклов (или с ограниченными цыклами, но концептуально это не суть).

Pi_1 утверждения - это уже утверждения вида "Для любого x.P(X)" где P(X) - это Pi_0 утверждение, например что не существует нечётных совершенных чисел (для любого x. x нечётное -> x не совершенное) более того RH тоже Pi_1 хотя доказать это и нетривиально, если проводить аналогию с естествознанием, то этому будут соответствовать "однопараметрические универсальные утверждения" в духе "солнце в момент времени t находится в месте f(t)".

Pi_2 уровень это уже "для любого х существует у . P(x,y)" упомянутая тобою теорема гудстейна - хороший пример, ещё один пример - гипотеза коллатца, используя аналогию с естествознанием дальше можно наверное сказать что этому соответствует что-то в духе "у любого тела малой массы в солнечной системе существует эллиптическая орбита".

Ну и тд, Pi_3 это "для любого х существует у для любого z" Pi_n это n раз альтерировать кванторы и начать с "для любого" Sigma_n тоже самое но начать с "существует".

Там есть ещё аналогии с вычислимостью, борелевской/проективной иерархией, множествами натуральных чисел и множествами точек в пространствах Бэра или Кантора, но всё расписывать - это умереть можно. Где об это прочитать (кроме статей на вики) - даже не знаю, это всё используют обычно чтобы называть какие-то вещи и рассказывать какие-то истории, а не как предмет исследования сам по себе. Лично я об этом отдельно никогда не узнавал, узнавал из вступлений ко всяким докладам, но если вспомню какой-то хороший источник для начинающих - то скажу.

>> No.4868185  
Файл: kuri216.jpg -(29 KB, 500x283, kuri216.jpg)
29

>>4868172
Да может, кто его знает. Насколько я понимаю там проблема в том, что вот это ограничение на то, чтобы диван куда-то там прошёл никак явно особо не запишешь по-хорошему, поэтому не понятно вообще как может выглядеть доказательство верхней оценки, ну тупо какие там формулы должны быть, если изначально в формулировке никаких формул нету.

>> No.4868214  
Файл: sin.png -(154 KB, 1549x795, sin.png)
154

ОП, а ты умеешь исследовать рекуррентные формулы?
вот есть такая рекуррентная формула
f1(0) = 1
f1(1) = 1
f1(n) = abs(f(n-1)-n)/f(n-2)

Если построить график, получается четыре каких-то синуса, которые вертятся вокруг почти что sqrt(x)
А если определить такую вот рекуррентную формулу
f2(0) = 1
f2(1) = 1
f2(n) = abs(f(n-2)-n)/f(n-1)
То получается как раз этот "почти что sqrt(x)" вокруг которого крутятся эти четыре синуса.

Как можно понять, почему вторая рекуррентная формула f2 так близка к квадратному корню? И почему там именно 4 синуса в первой формуле, а не 8 например? Как быстро меняется период и амплитуда этих синусов? Где прочитать про математический аппарат, который это позволяет делать?

>> No.4868215  

>>4868214

>Где прочитать про математический аппарат, который это позволяет делать?

Про производную почитай.

>> No.4868216  

>>4868215
Я знаю про производные, но что мне эти производные дадут? Я через производную смогу понять, почему там синусов ровно 4 на графике?

>> No.4868378  
Файл: kuri217.jpeg -(11 KB, 183x275, kuri217.jpeg)
11

>>4868216
Не умею но попробую!

>> No.4868689  
Файл: kuri218.jpeg -(8 KB, 284x177, kuri218.jpeg)
8

Бампну.

>> No.4868748  
Файл: f2de080e480ec3c5ed696c3a9db2a107.png -(19 KB, 611x612, f2de080e480ec3c5ed696c3a9db2a107.png)
19

>>4868378
Ну я посмотрел спектрограмму для отдельного "синуса" (можно каждую четвертую точку брать из последовательности) и получается что этот "синус" состоит из трех синусов... В общем сложная какая-то штука

>> No.4868955  
Файл: kuri219.jpeg -(8 KB, 190x265, kuri219.jpeg)
8

>>4868748
Да рекуррентности с модулем и прочими if/else очень неприятно анализировать, может на досуге погуглю есть ли нечто похожее, но что заметил такую закономерность - молодец конечно!

>> No.4869234  

>>4868955
А можешь привести примеры каких-нибудь решаемых тобою по работе задач, понятных для нематематиков? Ну чтоб сама задача имела относительно простую формулировку, доступную для понимания неспециалистам

>> No.4869235  

>>4869234

>А можешь привести примеры каких-нибудь решаемых тобою по работе задач

Ну точнее необязательно чтоб это была какая-то конкретная задача из твоей текущей работы, которую ты сейчас решаешь или решал когда-то, а просто пример какой-нибудь задачи, которая как раз по твоей части, и чтоб саму задачу можно было легко понять неспециалисту. Типа Теоремы Ферма

>> No.4869261  

Я "Кант-кун". Можно тут задать вопрос? Хотя я даже не знаю, какой. Я придумал способ классификации "текстур" объектов. Строго могу сказать только то, что чем-то способ похож на это:
https://en.wikipedia.org/wiki/Viola–Jones_object_detection_framework
И что у этих текстур ещё должны быть какие-то "топологические" свойства

>> No.4869269  

>>4869261
Запили прототип.

>> No.4869275  

>>4863734
дискретная

>> No.4869280  
Файл: kuri221.jpg -(20 KB, 220x253, kuri221.jpg)
20

>>4869234
Нету таких, наверное, чисто в теории многие мои задачи сводимы к решению счётной системы полиномиальных уравнений c коэффициентами в С, но это сведение настолько бесполезное и настолько отдаляет от сути задачи, что наверное его лучше было бы даже не упомянать.
>>4869261
Можно, задавай, как придумаешь! Что придумал - то молодец конечно!

>> No.4869285  
Файл: Dimension_levels.svg.png -(17 KB, 500x168, Dimension_levels.svg.png)
17

В математике есть просто числа (размерность 1) комплексные числа(когда есть Re и Im т.е. фактически две характеристики), есть кватерионы (1 i j k - 4 параметра), октонионы (8) и сенедионы (16) и думаю можно дальше сделать с 32 64... почему именно степени двойки (1 2 4 8 16), бывают ли какие-нибудь другие гиперкомплексные числа, где число параметров не степень двойки?

>> No.4869287  

>>4869285
Можно сделать сколько угодно чисел, но набор таких же свойств как у целых или комплексных не всем получится задать. Например кватернионы некоммунитативны, октонионы неассоциативны, а если сделать коммутативную систему, то в ней ещё что-нибудь пропадет.
Так-то есть векторы и матрицы, в них вообще сколько угодно элементов, но свои свойства.

>> No.4869289  

>>4869287

>Некоммутативны

Фикс. Хотя необщительные числа - тоже идея.

>> No.4869290  
Файл: kuri222.jpeg -(6 KB, 180x281, kuri222.jpeg)
6

>>4869285

>и думаю можно дальше сделать с 32 64...

Да, это процедура Кэли-Диксона.

>почему именно степени двойки (1 2 4 8 16) бывают ли какие-нибудь другие гиперкомплексные числа, где число параметров не степень двойки?

Вообще "гиперкомплекные числа" это не очень точный термин, и скорее больше для рассказов школьникам используется. Если под этим словом понимать алгебры Клиффорда, то там выходит это примерно по такой причине что если у тебя e_1,...,e_n это некоторые новые символы, которые ты добавляешь к R чтобы получить алгебру (как ты добавляешь i к R чтобы получить комплексные или i,j,k к R чтобы получить кватернионы), то e_i1 e_i2 ... e_ik c i1<...<ik тоже должны быть частью твоей алгебры и мы получаем \sum_{k=0}^n C_n^k = 2^n. Если не понял, то короткий ответ: по построению. Но если тебя вообще любые R-алгебры инересуют (пусть даже слабо похожие на комплексные числа, а потому к ним слово "гиперкомплексные" не очень хорошо применять), то соордуить трёхмерную несложная задача, скажем прямая сумма трёх R такой будет.

>> No.4869291  

>>4869287

>Так-то есть векторы и матрицы, в них вообще сколько угодно элементов, но свои свойства.

Ну а в чем особенность всех этих гиперкомплексных чисел, почему там надо чтоб число элементов было как степень двойки?

>> No.4869293  

>>4869291
Если особых свойств не требовать, то не обязательно степень двойки, можно сколько угодно элементов брать. А если налагать ряд ограничений, то для одного будут только целые и комплексные, для другого кватернионы добавятся, а если ещё ослаблять то и другие системы. Почему? Ну такая структура реальности, можешь посмотреть например
https://ru.wikipedia.org/wiki/Теорема_Фробениуса

>> No.4869299  
Файл: ftfy.png -(418 KB, 1186x296, ftfy.png)
418

>>4869293П

>Почему? Ну такая структура реальности

Позвольте, но разве математика обязана описывать какую-то реальность? Математика конечно была построена на основе наблюдения за реальностью, да, но тут что-то явно не сходится. Натуральные числа используются для обозначения количества чего-то (у меня 3 ореха, меняю их на одно яблоко). Рациональные числа это когда мы хотим выражать какие-то куски от целого (у меня пол яблока, меняю их на полтора ореха). Есть дальнейшие расширения с отрицательными (я тебе даю сейчас яблоко, а у тебя сейчас орехов нет, на тебе висит долг -3 ореха), со всякими вещественными, иррациональными числами я могу придумать пример, который их связывает с реальным миром, но когда там какие-то октонионы, то я не вижу тут очевидной связи (хотя может она есть где-то в квантовой физике)... или любая математика каким-то образом описывает структуру реальности?

>> No.4869303  

>>4869299
Ну ты можешь вообще принять 2*2=5 за истину. Проблема в том, что из этого потом можно вывести вообще любое равенство, истинное и ложное. Такая структура будет очень богатой на возможности, но малополезной практически.

>> No.4869304  

>>4869299

>Натуральные числа используются для обозначения количества чего-то (у меня 3 ореха, меняю их на одно яблоко).

Вот тут уже проблемы. У тебя нет трёх орехов, у тебя есть орех, орех и орех. То, что их можно свести в три ореха уже абстракция. Конечно попроще и попонятнее октонионов, но идея та же.

>> No.4869305  
Файл: kuri225.jpg -(141 KB, 900x860, kuri225.jpg)
141

>>4869293
Только "вещественные" вместо "целые"! Эта теорема это более-менее про то, что только S^1 S^3 и S^7 параллелизуемы, а что остальные не параллелизуемы - это вычисление полного класса Штиффеля-Уитни + вещественная периодичность Ботта, ну короче да, простыми словами не объяснишь, так как причина глубокая так-то.
>>4869299
Ты мои ответы специально игнорируешь? Я ведь стараюсь! Я ведь сказал, что если брать то (неоправдано широкое) определение гиперкомплексных, что в википедии - то "гиперкомплексные" будут в любых размерностях, а если брать какое-то более-менее вменяемое, то будет просто по построению. Ну типа "гиперкомплексные - это те которые мы получаем такой-то процедурой, а такой-то процедурой мы получаем только 2^n потому что вот так вот устроена процедура", то есть тавтология по сути.

>> No.4869306  

>>4869305

>Только "вещественные" вместо "целые"!

Да, что-то затупил, спасибо за фикс.

>> No.4869307  

>>4869305

>Ну типа "гиперкомплексные - это те которые мы получаем такой-то процедурой, а такой-то процедурой мы получаем только 2^n потому что вот так вот устроена процедура", то есть тавтология по сути.

ОК, понял. Есть процедура построения каких-то там алгебр, и вот поэтому там 2 4 8... А чем хороши числа, которые по такой процедуре были синтезированы? Ну вот есть просто комплексные числа, sqrt(-1)=i, ок, хорошо, допустим. Что полезного в более "расширенных" алгебрах, получаемых по этой процедуре Кели-Диксона? Описать ведь можно еще много всяких разных алгебр и понапридумывать всяких других операторов, например перемножения матриц и проч.

>> No.4869308  

>>4869307
Кватернионы например весьма полезны если ты хочешь писать трехмерный движок. Неплохо упрощают математику.

>> No.4869311  

А есть ли алгебра над бесконечными битовыми векторами?
Ну, грубо говоря, цифр у нас только два: 0, 1, и работает это примерно как двоичная позиционная система счисления, но их может быть бесконечность. Например вот соответствие:
0 это - {0, 0, 0, 0, 0, 0, ...}
1 это - {1, 0, 0, 0, 0, 0, ...}
2 это - {0, 1, 0, 0, 0, 0, ...}
3 это - {1, 1, 0, 0, 0, 0, ...}
-1 это - {1, 1, 1, 1, 1, 1, ...}
-2 это - {0, 1, 1, 1, 1, 1, ...}
-3 это - {1, 0, 1, 1, 1, 1, ...}
И чтоб были какие-то непонятные периодические штуки, вида
{1, 0, 1, 0, 1, 0, 1, 0 (и дак далее повторять 1 0 бесконечное число раз)}
которые вообще никакому числу не соответствуют, но с которыми определены операции
{1, 0, 1, 0, 1, 0, 1, 0 ...} + {0, 1, 0, 1, 0, 1, 0, 1 ...} = {1, 1, 1, 1, 1, 1, ...} = -1
в результате которых получаем обычное число "-1"
Что-то такое математики описывали в своих теориях?

>> No.4869312  

>>4869311
Не совсем то, но можешь p-адичные числа глянуть.

>> No.4869313  
Файл: Steins-Gate-Makise-Kurisu-Christina-80CM(...).jpg -(25 KB, 220x220, Steins-Gate-Makise-Kurisu-Christina-80CM(...).jpg)
25

>>4869307
Хороший вопрос, по правде говоря не знаю чем они так выделяются среди всех flexible и степенно-ассоциативных R-алгебр. Но вот, как уже было упомянуто первые четыре члена выделяются тем, что это единственные альтернативные алгебры с делением, по теореме Фробениуса. Спрошу твой вопрос у людей поумнее чуть позже, может они что-то скажут.
>>4869311
Да, это в точности 2-адические.

>> No.4869432  
Файл: kuri226.jpg -(10 KB, 300x225, kuri226.jpg)
10

>>4869313
Целые 2-адические, если быть точнее.

>> No.4869477  

>>4869432
но ведь если записать в этих целых 2-адических такую периодическую последовательность:
{1, 0, 1, 0, 1, 0...}
то никакому целому числу это не соответствует, это вообще непонятно что будет. Арифметика Пеано такое не описывает, можно это представить в виде суммы степеней двойки, 2^0 + 2^2 + 2^4 + 2^6... но это будет просто бесконечность. Да и неясно вообще, положительное это число или нет, если отрицательные числа кодируются бесконечным хвостом из единиц, а тут хвоста никакого и нет!
Но если это интерпретировать как число между от 0 до 1 включительно, то тогда это приобретает какой-то смысл

{1, 1, 1, 1, 1, 1, 1, 1... } это будет:
\sum_{n=1}^{\infty} \frac{1}{2^{n}} = 1

ну а чтоб {1, 0, 1, 0, 1, 0...} надо сделать

\sum_{n=1}^{\infty} \frac{((-1)^{n+1}+1)/2}{2^{n}} = 0.66666...

эта штука ((-1)^{n+1}+1)/2 нужна чтоб то 1 то 0 было в числителе.
А эти 2-адические числа допускают вообще апериодическую запись, типа
101001000100001000001...
когда количество нулей между единицами постоянно увеличивается на 1? Или там по задумке разрешены только периодические записи?

>> No.4869493  
Файл: kuri228.jpeg -(9 KB, 198x254, kuri228.jpeg)
9

>>4869477
Арифметика Пеано не обязана описывать 2-адические числа, она придумана чтобы описывать натуральные числа (ну или, по крайней мере, нечто, что похоже на натуральные числа). Чтобы последовательность 2^0 + 2^2 + ... сходилась к числу инверсная битовая запись которого выглядит как 101010... нужно переопределить "сходимость" в 2-адическом смысле, и ты даже понял как: сперва определим модуль (норму) 2-адического числа следу если на i-ом месте у числа стоит бит, то нужно считать что это добавляет 1/2^i в модуль, таким образом действительно |1111...| = 1 и |1010...|=2/3 таким образом по этой норме мы будем иметь как раз \sum_n 2^2n = 10101010... как мы того и ожидали. Норма любого 2-адического числа, как ты заметил, действительно лежит на отрезке [0..1], но на 2-адических числах невозможно придумать линейного порядка так, чтобы он был согласован с умножением и сложением, поэтому говорить что "это число между 0 и 1" некорректно. Не обязательно переодические, разрешены любые последовательности. Довольно круто что ты до такой абстрактной конструкции сам додумался, на самом деле, не могу не похвалить!

>> No.4869650  
Файл: MUG590-adventure-in-time-mug (1).JPG -(362 KB, 2000x1970, MUG590-adventure-in-time-mug (1).JPG)
362

>>4869280

> Можно, задавай, как придумаешь!

Задам тогда общий вопрос, по теме. Есть ли ещё примеры подобного?:

3Blue1Brown, "The hardest problem on the hardest test"
https://www.youtube.com/watch?v=OkmNXy7er84

Что-то типа "самоулучшения" доказательства по ходу решения, можно сравнить два в чём-то похожих решения

> Что придумал - то молодец конечно!

Тебя точно не интересует классификация, например, лиц? Шахматных поз? Это что-то типа "практической топологии":

Просто вместо "дырок" фокусируешься на любой какой-то паре свойств объекта (свойств в выбранных областях; и области как-то связаны). Например, замечаешь что где-то объект вытянут, а где-то закруглён (как, например, кружка (циллиндр) или Джейк the Dog)(иллюстрации есть в моём треде, и их ещё появится)

Если нет, то не буду больше мешать треду
>>4869269

> Запили прототип.

Так это метод и для людей тоже. (всё, не мешаю, на сторонние сообщения не отвечаю)

>> No.4869704  
Файл: kuri229.jpeg -(9 KB, 189x267, kuri229.jpeg)
9

>>4869650
Я, если честно, не понял где там "самоулучшение доказательства", там доказательство, по-моему, весьма одноходовое: придумать естественное действие Z_2 х Z_2 на конфигурационном пространстве троек точек так, чтобы в каждой орбите этого действия оказалась ровно одна нужная. Энивей красивых и остроумных доказательств в математике много! На похожей идее (подействовать группой на конфигурационное пространство комбинаторных объектов и выбрать по-умному что-то из орбит), скажем, построен подсчёт всяких штук при помощи леммы Бёрнсайда теоремы Пойя, скажем, задача об Ожерельях, только там действие группы чуть более нетривиальное и требует чуть более тонкого анализа. Если опишешь реквест как-то подробнее - я постараюсь вспомнить задач по больше.

Я же без иронии хвалил! Меня мало что интересует в математике (а в жизни - ещё меньше), но этот тред я создавал не для того, чтобы писать о том, что меня интересует, а чтобы вести изи-толки по-поводу того, что интересует других, так что можешь смело писать - буду только рад! Но то что ты описываешь, честно говоря, звучит как обычный перцептрон (даже не мультилеер), там ведь тоже каждый нейрон отвечает за какой-то "топологический признак". Чем тебе не нравятся перцептроны?

>> No.4869711  

>>4869704

> Если опишешь реквест как-то подробнее - я постараюсь вспомнить задач по больше.

Прости, я из этого всего знаю разве что видео о "Necklace problem" (того же автора). А ты получается согласен со сведением работы человеческого ума к математическому формализму?

Не-одноходовость для меня в том, что мы ввели объект, с помощью которого можно лучше переформулировать всю проблему вообще. В общем то, о чём говорит сам автор (однако развития этой темы на канале вроде нет)... Хм, а как тогда описывается первое доказательство? Вообще ноль-ходовое что ли? (То есть интересно, конечно, что в математике есть вещи, которыми можно описывать какие-то доказательства)

> так что можешь смело писать - буду только рад! Но то что ты описываешь, честно говоря, звучит как обычный перцептрон (даже не мультилеер), там ведь тоже каждый нейрон отвечает за какой-то "топологический признак". Чем тебе не нравятся перцептроны?

Мне и писать больше нечего. Не знаю, как работают перцептроны и другие сети, просто видел объяснение 3Blue1BrownComputerphile) и из-за этих видео и общего знания подумал:
1) Никто не парится, что должна делать сеть на высшем уровне абстракции. В частности, "feature construction" является проблемой судя по статье на вики (https://en.wikipedia.org/wiki/Feature_(machine_learning)#Selection_and_extraction)
2) Идея, которую описал, требовала бы веру в возможность классификации лиц (и прочего)/веру в существование этих "текстур"... Эта идея требовала бы придерживаться определённых идей о том как работает мышление в конце концов
3) Создалось впечатление, что у сетей вообще нет никаких глобальных стратегий работы
4) Другие проблемы (переобучение/проблемы с переносом знания) косвенно указывают на то, что сети так не работают. И в общем люди вроде говорят только о формализме на низшем уровне, не о том что могло бы помочь и другим людям тоже

Если это существует, это должно быть описано как общий метод, причём тут конкретная модель?

>> No.4869716  
Файл: kuri230.jpg -(1678 KB, 3840x2160, kuri230.jpg)
1678

>>4869711
Наоборот, считаю что даже сама математика в него местами не очень вписывается, не говоря уже о человеческом мышлении.

Ну пусть будет два шага, первый замечание того, что группа действует, а второй вывод из этого, мне не принципиально, со вторым решением (которое по ссылке?) нет времени разбираться сейчас, какое-то техническое. Вообще я вот какую мысль хотел донести: когда у тебя на объекте действует группа - это всегда хорошо, а это решение в видео это всего лишь игрушечная иллюстрация этого принципа. Например есть теорема о том, что свойство ядерности переносится на всю С*-алгебру с неподвижной точки действия компактной группы на ней.

Про МЛ знаю мало, но не очень понимаю, почему нужно считать проблемой то, что выделяемые алгоритмом фичи плохо описываются нашим языком с его бедным набором прилагательных вроде "округлое, контрастное, прямое, одноцветное" и т.д. зачем загонять машины в рамки нашего языка только потому, что мы сами ими ограничены? И зачем им нужно создавать впечатление для нас о том, что у них есть глобальная стратегия? Они же не в предвыборной гонке учавствуют, в конце концов, а работают. Расскажи что ты понимаешь под текстурой? И они работают не так как что? Про общий метод - нету к сожалению какого-то универсального языка, который бы позволял описывать какие-то "общие методы" так, чтобы это не звучало как-будто об этом говорили уже сто раз, поэтому чтобы высказать нечто по-настоящему нетривиальное углубляются в низкоуровневые детали. Это проблема, с которой я сталкиваюсь прямо сейчас, потому что я не понимаю чем твоя идея чтобы машина выделяла конкретные фичи (которые ты называешь текстурами) и затем искала эти фичи в последующих данных отличается от перцептрона, ну кроме того, что фичи тебя не устраивают своей кажущейся на глаз хаотичностью.

>> No.4869738  

>>4869716

(По ссылке на ютуб два решения, я только о ютубских говорю.) Про группы и прочее не знаю, прости

> Про МЛ знаю мало, но не очень понимаю, почему нужно считать проблемой то, что выделяемые алгоритмом фичи плохо описываются нашим языком с его бедным набором прилагательных вроде "округлое, контрастное, прямое, одноцветное" и т.д. зачем загонять машины в рамки нашего языка только потому, что мы сами ими ограничены?

Прилагательные могут быть и сложными/"новыми", неизвестными людям. Но одна из идей в том, что при определённом контексте как раз может быть достаточно использовать лишь пару бедных эпитетов

> И зачем им нужно создавать впечатление для нас о том, что у них есть глобальная стратегия? Они же не в предвыборной гонке учавствуют, в конце концов, а работают.

Это не обязательно требование, может быть просто отличие

> Расскажи что ты понимаешь под текстурой?

Пару областей на лице или картине или шахматной позиции и "содержание" этих областей (которое невозможно до конца выразить, просто обведя эти места). Вот алгоритм Виолы и Джонса, например — это формы (прямоугольники, например) + содержание этих форм (яркость пикселей)

Просто вместо того чтобы замечать то, как меняется яркость из области в область, замечаешь как меняется какой-нибудь (геометрический, например) паттерн.

>> No.4869743  
Файл: 1470069561124267048.jpg -(35 KB, 400x540, 1470069561124267048.jpg)
35

>>4869738
Пересмотрел видео внимательнее, не понял где там два решения, вроде же одно.

А паттерны формируются на ходу или мы сами их придумываем? Если сами, то это просто детекция по маске (я даже на фриланс в школе распозновалку автомобильных номеров по этому принципу писал) а если формируются на ходу, то самое интересное ведь в принципе по которому они формируются. Не мог бы в общих чертах описать?

>> No.4869747  

>>4869743

> Пересмотрел видео внимательнее, не понял где там два решения, вроде же одно.

Первое оборвано на полуслове, не доведено до конца: про интегрирование (или чего-то там) областей сферы.

> А паттерны формируются на ходу или мы сами их придумываем? Если сами, то это просто детекция по маске (я даже на фриланс в школе распозновалку автомобильных номеров по этому принципу писал) а если формируются на ходу, то самое интересное ведь в принципе по которому они формируются. Не мог бы в общих чертах описать?

Я нашёл только примеры совсем простых масок на википедии, но наверное это можно сравнить с маской (ведь сам же сослался на то, что наверное тоже маска)

В смысле сообщаем ли мы их машине или она сама их придумывает? Мне сложно сказать, т.к. моя идея и для людей тоже. И не знаю, важно ли это. Но, кажется, могу сказать, что паттерны по-любому будут немного, но меняться при получении нового опыта

Не согласен, что самое интересное в принципе (кроме описанного принципа двух областей). Я пытался осмыслять свои методы классификации и идея выделеняе двух областей для меня была совсем неочевидна (были тыщщи других вариантов, в частности связанные с вопросами надо ли "нормализовать" то что классифицируешь, т.е. приводить сначала к единому виду — какое-то там количество областей даже на краешке сознания тогда не было, этой идее даже придти было неоткуда особо (я думал, что дело в комбинации/синергии свойств, и это конечно казалось мне намного важнее всяких областей о которых из-за этого даже подумать не мог); когда же брался раньше обводить что-то, то думал что должен получать уникальные узоры для уникальных объектов, то есть полностью фиксировался только на обводке). Но, понимаю, без контекста это может выглядеть как ничто, и я тут не в праве возмущаться

Я вообще начал с философской концепции, в которой объекты как бы состоят из деталей, но в то же время из уникальных деталей (словно конструкторы, из которых собирается только по одному объекту: словно эти объекты абсолютно отличны друг от друга, каждым своим свойством и каждой "деталью"). Истинность такого подхода была бы удобна тем, что ты можешь классифицировать вещи по какому угодно признаку (т.к. отличаются все признаки). Это был анти-комбинаторный подход (отсюда и понятие "текстуры"). И всего этого тоже уже не видно за настоящей формулировкой

А ты видел картинки из треда? Вот например: ( в частности с Джейком)
http://iichan.hk/b/src/1552285487331.jpg

>> No.4869767  

>>4869493

>Арифметика Пеано не обязана описывать 2-адические числа, она придумана чтобы описывать натуральные числа (ну или, по крайней мере, нечто, что похоже на натуральные числа).

А какая есть аксиоматика для этих 2-адических чисел, которая б их так же точно и полно описывала, как аксиомы Пеано описывают натуральные числа?

>> No.4869820  
Файл: kuri231.jpg -(17 KB, 236x360, kuri231.jpg)
17

>>4869767
Не сказал бы, что аксиомы Пеано что-то там полно и точно описывают, но вопрос вкуса, так сказать. Вопрос хороший, думаю есть, но я о них не знаю, в любом случае есть гораздо-более полезные для таких вещей характеризации через универсальное свойство: во-первых целые 2-адические это проективный предел системы ->Z_8->Z_4->Z_2 где каждый морфизм это умножение на 2, а во-вторых. ещё более абстрактно, это эндоестественные преобразовании забывающего функтора из категории абелевых групп с 2-кручением в категорию множеств. Первое определение можно перевести в громоздкую схему аксиом, принципиальных трудностей тут нет.
>>4869747
А, ну да, по ссылке под видео оно объяснено, я про него и говорил, но оно какое-то техническое поэтому лень было разбираться.

Картинки не видел, я редко в твой тред захожу так как редко выхожу из своего - он у меня просто во вкладке висит вверху и я его обновляю раз в пару часов. Ты хочешь детектить лицо, как, грубо, одну овальность внутри другой? Думаю такое уже есть, надо будет поспрашивать.

Про то, что каждый объект отличен от другого - это мне напоминает витгенштейновскую тему, у него как раз в ЛФТ каждый объект представляет из себя нечто, что языком ухвачено быть не может. По крайней мере как я его понял когда его читал лет 5 назад.

>> No.4869916  
Файл: kuri232.jpeg -(6 KB, 227x222, kuri232.jpeg)
6
>> No.4869918  

>>4869820

>в любом случае есть гораздо-более полезные для таких вещей характеризации через универсальное свойство: во-первых целые 2-адические это проективный предел системы ->Z_8->Z_4->Z_2 где каждый морфизм это умножение на 2, а во-вторых. ещё более абстрактно, это эндоестественные преобразовании забывающего функтора из категории абелевых групп с 2-кручением в категорию множеств. Первое определение можно перевести в громоздкую схему аксиом, принципиальных трудностей тут нет.

Как много непонятных слов!
Понятие морфизма немного знакомо мне из теории категорий (про нее я немного читал, да. Интересовася всяким там хаскелями) но у теории категории есть своя аксиоматика, определения для понятий вроде "проективный предел"... теория категорий является явно избыточным инструментом для описания этого, аксиомы для натуральных чисел отлично описываются без каких-либо понятий из теории категорий. Аксиоматика Пеано формулируется в логике первого порядка (ЛПП), а теория категорий это намного сложнее, там какие-то моноиды в категории энофункторов, иломорфизмы и вообще непойми-что!

Но вообще, у меня возник еще один вопрос. Получается, пользуясь теорией категорий (ТК), мы можем выразить некий новый класс математических объектов (типа тех же p-адичных чисел) и каким-то образом вывести аксиоматику из каких-то определений, да? А описывает ли ТК саму себя, т.е. может ли сама ТК быть объектом рассмотрения с точки зрения ТК? Насколько ТК метаматематична по своей сути? Описуемы ли теории (системы аксиом) в ТК?

>> No.4869945  
Файл: kuri233.jpg -(61 KB, 648x1024, kuri233.jpg)
61

>>4869918

>аксиомы для натуральных чисел отлично описываются без каких-либо понятий из теории категорий. Аксиоматика Пеано формулируется в логике первого порядка (ЛПП)

Только как известно по теореме о неполноте эти аксиомы описывают не только натуральные числа, но и кучу нестандартных моделей в придачу (по крайней мере если использовать стандартную первопорядковую семантику). Аксиомы Пеано они скорее нужны чтобы формализовать понятие доказуемого утверждения о натуральных числах, чем сами натуральные числа.

>Получается, пользуясь теорией категорий (ТК), мы можем выразить некий новый класс математических объектов (типа тех же p-адичных чисел) и каким-то образом вывести аксиоматику из каких-то определений, да?

Ну типа того, её исторически за тем и придумывали, чтобы формализовать некоторые вполне определенные и понятные всем конструкции (что морфизм гуревича функтореален). По-поводу аксиоматики cовсем прямо не можем, но можем в следующем неинтересном смысле: технически любое определение в любой теории (скажем, в теории множеств) это extension by definition, то есть добавление в синтаксис нового символа и аксиомы связанной с ним, но правда это определение чего-то внутри теории множеств, а не "само по себе", так что это не очень интересно мне кажется. Что интереснее, так это то что если категория достаточно хорошая, то у неё есть внутренний язык (internal language) который представляет из себя некоторую теорию типов, на которой можно записывать всякие утверждения об объектах этой категории. Но этот язык описывает всю категорию целиком, а не отдельные её члены.

> А описывает ли ТК саму себя, т.е. может ли сама ТК быть объектом рассмотрения с точки зрения ТК?

Технически - возможно описать некоторую первопорядковую теорию "теорию 2-категории всех категорий" в которой было бы достаточно синтаксиса для определения любой категории, которую можно определить при теоретико-множественном подходе. Но на практике никто пока не сделал, но у Лавье и Шульмана были попытки, всякие ET2CC и т.д. но там какие-то недостатки были, вроде. Но если не пытаться всё зааксиматизировать, то с 2-категорией всех категорий люди вроде работают как-то.

>Насколько ТК метаматематична по своей сути?

Теория 1-категорий математична, а вот теория n-категорий мне кажется довольно-таки метаматематична пока что, так как нету даже вменяемых определений 3-категории, еле- еле, вернее есть, но их там несколько в зависимости от слабости/сильности композиций, и в комбинаторике которых разбирается 1.5 человека, есть более менее вменяемое определение 2-категорий (бикатегорий) и всяких (\infty,1)-категорий (слабые комплексы Кана).

>Описуемы ли теории (системы аксиом) в ТК?

Ну как я уже говорил принципиально что угодно можно описать, но ещё не сделали (да и не особо нужно мне кажется). Ещё раз повторю, что из некоторых категорий целиком извлекается синтаксис который объекты и морфизмы этой категории описывает и наоборот, любая теория типов естественно имеет категорную семантику. Это инстанция syntax-semantics duality. Ещё есть категорный подход к логике, в котором рассматривается категория всех моделей какой-нибудь теории, или категория самих теорий и т.д.

Как-то немного путано ответил, но и вопросы были расплывчатые, так что если что задавай уточняющие вопросы!

>> No.4870061  
Файл: kuri233.jpeg -(10 KB, 178x283, kuri233.jpeg)
10

Бюрократия - худшее изобретение человечества!

>> No.4870165  
Файл: 1280px-Booba-Kiki.svg.png -(38 KB, 1280x653, 1280px-Booba-Kiki.svg.png)
38

>>4869820

Я на самом деле вчера на Реддите так же опростоволоcился/сел в лужу. Как "вдруг" пошли вопросы (которые сам для себя исключил/"не включил") — так я и растерялся!

Поэтому (и раз философское отвлечение оказалось не лишним для тебя) на всякий случай перескажу один раз короткими(? получились не короткими, прости) пунктами:

  • Была идея, что нужно искать "комбинации свойств" ("переплетения": будто два выразимых свойства образуют некую невыразимую комбинацию (уникальную для какого-то одного объекта), по аналогии с твоими словами о ЛТФ). Но из этой идеи было не очень понятно, что это за переплетение и откуда оно берётся (было лишь условие уникальности). Идея "двух областей" делает шаг в сторону более конкретного и более простого понимания (не надо даже упоминать больше всю эту философию)
  • Была идея, что мозг оперирует "комбинациями семантических тегов/облаками тегов". Но это совсем ничего не говорило. "Две области" уточняют идею и конкретизируют её для изображений — "теги" это более-менее легко выразимые языком свойства/"формы" областей из предыдущего пункта (которыми легко оперировать и которые легко передавать), а "опыт" (или более мелкие теги/"содержание" форм) за ними отвечает уже за "невыразимую" сторону идей
  • Это рекурсивный/итеративный метод. Он может даже претендовать на универсальный алгоритм мышления — просто вместо форм (обведений) на картинах у тебя будут обведения на "облаках" тегов для других сфер мышления. Ведь и обведения на картинах это на самом деле обведения на облаках топологических тегов (ведь мы уже как-то проанализировали то, что мы видим)
  • Философия метода в том, что какой-то паттерн на 100% имеет смысл только в контексте другого паттерна (абсолютных свойств не существует). В этом фишка метода, предельная абстракция и передаваемость тегов между "кластерами опыта" (для человека это может быть даже опыт от разных органов чувств, т.е. синестезия/идеастезия). А для настоящих сетей это только побочная плюшка (Transfer learning), которую хотят получить как-то косвенно само собой (как я понял) https://en.wikipedia.org/wiki/Ideasthesia#In_normal_perception
  • (Отсутствия абсолютных свойств исключает некоторые универсальные обобщения и в итоге может иметь даже политические последствия)
  • Как писал, была идея, что можно классифицировать по любому признаку (различны все). С "двумя областями" это соответствует тому, что на лице можно выделять разные пары областей, но вывод из этого будет часто один и тот же. Это во многом удобно и позволяет одним текстурам "обучать" другие (научившись лучше различать челюсти можешь лучше научиться различать глаза и носы, например). С одной стороны это эвристический постулат (что объекты часто в чём-то самоподобны), а с другой стороны заслуга самого метода (он сам представляет объекты в наиболее самоподобном виде). Подразумевается самоподобие внутри одного объекта и подобие между разными (раз они абсолютно различны, они в каком-то смысле абсолютно одинаковы), но конкретный метод позволяет избежать лишней строгости в этих подразумеваниях
  • Также "метод" частично основывается на идее, что существует некие сверх-абстрактные (доселе не бывшие известными) свойства. Или что свойства, которые обычно подразумеваются общими, на самом деле специфичны, а специфичные общи. Например, что картины (и всё прочее) можно классифицировать по некоторому метафизическому "размеру" (на который косвенно влияет то что изображено)
  • Ты спросил, задаём ли мы паттерны или они формируются сами. Но есть же и третий вариант, который как раз описывается — мы задаём некое абстрактное условие для поиска/формирования свойств/паттернов. Или (более жалкий вариант) просто изобретаем гипер-абстракцию и пытаемся научить ей обычную машину/заточить её под способность находить такое. По-моему этот вариант интересен сам по себе: протестировать, что будет, даже если это заведомо не даст универсальную машину. Просто по-моему само по себе сложно догадаться, какое же такое условие может быть (поэтому каждое возможное уже интересно).
  • "Каждый нейрон отвечает за топологический признак" это вроде ничего не говорит об областях или их сравнении, ведь так можно и сразу всю картину анализировать (как одну область: можно один единственный признак рядом нейронов выражать). Метод абсолютно параллелен конкретной реализации (анализируй содержимое областей чем угодно). И в этом дело: то чем занимаются сейчас может оказатся просто параллельно концептуальному решению проблемы — т.е. может оказаться, что в принципе "до ума" можно было бы довести соверешнно любую реализацию.
  • (в этом была идея моего last треда) По-моему пытаться решить мышление с низкого уровня — даже антинаучно. Алхимики и флогистонщики пытались найти конкретную вещь, "создающую огонь", а потом эту вещь просто пометили как "неизвестное" (потому что ещё просто права нет о таком задумываться) и стали развивать область знаний о химических реакциях (казалось, что разгадка близко, но новые идеи по сути даже отодвинули её). Никакой Ктулху мне или даже учёному не расскажет сразу, как всё обстоит на всех уровнях, а до какого-то промежуточного вывода гипотетически может дойти даже домохозяйка. По-моему только так и возможна разгадка, через догадки о промежуточных этапах и через заполнение промежуточных областей знания (так развивал и собственные идеи, сначала надо было "заполнить уровень" философии и общих качественных заметок о методе, а как нафилоствовался и набрался опыта — пошёл дальше так "уровнево" вообще все идеи развилвались: наигрался с какой-то формулировкой — переформулировал проще/шире). (тоже имхо) Низкоуровневые штуки сообщают небанальные факты только для собственного матана, а не "вообще" — я "как физик" пытаюсь сообщить тебе некоторые [промежуточные] эффекты, просить сразу рассказать как всё на самом низком уровне может быть не честно (даже если так делают сейчас)
  • Была идея, что люди в каком-то смысле абсолютно безвольные "статистические машины" (которые на всех уровнях подчинены достаточно простому закону), отсюда у меня интуиция что и всё описанное тоже возникает "само". Например, я верю (или это скорее просто удобное для иллюстрации упрощение), что представляю картины (да и любые изображения) массивами/облаками точек. То есть как будто картина это некое "реальное" место, которое даже состоит из "реального" материала. И по сути всё, что я делаю, это просто "обвожу" скопления наибольшего количества информации, то есть "точек" (эти "точки" могут быть стартовыми "топологическими тегами"). Выражаю количественные вещи качественно: "обведением" может быть представление "текстуры материала" или "поверхности" или "формы" (во всём этом я уже набрался опыта, поэтому сейчас просто буквально условно обвожу). Так может оказаться и с мышлением: мол нам лишь кажется, что мы используем логику или какие-то принципы ("качественное"), а на самом деле мы просто абсолютные рабы наших интересов (одинаковая информация = одинаковые выводы) и все наши выводы это просто "поллюции" от наших интересов (понимаю, что эту мысль тоже надо уточнять, чтобы она отличилась от труизма, но надеюсь ты просто понял идею: я сам по аналогии с физикой просто решил принять обе идеи сразу, типа количественно-качественный дуализм). Типа мы мыслим, в каком-то смысле, на уровне простейшего кластерирующего алгоритма ("обведи самые приятные/важные для себя вещи и сделай самый приятный/важный для себя вывод"). Но эта идея ещё не дозрела так, как дозрела идея "двух областей" (однако можно сравнить с Меметикой Докинза, где скопления информации похожи на живых существ со своими собственными потребностями)
  • По сути буквально всё что написал (и то как расставил акценты везде) "топит за" Коннекционизм: https://ru.wikipedia.org/wiki/Коннекционизм. И могло бы усилить коннекционистские аргументы/отбить возражения на них в https://arxiv.org/pdf/1604.00289v2.pdf ("Building Machines That Learn and Think Like People": доступно написано не обязую читать конечно, там длинно) — поэтому-то я и считаю свою идею важной/несущей информацию (она исключяет достаточно много альтернатив!) — и поэтому странно сравнения с перцепртоном (это же парадигма, а не перцептрон!)
  • Из рекурсии и коннекционизма получаем, что сами описанные структуры могут образовывать подобия сетей. Сами концепты могут вести себя как нейроны. Можно придумать высокоуровневый аналог "Backpropagation", не с нейронами а с концептами (по сути описал пример в пункте про разные возможные выделения). Или высокоуровневый аналог "feature space": объект, собранный из всех известных типов объектов. Может быть и что-то ещё более интересное и забавное: например, можно задать тем "точкам" из позапрошлого пункта правила взаимодействия друг с другом, правила взаимодействия более крупных их скоплений... типа сети, которую "выплюнули" на картину (которая при этом одновременно похожа и на физическую или биологическую систему).

Но это не прямые возражения тебе, если что: я понимаю, что можно не доверять мне или считать что это всё ещё слишком общее описание. Я сам смирился с фактом, что в чужом мозге твои идеи не могут иметь такого же значения — у чужого нет твоего "топлива", которое помечает твои (даже совсем слабо выраженные) идеи как важные (либо у чужого мозга есть то топливо, которое зажигает вещи поважнее твоих идей)

Просто теперь буду спать спокойно, что всё объяснил что мог

> Картинки не видел, я редко в твой тред захожу так как редко выхожу из своего - он у меня просто во вкладке висит вверху и я его обновляю раз в пару часов. Ты хочешь детектить лицо, как, грубо, одну овальность внутри другой? Думаю такое уже есть, надо будет поспрашивать.

(Удивительно, если ты там вообще бывал!)
Пункты как-то отвечают на твой вопрос? Овал это просто условность, которой выделяю то место в котором скапливается больше всего интересного (и более-менее однородного: когда однородность нарушается, требуется ещё форма либо ещё одна "итерация" метода, чтобы представить неоднородную вещь однородной [под-]текстурой): наверное, практическая реализация будет детектить ирерахию "скоплений" (чего-то)

"Скопления" можно помечать и чертой, и точкой, и другими формами наверное (это просто языковой символ наполовину, подсказка уму, костыль) — наверное, для практики можно остановиться на одних овалах

(Может ещё быть проблема, что формально концептуальные идеи нельзя отличить)

Я всё-таки не понимаю: разве может "просто быть" то, что вообще требует проглотить тонны идеологии? Если бы это было, это должно было бы отмечаться как какой-то аргумент в спорах, как "тест идей такой-то идеологии"

Вот более разнообразная картинка:
http://iichan.hk/b/src/1552204277584.jpg
Мне эти костыли помогли для себя подкрепить какие-то свои идеи
И вот ещё: (но я там пытался экспериментировать/делать усилие над собой, может быть меньше понятного)
http://iichan.hk/b/src/1552421274099.jpg

Я в общем постараюсь "успеть" сам добиться чего-то в области шахматных позиций с этим, если всё это не способно убеждать людей.

Конечно, делить на области кому-то должно было приходить в голову. Но насколько последовательно это могло быть реализовано без пачки всех остальных идей? Насколько широким виделось применение? (если применение виделось узким, это могло повлиять на ряд решений при реализации)

> По крайней мере как я его понял когда его читал лет 5 назад.

По одной из идей в этом и суть, что мы "призмируем" знание (обрубаем маловажное и достриваем важное). Из-за этого по-моему и хорошо, что тред в каком-то смысле посвящён тебе как личности

В каком-то смысле вообще не важно, что проходит через призму. Она просто "качается" (как мышца) от проходящего через неё опыта (в этом идея) с течением времени

Хотя можно видеть и пессимистично: мы находимся в яме своего эгоизма, которую можно лишь углубить. Тут можно вспомнить ещё "монады" математика Лейбница (вот же связь с темой треда! поздно вспомнил)

Монады Лейбница — это метафизические единицы, абсолютно неделимые и целостные, подобно математическим точкам. Монада — это не арифметическая «единица», поскольку в математике любая единица делима. Похожи своей неубиваемостью на атомы Демокрита, но точно не атомы в нашем понимании, поскольку атомы материальны, следовательно протяжённы и делимы, их только по недоразумению называют «атомами».

Для начала монаду можно представить как одушевленный фрактал, который стремится выразить маленьким собой всю суть Вселенной. Монада это «cogito ergo sum» Декарта, перенесенное на все отдельные явления в мире. По этой причине каждое явление sum, то есть субъект и сознание. Монады самого низшего уровня — «спящие»: они не осознают себя, и, тем не менее, мы не можем сказать, что у спящего нет восприятия.

Или так оно только в пересказе этого человека

>> No.4870270  

>>4866721
>>1+1=2

>В каком контексте? Один математик плюс один студент равно полтора математика.

Контекст задается формальным определением синтаксиса и семантики. Если у нас какая-то особая параллельная вселенная, где одинаковые вещи не могут находиться в одном месте (ну типа если мы возьмем 1 и 1 камень и положим их в одно ведро, то произойдет какая-то химическая или ядерная реация, и орехов станет 1 или даже 0) то там действительно 1+1 может равняться чему-то другому. Вообще, математика это язык описания моделей для всяких естественных наук (физики, химии). Если б мы жили в какой-то другой реальности, где предметы так просто не складываются, то и математика там будет какой-то другой (по крайней мере в базовой ее форме, которую дедсатовцам и школьникам начальных классов объясняют на палочках)! Но и тогда можно было б придумать и ту математику, которая б описывала тот мир, в котором мы тут с вами живем. Можно придумать математику, которая описывает то, что никакого отношения к реальным законам физики и пространства/времени не имеет! А если б мы жили в двумерном пространстве, мы б всё равно могли бы "придумать" трехмерное пространство и формализовать какие-то его свойства.

>Это не так считается. Возьмем менее тривиальный пример: 81*19 и LXXXI помножить на XIX. Все еще разницы не чувствуешь?

А от чего вообще должна возникнуть разница? Какая разница, через какие именно закорючки мы что-то там считаем, если результат тот же? Разница в результате может быть только если сами законы арифметики у нас другие, а форма записи тут причем?

>> No.4870396  
Файл: kuri235.jpg -(11 KB, 236x337, kuri235.jpg)
11

>>4870165
Мне теперь стыдно даже что-то короткое писать после таких больших откровений, чувствую себя, как-будто на признание в любви реагирую отмашкой, но длинные тексты не по математике (да и по математике выходит неуверенно), к сожалению, из себя выжимать не выходит, так что не принимай на свой счёт!

Мне в твоей всей идеологии не нравится во-первых то, что она заточена на применение, как-будто ты разработчикам AI хочешь какой-то дар сделать своим текстом (но не потому что они и без тебя обойдутся, а потому что, как ты сам и говорил, это может повлиять на реализацию, в данном случае - реализацию твоих идей в виде текста); во-вторых что ты вообще "задачу мышления" воспринимаешь как задачу, требующую разрешения в виде реконструкции в некоторой теории, а не воспринимаешь так, как-будто всё уже дано изначально, говоря в несколько стыдных терминах: мыслишь диахронически, а не синхронически; в-третьих несмотря на то, что ты как-будто стремишься к какой-то fuzziness cистеме, у тебя всё равно всё сводится к каким-то атомам, будь-то "сверх-абстрактные (доселе не бывшие известными) свойства" или "концеты" пусть даже они "не могут быть мыслимы сами по себе, а только в отношении к другим свойствам", собственно это и у Витгенштейна мне не нравилось, он свои "элементарные факты" ввёл только потому, что испугался корекурсию без базы делать, то есть не захотел работать с чистыми коданными. Пока на этом остановлюсь, наверное, может чуть позже перечитаю и что-нибудь ещё подмечу.

>>4870270
А ты попробуй думать, что он не не знает что 1+1=2, а что за этим его примером стояло некоторое желание, к которому не нужно относится пренебрежительно, мы же тут не в университете в конце концов!

>> No.4870558  
Файл: Alignment.JPG -(16 KB, 685x500, Alignment.JPG)
16

>>4870396

Ничего, для меня же к твоему посту прилагается целая энциклопедия неизвестных мне доселе слов. (Помещу ссылки для других или чтобы ты их прокомментировал, как с определением гиперкомплексных)

Кажется, на применение заточены сами мои идеи: моя собственная цель как в "мнемотехнике". Просто хочу помнить то, что мне нравится (шахпозы для меня как картины, даже без всяких ассоциаций)

"Диахронически" — это "сквозь время", то есть подразумевая что мы лишь какой-то этап во времени? Что настоящие машины не дают мышления, что мы не обладаем всем достаточным знанием/опытом? Что мы пытаемся приблизиться к мышлению чередой улучшающихся теорий, чего на самом деле не происходит?

"Корекурсия" как рог изобилия пролилась: там и о "коиндукции" какой-то узнал. Английская статья однако представила всё очень просто, как будто это то же самое просто в другую сторону
https://en.wikipedia.org/wiki/Corecursion
corecursion allows programs to produce arbitrarily complex and potentially infinite data structures, such as streams, so long as it can be produced from simple data (base cases) in a sequence of finite steps.

Но это видимо "с базой"? А коиндукция меня сломала
https://en.wikipedia.org/wiki/Coinduction

И ещё узнал о
https://en.wikipedia.org/wiki/Mutual_recursion#Terminology

То что ты упомянул из поста — разве это не результаты этой самой корекурсии? Об "элементарных фактах" я вроде не забочусь, т.к. как человек сам не знаю с чего именно начинает моё мышление, ну и как писал считаю что пытаться сразу установить изначальную причину/изначальный объект ("элементарный факт") неправильно

А что бы ты сам сделал/подумал на эту тему, как обладающий опытом мат. мышления? Точнее коректнее спросить просто "Как жить иначе?" (не требуя конкретную альт. модель)
(Отвлечённо:)

> Мне теперь стыдно даже что-то короткое писать после таких больших откровений, чувствую себя, как-будто на признание в любви реагирую отмашкой, но длинные тексты не по математике (да и по математике выходит неуверенно), к сожалению, из себя выжимать не выходит, так что не принимай на свой счёт!

Кстати, думал на эту тему: важно ли как мы выглядим в чужих глазах и если важно то почему? Решил что да (важно), основываясь на идеях что мораль это твоя услуга для других людей/существ и единственный способ узнать причиняешь ли ты вред другому это его собственные слова — что сам смысл морали в том что ты попадаешь в "поле" мыслей другого человека (поэтому "казаться" хорошим самоценно даже без названных до этого причин)

А ты по какому Alignment'у?

> А ты попробуй думать, что он не не знает что 1+1=2, а что за этим его примером стояло некоторое желание, к которому не нужно относится пренебрежительно, мы же тут не в университете в конце концов!

По-моему это очень хороший подход.

>> No.4870591  
Файл: kuri237.jpg -(53 KB, 427x604, kuri237.jpg)
53

>>4870558
Понятно, я вот скорее стараюсь наоборот избавиться от своего этого желания удерживать какие-то знания.

Ну, вроде этого. Под синхронией я понимаю, в частности, не задавать вопросы "почему" и не считать ответом на них какую-то более глубокую и остроумную теорию, которая в своих терминах бы вскрывала и описывала некоторый тайный внутренний механизм происходящего, а скорее просто нечто подмечать и обозначивать, что так оно и происходит (даже если так оно не происходит).

Коиндукция и корекурсия, ровно как индукция и рекурсия особо ничем между собой не отличаются, обычно говорят, что (ко)индукция это типа метод доказательства или вывода чего-то, а (ко)рекурсия это логическая фигура самоприменимости as it is, но я не вижу смысла в такого рода различии.

Рекурсия, грубо, это когда у тебя есть начальные примитивные объекты и конструкторы как строить из них новые более сложные, а корекурсия это когда у тебя есть начальные очень сложные объекты и деконструкторы как их разбивать на новые более простые. Кажется что это об одном и том же, но разница на самом деле в том, что во-втором случае мы не обязаны приходить к каким-то начальным примитивным объектам. Объясню на примере. Есть индуктивный тип "бинарная строка" он содержит конструктор "прибавить 0 или 1 к началу строки" и базу "пустая строка", таким образом ты можешь строить из пустой строки новые строки, скажем ""->"0"->"10"->"0101. А есть двойственный к этому коиндуктивный тип "бесконечная бинарная строка" у которого есть деконструктор "взять голову" и "взять хвост", и таким образом ты можешь деконструировать твою бесконечную бинарную строку "011010..."->("0","11010...")->("0",("1","1010...")) но при этом так как бесконечная бинарная строка бесконечна, то мы не обязаны упереться в пустую строку при деконструкции, так и у витгенштейна, вовсе не обязательно было вводить какие-то "элементарные факты"!

Mutual recursion - это ничего особо интересного, просто одна функция определяется через вторую, а вторая через первую, главное чтобы база в итоге была. В олимпиадном программировании часто такие штуки писать надо, но естественного примера придумать не могу сходу.

Ну, мне нравится как на эту (или почти на эту) тему рассуждает Лакан, акцентируясь на существенных эффектах публичной речи, а не на выстраивании оснований в квази-теоретико-типовом смысле, в последнии несколько лет только его и читаю поэтому из всего такого.

Думаю важно, но не потому что другой что-то там говорит, это как раз не имеет значения, а потому что сам язык устроен так, что обращаться можно только к другому. Скажем, я помню что каждый учитель выходил в моей школе и все начинали шуметь, то шумел я не потому что никто меня не видел и поэтому я мог делать нечто безнаказанное, а как раз шумел на глазах отсутствующего места учителя, в некотором роде я считал это чуть ли не жестом флирта с учителем.

Спросил друзей в чятиках, сказали что neutral neutral - им виднее!

>> No.4870634  

Я всю неделю пытался сформулировать ответ на дискуссию про абсолютность математики, но в итоге не очень чётко сформулировал, и мне стало лень его писать.

Я хотел написать о том, что если мы можем посчитать 2+2=4, то мы можем посчитать и число Пи, которое будет равно тому же самому. Что даже если пространство математика, который это считает, обладает упоротой метрикой, то рассматривая все варианты метрик, он найдёт эвклидову, и она покажется ему "естественной", то есть удовлетворяющей хорошим свойствам. А если 2+2=4 не истинно, то это либо не 2, либо не 4, либо не +, либо =. То есть в том смысле, что это вполне определённые конкретные объекты, хоть и абстрактные.

И если мы хотим изменить математику так, чтобы Пи изменилось (или какое-то другое "красивое" число вроде e), то нам придётся менять базовую логику, из которой мы исходим, когда придумываем числа. И тот-то я и залип, пытаясь понять, можем ли мы как-то формализовать эту самую логику (или, возможно, метаязык). Примерно как в http://www.ditext.com/carroll/tortoise.html (спасибо >>4870591 за ссылку).

>> No.4870646  

Пытаясь всё-таки написать что-то из того, о чём я думал, хотя мне безумно лень печатать.

Вот, допустим, мы хотим, чтобы бог мог изменить число Пи. Допустим, что наш тот самый бог это какое-то абстрактное сознание в пустоте (это может быть Больцмановский мозг, например, так как не важно, где он находится, если он не может вокруг себя ничего воспринимать). У него нет понятия пространства, у него должно быть какое-то время, но он не может измерять его. Очевидно, в таком состоянии придумать числа ему не из чего.
Но стоит дать ему хотя бы что-нибудь, что можно считать (например, количество раз, когда ему хочется кричать, но нечем), то он может начать придумывать слова вида А, АА, ААА... Но из самих этих чисел математики всё равно не построить. И именно в этом месте мы должны вложить ему какую-то базовую логику, в которую, входит, например, возможность понимать, одинаковые слова или разные. И помимо этого ещё строить какие-то гипотезы и проверять их. Например, он может запустить несколько параллельных счётчиков, и считать в одном два раза А, АА, ААА, АААА, ААААА а в другом сначала А, АА, А, АА, ААА, а потом А, АА, ААА, А, АА, и таким образом он может придумать сложение и понять его свойства. Затем он может попробовать понять, какой минимальный набор аксиом ему нужен, чтобы доказать эти свойства из этих аксиом. Имея натуральные числа, он может построить целые, рациональные, действительные, комплексные и какие угодно. И потом просимулировать у себя в сознании выдуманный мир, в котором мы живём, и именно так оно всё и происходит. Шутка.

Но значение числа Пи зависит только от той самой базовой логики, которую мы ему вложили. Которая, наверное, также является метаязыком для языка его математики. И я пытался понять, что о неё можно понять и можем ли всё-таки придумать альтернативную базовую логику... Проблема в том, что когда мы рассуждаем об этой металогике, то мы используем её же самую. И выглядит так, как будто она всё время уезжает от нас, потому что мы должны изучать и то, как металогика используется при рассуждениях о металогике, как в той басне про черепаху по ссылке выше.

Наверное, я написал полную ерунду, но надеюсь, что это не важно.

>> No.4870650  

Или можно придумать другой пример. Например, мы в нашем мире построили из NOR-гейтов машину, которая при нажатии на кнопку показывает лампочкой следующую двоичную цифру числа Пи (пусть она показывает только первые сколько-то цифр, а затем заново, чтобы избежать необходимости в бесконечной памяти).

В отрыве от семантики эта машина выдаёт просто какую-то последовательность, которая с числом Пи сама по себе общего имеет мало. Она становится числом Пи только, когда мы интерпретируем её правильным образом. И доказательство того, что машина выдаёт именно это число, тоже неотделимо от интерпретации. Но допустим, нам это не важно. Просто последовательность.

Так вот вопрос в том, можем ли мы или бог придумать такой мир, в котором NOR-гейт работает точно также (даже если устроен по-другому), гейты соединены точно так же, но последовательность она выдаёт другую?

Я верю, что не можем, но доказать я этого не могу. И может быть доказать это и невозможно, но доказать возможно или невозможно тоже не могу.

Но даже если можем как-то доказать, что не можем изменить последовательность, это не показывает, что число Пи изменить нельзя, потому что мы, возможно, можем изменить интерпретацию, так чтобы это последовательность в число Пи не складывалась.
Но показывает, что есть что-то абсолютное, что даже всемогущий бог изменить не в состоянии.

>> No.4870663  
Файл: kuri238.jpeg -(8 KB, 316x159, kuri238.jpeg)
8

>>4870634
>>4870646
>>4870650

>Но значение числа Пи зависит только от той самой базовой логики, которую мы ему вложили. Которая, наверное, также является метаязыком для языка его математики. И я пытался понять, что о неё можно понять и можем ли всё-таки придумать альтернативную базовую логику... Проблема в том, что когда мы рассуждаем об этой металогике, то мы используем её же самую. И выглядит так, как будто она всё время уезжает от нас, потому что мы должны изучать и то, как металогика используется при рассуждениях о металогике, как в той басне про черепаху по ссылке выше.

Да, я понимаю о чём ты это тоже меня тревожило, об этом пишут в любом хорошем учебнике мат. логики в начале, что как это так, мы хотим дать основания, скажим, натуральным числам, но при этом используем как минимум конечные строки вместе с огромным количеством машинерии вокруг них (вроде формирования терма по индуктивным правилам, подстановки подстроки в определенное место другой строки, конкатенации строк и тд и тд), а конечные строки - это те же натуральные числа, если забыть различать символы. И во всех учебниках от этой проблемы как-будто отмахиваются, а не решают. Но похоже что по-другому не получится, должен быть момент, когда ты принимаешь все правила игры целиком, и только после этого тебе можно что-то начать объяснять. Спекулятивно можно сказать, что это инстанция теоремы Гёделя о неполноте - не получится на нашем языке дать основания для нашего же языка. Не получится объяснить кому-то что-то о натуральных числах, если ты уже о них изначально не знаешь.

Почитай ещё:
https://math.stackexchange.com/questions/1334678/does-mathematics-become-circular-at-the-bottom-what-is-at-the-bottom-of-mathema
и далее по ссылкам, ещё об этом писал Витгенштейн:
<QUOTE>
31. Если кому-нибудьпоказывают фигуру шахматного короля и говорят "Это король", то
этим ему не разъясняют применения данной фигуры разве что он уже знает правила игры.
Кроме вот этого последнего момента: формы фигуры короля. Можно представить себе,
что он изучил правила игры, но ему никогда не показывали реальной игровой фигуры. В
этом случае форма шахматной фигуры соответствует звучанию или визуальному образу
некоторого слова.
Можно также представить себе, что кто-то освоил игру, не изучая или не формулируя ее
правил. Он мог бы, например путем наблюдения, усвоить сначала совсем простые игры на
досках и продвигаться ко все более сложным. Ему можно было бы дать пояснение "Это
король", показывая, например, шахматную фигуру непривычной для него формы. И
опять-таки это объяснение учит его пользоваться данной фигурой лишь потому, что
предназначенное ей место, можно сказать, уже подготовлено. Иначе говоря: мы только
тогда скажем, что объяснение обучает его применению, когда почва для этого уже
подготовлена. И в данном случае подготовленность состоит не в том, что человек,
которому мы даем пояснение, уже знает правила игры, а в том, что он уже овладел игрой в
другом смысле.
Рассмотрим еще и такой случай. Я поясняю кому-нибудь шахматную игру и начинаю с
того, что, показывая фигуру, говорю: "Это король. Он может ходить вот так и так и т.д." В
этом случае мы скажем: слова "Это король" (или "Это называется королем") лишь тогда
будут дефиницией слова, когда обучаемый уже "знает, что такое фигура в игре". То есть
когда он уже играл в другие игры или же "с пониманием" следил за играми других и тому
подобное. И лишь в этом случае при обучении игре может быть уместен его вопрос: "Как
это называется?" именно эта фигура в игре.
Можно сказать: о названии осмысленно спрашивает лишь тот, кто уже так или иначе
знает, как к нему подступиться.
Можно даже представить себе, что человек, которого спрашивают, отвечает: "Установи
название сам" и тогда спрашивающий должен был бы до всего дойти сам.
</QUOTE>

>Но даже если можем как-то доказать, что не можем изменить последовательность, это не показывает, что число Пи изменить нельзя, потому что мы, возможно, можем изменить интерпретацию, так чтобы это последовательность в число Пи не складывалась.
>Но показывает, что есть что-то абсолютное, что даже всемогущий бог изменить не в состоянии.

Уверен что на это есть разные аналитико-философские точки зрения но в целом, мне кажется математика на уровне металогики не работает, она принимает эту металогику как данность и используя её уже может строить какие-то конструкции и рассуждения (в том числе и другие логики, но они не принципиально другие, так как металогика-то у них одна). Продолжая свою предыдущую мысль я бы сказал, что мы не можем представить принципиально другую металогику, не будучи к ней подключенной, а подключиться к ней не можем, так как металогика у нас одна.

На самом деле у меня есть много ещё чего написать касательно твоего текста, так как тревожили подобные вещи меня довольно долго, но пока хватит, думаю.

>> No.4870694  
Файл: 149022_original.jpg -(47 KB, 991x610, 149022_original.jpg)
47

>>4870591

> Ну, вроде этого. Под синхронией я понимаю, в частности, не задавать вопросы "почему" и не считать ответом на них какую-то более глубокую и остроумную теорию, которая в своих терминах бы вскрывала и описывала некоторый тайный внутренний механизм происходящего, а скорее просто нечто подмечать и обозначивать, что так оно и происходит (даже если так оно не происходит).

(Это мнение как-то вдохновляется и математикой тоже? Замечать свойства, выводить следствия. Или как Теория Категорий/группы/другое — выделять замеченный частовстречающийся объект?)

Блин, в этом же как раз была идея моего треда (родившего эти чёртовы "овалы")! (Чувствую себя совсем дураком.) Последняя формулировка, до которой дошёл: надо обращать внимание не на объект "X" (являющийся чем-то или делающий что-то), а на "смежные" объекты на которых есть "отпечатки" X. Не искать дырку от бублика, а искать стенки бублика ("тесто", в котором можно видеть "отпечатки" этих дырок). Поэтому абстрагируюсь от низкоуровневых алгоритмов, которыми ИИ-сты занимаются сейчас (это X: то неизвестное, что делает мышление) и постулирую некие субстанции, на которых можно видеть промежуточные эффекты работы мышления (выделение этих "овалов", их соединение и разъединение: что этим занимается я не знаю). Поэтому утверждаю — паттерны сами по себе не имеют никакого значения, любой паттерн имеет значение только в контексте другого (по идее сети и должны так работать, но при этом может быть что на практике получается ровно наоборот поэтому и желал/желаю поговорить с кем-то знающим). Сейчас вот это вылилось в конкретную идею некоего сравнения двух смежных областей (т.к. то что можно задетектить (хоть каким угодно алгоритмом) в одной отдельной области не имеет никакого значения вообще то есть и опасение стало более конкретным — приняв эту идею можно примерно представить, каким образом можно зафейлить любой сколь угодно хороший алгоритм/"аппроксиматор" что бы он ни делал: примерно как умный подслеповатый человек мог бы обойти зрячего... хотя "сети" вроде и про это тоже). Вдруг современные сети относятся к "настоящему решению" примерно так же как целая сеть к отдельному нейрону? Немного думал о применении этого и к языковым концептам — можно пытаться описать сам концепт, как некий объект или алгоритм (типа, например, "сидеть" это некий алгоритм принятия сидячего положения или особый изгиб тела: или, абстрактнее, какая-то формула которая более-менее строго выбирает сидящие объекты из всех/отличает от не сидящих — и угодить в страшную войну с исключениями и "монстрами") и придти к выводу что с человеческим языком всё совсем мутно, а можно просто перечислить побочные объекты на которых могут быть "отпечатки" определяемого (например, список мест, на которых обычно сидят). Тупо перечислить всё и забить на то на чём обычно фокусируются. И оперировать этими "списками" будет в тысячу раз проще, и сравнивать их и прочее. То есть именно что не стоит пытаться выдать более странную для людей "формулу", которая будет "определять сидение лучше", а оставаться в пределе привычных простых понятий (т.к. их усложнение просто не несёт никакой перспективы, как считал и Лакатос по теме математики например "Доказательства и опровержения"). То есть существует вроде бы важная двойственность — сеть может работать как большая сложная формула, а может как повторение простых, и то что она на низком уровне устроена именно как второе не говорит что это её свойство будет последовательно сохранено

И это применимо и к идее Universal Grammar — считаю её не существует (как и никаких других "правил"/"формул"), однако существует нечто очень похожее/то что можно легко с этим спутать (см. пункт про "количественно-качественный дуализм"). И к идее того, что в мозгу есть упрощённая "физическая модель" — по-моему такой нет, а есть что-то типа библиотеки естественных для предметов движений (подобно языку)
Либо же это существует, однако то чем это "обрастает" в миллионы раз важнее

То есть в таком смысле я не считаю, что теория должна реконструировать мышление, и не реконструирую его (по крайней мере теперь я тебя понял). И даже активно отказываюсь от желаемых некоторыми реконструкций (UG, физмодели, Байесовские сети). И в таком смысле я не считаю, что нам чего-то не дано — пожалуйста, будь любой домохозяйкой, и просто делай наблюдения за самим собой

Я же сам классифицирую — идея основана на моём опыте и местами граничит с труизмами (тебе только недавно было вообще сложно отличить её от того что уже сделано). Вдруг у нас просто разный опыт и то что кажется тебе arbitrary придуманными "тайными механизмами" для меня самые скромные обобщения банальностей? Хотя бы что-то реконструировать же всё-таки надо (и в каких-то местах да задаться вопросом "почему/как?" поэтому я и хотел уточнить (точнее "расширить", т.к. уточнения бесперспективны) этот принцип: где-то даже "как" спрашивать может быть не верно, а где-то надо наоборот сыпать обоими вопросами — все эти принципы фундаментально не могут быть последовательны на 100 процентов и с этим надо как-то жить (имхо))

Ты видишь мои идеи в самом худшем свете/обращаешь внимание на то что наиболее всего напоминает нечто плохое? По-моему просто обратив внимание на другие моменты ты мог бы по тем же принципам сделать совершенно противоположный вывод

> А есть двойственный к этому коиндуктивный тип "бесконечная бинарная строка" у которого есть деконструктор "взять голову" и "взять хвост", и таким образом ты можешь деконструировать твою бесконечную бинарную строку "011010..."->("0","11010...")->("0",("1","1010...")) но при этом так как бесконечная бинарная строка бесконечна, то мы не обязаны упереться в пустую строку при деконструкции, так и у витгенштейна, вовсе не обязательно было вводить какие-то "элементарные факты"!

По-моему вот этот метод просто лучше и просто параллелен моей идее. Не важно, как и откуда мы взяли то, с чем оперируем. Или даже вообще вся эта философия скоплений как раз основана на "обрубании" лишних кусков информации — нашли первые "скопления", "обрезали" немного — нашли скопления скоплений... и т.д. (да, даже точно оно так!)

Идея описывает процесс, не обязанный иметь начала или даже конца

> Коиндукция и корекурсия, ровно как индукция и рекурсия особо ничем между собой не отличаются, обычно говорят, что (ко)индукция это типа метод доказательства или вывода чего-то, а (ко)рекурсия это логическая фигура самоприменимости as it is, но я не вижу смысла в такого рода различии.

Ну, ты упоминанием этого различия хотя бы вообще связал для меня эти вещи, спасибо.

> Думаю важно, но не потому что другой что-то там говорит, это как раз не имеет значения, а потому что сам язык устроен так, что обращаться можно только к другому. Скажем, я помню что каждый учитель выходил в моей школе и все начинали шуметь, то шумел я не потому что никто меня не видел и поэтому я мог делать нечто безнаказанное, а как раз шумел на глазах отсутствующего места учителя, в некотором роде я считал это чуть ли не жестом флирта с учителем.

А это разве не подтверждает мою идею, если только не трактовать её в каком-то строго причинно-следственном виде: "как только никто не видит я мгновенно лишаюсь всех оков морали" и "я учитываю только слова других" и "делаю то о чём бы хорошо подумал друг только прямо у него на глазах" (но просто "мимолётная" мысль, её принимать требуется ещё меньше чем ту ИИ-парадигму всё равно)? То что ты сам черпаешь важность из языка интересно

Просто вдруг и другие идеи по этой же причине "не поняты"?

И ещё потому что это косвенно связывается с "почему"-идеологией — "обёртка" поступков в виде мыслей других людей (или "облака" ассоциаций со схожими ситуациями) может быть важнее самих поступков. "Важнее объекта то чем он обрастает" (но зрение можно либо слишком близко приблизить, либо слишком далеко отдалить)

Просто потенциал глупости ситуации очень большой: вот у нас есть теория, которая может и основана на заметках, а мы машем руками что не основана и дропаем. Я же показал, как идея собиралась и эволюционировала из других идей. Те идеи в частности можно разбить на другие и на искомые "заметки" (или это называется порочным (раз)углублением?) — я-то сам бы в любом возрасте и без любых идей проголосовал бы за коннективизм и против редукционистов

https://ru.wikipedia.org/wiki/Лакан,_Жак#Воображаемое_—_Символическое_—_Реальное
(что нашёл по Лакану)

По-моему Лакан должен был устареть к нашему времени из-за новой информации, как и его понятия и даже вся его область. Даже иначе стоит сказать... что ты, что Лакан, что я, что кто угодно ещё, это совершенно the same sh-t, который занимается абсолютно одинаковым делом. Просто у нас разные любимые области, вот мы и сконцентрировались на разных заметках и постулировали разные субстанции. Тот же самый подход что у любого человека (или "Коннекционизма") — сделали заметки, немного чего-то постулировали, чтобы нагенерить (увязать) ещё больше заметок и ещё больше напостулировать (полные штаны!)

Вот от этого мне особенно больно в районе пятой точки (эти потоки мнений уже видятся просто неразличимым потоком семени в лицо, и никто не хочет ломать этот порочный круг спорящих друг с другом клонов, выдумывающих различия там где их нет) (на Реддите в последний раз у меня вообще с этого стул улетел, настолько стало бо-бо но, конечно, "смиряюсь" и на тебя не бычу)

Просто я где-то год тратил на то, чтобы вывести "Принцип(ы) истины", охватывающий мои вкусы и интересы, сделаться последовательным...
Какого же было моё удивление, когда я узнал что каждый человек (тем более "нажравшийся" опыта жизни или диалогов) ведёт себя точно так же как я...
Только вот никакого сверх-принципа там нет и не планируется (просто фанатизм от конкретных концепций или необкатанных и непроверенных квази-недопринципов), гарантии последовательности или даже банального подобия логики 0 (прости ещё раз, просто так наболело) — я просто хочу закончить это лицемерие, пусть даже это бы потребовало пожертвовать свобственной правотой (я оправдываюсь тем что молод, но вы-то всю жизнь варитесь в этом, зачем, почему, как?)

>> No.4870697  

>>4869945

> >аксиомы для натуральных чисел отлично описываются без каких-либо понятий из теории категорий. Аксиоматика Пеано формулируется в логике первого порядка (ЛПП)
> Только как известно по теореме о неполноте эти аксиомы описывают не только натуральные числа, но и кучу нестандартных моделей в придачу (по крайней мере если использовать стандартную первопорядковую семантику). Аксиомы Пеано они скорее нужны чтобы формализовать понятие доказуемого утверждения о натуральных числах, чем сами натуральные числа.

Ты точно ничего не путаешь? Вопрос полноты системы аксиом ортогонален вопросу о её категоричности, то есть об изоморфности всех её моделей. Последнее свойство доказано для аксиоматики Пеано из 4 аксиом; вот если выкинуть аксиому индукции, то уже могут быть нестандартные модели.
>>4870650

> цифру числа Пи (пусть она показывает только первые сколько-то цифр, а затем заново, чтобы избежать необходимости в бесконечной памяти)

Если машина считает π как сумму некоторого бесконечного ряда, то может быть известна верхняя оценка на модуль суммы оставшегося хвоста, позволяющая забывать о не меняющихся первых цифрах суммы и избежать бесконечного увеличения потребности в памяти по мере вычисления всё более дальней цифры представления числа π.

> Так вот вопрос в том, можем ли мы или бог придумать такой мир, в котором NOR-гейт работает точно также (даже если устроен по-другому), гейты соединены точно так же, но последовательность она выдаёт другую?
> Я верю, что не можем, но доказать я этого не могу.

Если машина может вычислять только до некоторой n-ой цифры числа π, то по самому построению две логически изоморфные машины M1 и M2 на каждом шаге от 1 до n будут переходить в состояния m1 и m2, переводимые этим изоморфизмом друг в друга. Это достаточно очевидно? Если же машины могут вычислять какую угодно большую по порядковому номеру цифру π, то для доказательства того, что последовательность одна и та же, потребуется принцип математической индукции, то есть это доказательство будет неявно опираться на непротиворечивость аксиоматики Пеано для арифметики. О непротиворечивости арифметики же ничего не доказано (можно ли в принципе доказать, что в ней не может быть противоречий?).
Кстати, поздравляю всех с Днём числа π!

>> No.4870702  

>>4870663
На самом деле ЛФТ Витгенштейна это единственная книга по курсу философии, помимо Платона, которую я пытался прочитать. Но не уверен, что что-то оттуда понял.

Но мне показалось, что там он пытается провести процедуру бутстрапа, как, например, когда нужно написать компилятор, написанный на том языке, который он компилирует, для этого самого метаязыка. То есть он вводит сначала понятия, которые основаны на интуиции, а затем переформулирует их так, чтобы они опирались друг на друга, а не на интуицию, таким образом отвязывая их от неё. Но может быть мне только так показалось.

>> No.4870705  

>>4870697

> Если машина считает π как сумму некоторого бесконечного ряда, то может быть известна верхняя оценка на модуль суммы оставшегося хвоста, позволяющая забывать о не меняющихся первых цифрах суммы и избежать бесконечного увеличения потребности в памяти по мере вычисления всё более дальней цифры представления числа π.

Там точно константа, от длины номера цифры, которая логарифм, не будет сложность вычисления зависеть? Если так, то хорошо, конечно. В реальной жизни логарифмом обычно можно пренебречь, но тут теория.

>> No.4870707  
Файл: kuri239.jpg -(21 KB, 236x327, kuri239.jpg)
21

>>4870697
Слишком тред активным стал, нет времени сейчас всё читать напишу только по факту.

>Ты точно ничего не путаешь? Вопрос полноты системы аксиом ортогонален вопросу о её категоричности, то есть об изоморфности всех её моделей. Последнее свойство доказано для аксиоматики Пеано из 4 аксиом; вот если выкинуть аксиому индукции, то уже могут быть нестандартные модели.

Ты всё напутал! Из категоричности в любом бесконечном кардинале + отсутствие конечных моделей теории первого порядка в первопорядковой семантике следует полнота - это Los-Vaught test называется.
Про категоричность 5 аксиом ты имел в виду скорее формулировку аксиом Пеано на L2 языке и его интерпретацию в full second order semantics, что называют "арифметикой второго порядка Z_2", но тогда мы теряем полноту underlying логики, у меня с этим проблем нет, но некоторые люди из-за этого эту самую "логику второго порядка" логикой не считают вообще.

>> No.4870720  

>>4870707

Я тут ещё вспомнил то, что может быть важно:

  1. Эти "сверх-абстрактные свойства" по сути и есть "хвост" (хотя я не знаю, что это) от строки, только не из битов, а из "обычных" свойств. Это был один из старых способов описать идею ("описание это бесконечная череда синонимов, которые чуть-чуть корректируют друг друга")
  2. Мои "тайные механизмы" оправдываются ещё тем (возможно), что позволяют обобщить конкретные операции с казалось бы совершенно разными вещами (ну как может быть связана аргументация и распознавание образов, например? А это позволяет как раз сделать заметки о том, чем связана кроме того что и то и то делает мозг)
>> No.4870721  

>>4870720 То есть знаю, т.к. ты это объяснял, сорри, мозговое затмение

>> No.4870741  
Файл: kuri240.jpeg -(7 KB, 190x266, kuri240.jpeg)
7

>>4870694

>(Это мнение как-то вдохновляется и математикой тоже? Замечать свойства, выводить следствия. Или как Теория Категорий/группы/другое — выделять замеченный частовстречающийся объект?)

Думаю да, я люблю повторять, что история начинается и заканчивается в пределах одной статьи.

>Ты видишь мои идеи в самом худшем свете/обращаешь внимание на то что наиболее всего напоминает нечто плохое? По-моему просто обратив внимание на другие моменты ты мог бы по тем же принципам сделать совершенно противоположный вывод

То что я не хочу полюбить твои идеи это правда, но мне кажется ты это интерпретируешь слишком ресентиментно, что мол я это делаю потому, что у тебя недостаточно символического капитала и мне мешает мой здоровой скептицизм, а вот будь ты каким-нибудь видным философом - этот барьер в миг бы исчез. А на самом деле мне просто кажется что наше желание немного в разных местах лежит. Как я и говорил, мне очень мало что интересно, обычно в таких случаях я стараюсь как-то критиковать чтобы показать свою любовь, а не хвалить и соглашаться, потому что сам любую даже самую глупую критику в духе "да это всё билеберда какая-то" в адрес каких-то своих слов воспринимаю скорее как признание в любви и выражение интереса, а вот к похвале и соглашательству всегда отношусь как-то плохо и настороженно (потому что, казалось бы, никто ни о чём не спорит, а мне в чём-то первенство отдают - кому такое понравится?) (Только не воспринимай это как просьбу замолчать, пожалуйста!)

>А это разве не подтверждает мою идею, если только не трактовать её в каком-то строго причинно-следственном виде: "как только никто не видит я мгновенно лишаюсь всех оков морали" и "я учитываю только слова других" и "делаю то о чём бы хорошо подумал друг только прямо у него на глазах" (но просто "мимолётная" мысль, её принимать требуется ещё меньше чем ту ИИ-парадигму всё равно)? То что ты сам черпаешь важность из языка интересно

Думаю я думаю нечто прямо противоположное, когда учитель в классе - я развожу никому из нас двоих ненужный университет и формальности, что конечно же аморально и зло, и только когда учителя нет я могу выразить ему свою любовь в виде флирта у него на глазах, что очень морально и любовь, при этом слова учителя не играют никакой роли, он мог не сказать ни одного слова, тем не менее занимать некоторую позицию по отношению к моему желанию. И уж тем более я не из тех моралистов которые думают, что естественным состоянием для человека является голая бесконечная оргия, и только тонкая плёнка культуры и морали удерживает их от того, чтобы ввергнуться в бесконечное грехопадение (опять же, если и мыслить в таких терминах - то образом прямо противоположным).

>Просто потенциал глупости ситуации очень большой: вот у нас есть теория, которая может и основана на заметках, а мы машем руками что не основана и дропаем. Я же показал, как идея собиралась и эволюционировала из других идей. Те идеи в частности можно разбить на другие и на искомые "заметки" (или это называется порочным (раз)углублением?) — я-то сам бы в любом возрасте и без любых идей проголосовал бы за коннективизм и против редукционистов

Мне кажется тут и выдаёт некоторая диахроничность твоего способа мышления в очень прямом смысле: зачем тебе постоянно держать в голове и прослеживать эволюцию своих идей? Что тебе даёт это измерение времени? У меня чувства скорее обратные: что меня никто не понимает и даже не понимает в каком месте меня не понимает.

>По-моему Лакан должен был устареть к нашему времени из-за новой информации, как и его понятия и даже вся его область. Даже иначе стоит сказать... что ты, что Лакан, что я, что кто угодно ещё, это совершенно the same sh-t, который занимается абсолютно одинаковым делом. Просто у нас разные любимые области, вот мы и сконцентрировались на разных заметках и постулировали разные субстанции. Тот же самый подход что у любого человека (или "Коннекционизма") — сделали заметки, немного чего-то постулировали, чтобы нагенерить (увязать) ещё больше заметок и ещё больше напостулировать (полные штаны!)
>Вот от этого мне особенно больно в районе пятой точки (эти потоки мнений уже видятся просто неразличимым потоком семени в лицо, и никто не хочет ломать этот порочный круг спорящих друг с другом клонов, выдумывающих различия там где их нет) (на Реддите в последний раз у меня вообще с этого стул улетел, настолько стало бо-бо но, конечно, "смиряюсь" и на тебя не бычу)
>Просто я где-то год тратил на то, чтобы вывести "Принцип(ы) истины", охватывающий мои вкусы и интересы, сделаться последовательным...
>Какого же было моё удивление, когда я узнал что каждый человек (тем более "нажравшийся" опыта жизни или диалогов) ведёт себя точно так же как я...
>Только вот никакого сверх-принципа там нет и не планируется (просто фанатизм от конкретных концепций или необкатанных и непроверенных квази-недопринципов), гарантии последовательности или даже банального подобия логики 0 (прости ещё раз, просто так наболело) — я просто хочу закончить это лицемерие, пусть даже это бы потребовало пожертвовать свобственной правотой (я оправдываюсь тем что молод, но вы-то всю жизнь варитесь в этом, зачем, почему, как?)

То что Лакан устарел выставляет его ещё в более выгодном свете для меня, так как, во-первых, в последнее время любая новизна вызывает у меня только раздражение, а во-вторых чем меньше внимания к нему, тем меньше его мероприятие будут замыливать и тем легче мне будет оставаться его мероприятию верным. Я не очень понимаю зачем мне подстраивать свой способ мышления под какие-то банальные подобия логики и последовательности, этого мне и в математике хватает и это зачастую не лучшая её часть. Я вот сломал этот круг тем, что ни с кем не спорю например (если они сами меня об этом не просят конечно, тогда я им могу чуть-чуть подыграть!) И я не такой уж и старый!

>>4870697

>Ты точно ничего не путаешь? Вопрос полноты системы аксиом ортогонален вопросу о её категоричности, то есть об изоморфности всех её моделей. Последнее свойство доказано для аксиоматики Пеано из 4 аксиом; вот если выкинуть аксиому индукции, то уже могут быть нестандартные модели.

В дополнение к посту выше: на самом деле даже лос-ваугт теста не нужно, достаточно обычной теоремы Гёделя о полноте чтобы понять что что-то не чисто: если модель одна, то любое утверждение либо верно в ней либо верно его отрицание, то есть категоричная теория первого порядка всегда полна.

>> No.4870755  
Файл: Schrödingers_plates.jpg -(49 KB, 600x479, Schrödingers_plates.jpg)
49

>>4870741

> То что я не хочу полюбить твои идеи это правда, но мне кажется ты это интерпретируешь слишком ресентиментно, что мол я это делаю потому, что у тебя недостаточно символического капитала и мне мешает мой здоровой скептицизм, а вот будь ты каким-нибудь видным философом - этот барьер в миг бы исчез. А на самом деле мне просто кажется что наше желание немного в разных местах лежит.

Я просто думаю что ты непоследователен. Что можно было взять другие места из того поста и сделать обратный вывод. Ну или попробовать чуть расписать свой критерий/поподробнее сравнить с чем-то (сделать "Дубль два") и увидеть, что обвинение просто было основано ни на чём

Есть вариант и "временно захотеть" полюбить чужие идеи (или даже не чужие, а собственные что приходят в голову от текста)

А ты уже начал Фрейдистское "гадание по мамкам": свои мотивы очищаешь ("просто другой интерес"), а чужие загрязняешь (ресентимент). Конечно я понимаю, что у тебя "немного другой интерес", и даже оцениваю насколько и где он другой... я же персонажей по лицам классифицировал и философию расписывал, в которой всё похоже и не похоже, мне не привыкать к гомологичности и "уникальным пирожкам" (опять же, можно было обратить внимание на это; можно было и просто прочитать второе предложение из цитаты)

Как я могу не думать мысль, которую сам продвигал ещё до прошлого поста? "Ямы эгоизма" (когда каждый воспринимает абсолютно всё под каким-то похожим, но собственным углом, и это ведёт к бо-бо последствиям, т.е. полному непониманию и невозможности выбраться) в посте до монад Лейбница

> Думаю я думаю нечто прямо противоположное, когда учитель в классе - я развожу никому из нас двоих ненужный университет и формальности, что конечно же аморально и зло, и только когда учителя нет я могу выразить ему свою любовь в виде флирта у него на глазах, что очень морально и любовь, при этом слова учителя не играют никакой роли, он мог не сказать ни одного слова, тем не менее занимать некоторую позицию по отношению к моему желанию. И уж тем более я не из тех моралистов которые думают, что естественным состоянием для человека является голая бесконечная оргия, и только тонкая плёнка культуры и морали удерживает их от того, чтобы ввергнуться в бесконечное грехопадение (опять же, если и мыслить в таких терминах - то образом прямо противоположным).

Я в кавычках пародировал твоё представление моих слов. Да, не всё можно выразить "на глазах", согласен. Но эта идея ортогональна причинно-следственным связям, которые ты пытаешься инвертировать/наложить на эту идею

Если ты считаешь мою идею сильно связанной с идеей "ограничивающих плёнок", то ты можешь опять допускать ошибку интерпретации (добавляя свои причинно-следственные связи вместо заявленных принципов и контекстуальных связей). Согласно моей философии не важны объекты, а важны их "края", то есть "плёнка" это не плёнка а самая важная вещь. Важны не первичные мотивы ("животные инстинкты оргии"/Фрейдизм и любовь к маме/эгоизм), а то чем они обрастают и по каким "путям" идут

По-моему если ты не спрашиваешь нужна ли твоя любовь училке, подразумевая абсолютное добро без чужих мнений, это плохо/эгоистичная любовь

> Мне кажется тут и выдаёт некоторая диахроничность твоего способа мышления в очень прямом смысле: зачем тебе постоянно держать в голове и прослеживать эволюцию своих идей? Что тебе даёт это измерение времени? У меня чувства скорее обратные: что меня никто не понимает и даже не понимает в каком месте меня не понимает.

Я сам забываю их, но эффект от них остаётся. Как от тяжестей, которые смог поднять (а сейчас это просто пушинки или вещи оригинальностью уровня "трава зелёная"). Но как я смогу передать эффект другим? Как я опрусь на факты, которые для других не факты?

(В последний раз это вспоминал вообще исключительно для тебя, т.к. тебе конечные продукты теории показались оторваны от поверхностного опыта, as it is данного нам и т.д.)

По моей концепции твоё чувство совершенно естественное. Даже Элезиер Юдковский с этой мысли пригорал

Но и есть более глубокие уровни выводов из уникальности людей: что другой человек даже "прочитать" твой текст не может, что другой человек не может быть последовательным для тебя и т.д. (что даже любя твою же область он сделает из неё другие выводы) по-моему группа Инкогнито в синглах "Воздух" и "Созвучны" касается темы (думаем лишь о себе/словно Везувий глотаем свою пыль)

> Я не очень понимаю зачем мне подстраивать свой способ мышления под какие-то банальные подобия логики и последовательности, этого мне и в математике хватает и это зачастую не лучшая её часть. Я вот сломал этот круг тем, что ни с кем не спорю например (если они сами меня об этом не просят конечно, тогда я им могу чуть-чуть подыграть!) И я не такой уж и старый!

Это типичная ошибка, по-моему. Мораль, догмы, идеологии (их логики) — это не только "качественные" штуки, это и "количества" (количества информации). Ты пытаешься игнорировать информацию и выходишь из данжена, дающего опыт

Ещё у тебя переизбыток "нормальных" мнений (как это, из цитаты) — мнений типа "никто ничего никому не должен". Чтобы сделать хоть какой-то вывод, где-то нормальность надо сломать. Или сломать её для кого-то

(Однако, для нормальности тебя могло сильно перекосить в обратную сторону "отнекивания". "Дропаем память, дропаем логику, проявляем любовь самым странным способом")

Занятие математикой, например, это специфическое занятие. Можно послать это занятие или предъявить ему требования из более общей области

Идея "мнения это желания" это то чем занимался для вывода "Истины". Ещё занимался идеей отрицания причинно-следственных связей и возможности последовательной неформальной логики. Но без знания этой истории тебе легко может быть принять меня за "плёночника-морального-нигилиста" или принять слова слишком буквально

Про ресентимент ещё — по-моему любая несправедливость основанна на принижении чего-то чужого и при повышении чего-то своего. Поэтому может и разгораться в том или ином виде нехилый ресентимент с желанием напомнить индивиду разницу между его реальным положением и его самомнением

По-моему ты порой только и ждёшь, чтобы выхватить какую-то вещь и не согласиться. А порой просто указываешь пальцем на что-то, называешь какое-то слово и это почему-то должно быть плохо (по-моему когда последний раз упоминалось слово "диахроничность" это было совсем плохо). Но качеств, которыми оперируешь ты, можно придумать бесконечно, доказывая так что угодно, и это я сейчас назвал лишь самую общую проблему...

>> No.4870769  

>>4870741

Ну или вот ещё моя цитата:
Я сам смирился с фактом, что в чужом мозге твои идеи не могут иметь такого же значения — у чужого нет твоего "топлива", которое помечает твои (даже совсем слабо выраженные) идеи как важные (либо у чужого мозга есть то топливо, которое зажигает вещи поважнее твоих идей)

Зачем выдумывать про меня "ресентимент", когда всё уже написано? Я понимаю, что у тебя другие интересы и другой контекст в голове (из цитаты)

Ты хочешь остро не соглашаться (выражать так любовь), но для этого нужны действительно проработанные и широкие принципы. А ты (как и другие) хочешь сделать то же самое "без труда"/"без ничего"
И ничего не остаётся как оценивать лишь плохое (а где оценка и хорошего тоже?), подразумевать наихудший вариант, использовать недо-принципы неконсистентным способом без опоры на сравнение

Вижу проблему не в скептицизме, а в "фарисействе". Каждый крутит законы как хочет (придуманные при этом лично самим собой и не-единственно возможные) и забрасывает этим других как будто за ним стоит идеология, только вот сама идеология упускается... Как это вообще работает и как вы из этого выводите именно собственную правоту? Как создаёте дисбаланс в собственную пользу?

По-моему тут действительно бы не помешало понимание, что кто-то уже прошёл этот путь и зашёл по нему намного дальше тебя (хотя бы что такой вариант может быть)

Тебе просто представить что мы одинаковы помогло бы понять многое что ты не понял

>> No.4870821  
Файл: kuri241.jpg -(146 KB, 706x990, kuri241.jpg)
146

>>4870755
>>4870769
А я могу думать о чём-то, при этом не претендуя на необычность и новизну мнения, не защищя какие-то свои принципы и идеологию, не считаясь с объективной истиной и не выводя свою правоту? Где-то в речи вообще существует место, где можно не претендовать на новизну мысли и правоту или любая моя даже самая невинная фраза должна трактоваться как выстрел в баталии сторонников каких-то идеологий (которые, к тому же ещё, говорят об одном и том же)? Если есть, то скажи, пропишу свою речь именно по этому месту.

Я пытаюсь представить что мы одинаковые из-за всех сил и твоя логика и построения мне местами сильно близки (вот оценка хорошего тоже), но чтобы хоть как-то поддерживать разговор и выражать свою любовь несогласием, я постарался указать на то место, где наше желание различно, если на различия вообще указывать: ты претендуешь на новизну, последовательность и логичность, а я ни на что не претендую (даже на то, чтобы ни на что не претендовать не претендую, хотя тебе и могло так показаться).

>> No.4870903  

>>4870821

Прости, я "деградировал" диалог. На >>4870558 готов был сказать "ну ладно", а когда узнал что такое диахроничность (значение, напомнившее собственный принцип) тут и "затянуло" (постепенно, поэтапно). Последний пост совсем про лишнее/побочное

Ещё в довесок ко всему не понял и какой "сигнал" послан: не замолчать, проявлять любовь несогласием, но не (?) спорить

Лакан (если вообще того нашёл) судя по ссылке сам осмыслял этапы своих идей ("Эту Схему трёх регистров Лакан применял к самому себе") и в его идеях вроде нет альтернативной модели... то есть "того что напоминает хорошие стороны моделей" (т.к. "модели" это не очень хорошо). (Единственное что по диалогу могу придумать, не оспаривающее что-то прямо)

Хотя теперь я понял и чем практичность не нравится ("кажется" противоречащей тем же заметкам). Но это же уже спор не с конкретной идеей (?), а со всеми сразу (практичны все в этой области)... и не презумпция ли типичности это? Ты не ищешь отличий, только схожести
https://lesswrong.ru/w/Презумпция_типичности_—_худший_аргумент_в_мире
(Вот ты слышишь "старое + новое" и автоматически говоришь "диахроничность и эволюция". А вдруг старые идеи это просто составляющие новых или просто стали чем-то другим, типа классификации того кто на каком их этапе может быть?)

Ситуация для меня-то имхо так и остаётся безнадёжной. Люди либо будут говорить, что "это уже было" (сначала), либо доставать собственную философию (даже не по теме ИИ-разработок! просто любой их принцип или вкус) и противопоставлять её моей (без обязательств обоснования/сопоставимости причём)... а как общение и обмен идеями работает в такой системе? По-моему можно было нечто сделать кроме обоснования истины или предложения чего-то ещё более нового

>> No.4870954  

Ну вот, скатили математика-тред к "фарисейству", фройдистским "гаданием по мамкам" и прочим спецолимпиадам.
В каждом треде выясняете, чьё кунгфу круче. Поэтому с вами никто и играть не хочет.

>> No.4870956  

>>4870954
Но это не ОП, а шизик. Ему своего треда мало.

>> No.4871040  
Файл: Sigmund_Freud,_by_Max_Halberstadt_(c(...).jpg -(1237 KB, 1647x2240, Sigmund_Freud,_by_Max_Halberstadt_(c(...).jpg)
1237

>>4870821

Вот по-моему хороший пример для объяснения, что может происходить:
Я говорю, что чтобы задать вопрос "почему" (или другой) на уровне N2 надо накопить поверхностных заметок на N1. Или показываю, как сам избавляюсь от мышления о механизмах

Из этого можно заметить

  • Что я ЗА поверхностные заметки и ПРОТИВ механизмов
  • Можно услышать что-то про "уровни", а это эволюция и история, а это плохо. (но вроде прошлый пункт перевешивает этот? Два или три "за" против одного "против")

Ты действительно так читаешь мой текст?

По-моему честно сравнить количество за и против по своему принципу. Или задуматься о своём принципе, коли возможно одновременное выполнение и не-выполнение этого принципа. Он, очевидно, существует на разных уровнях (и эти уровни могут противоречить друг другу)! Может, происходит некое быстрое пересаживание со стула на стул, жонглирование разными формулировками и идеями?

Можно ещё попытаться вывести следствия из этого принципа, которые могут вступить с твоей его версией в противоречие. Например:

"Поверхностность" (основа на заметках) не может быть константной. Она зависит от времени и конкретного опыта. Когда-то идея нейронной сети не была поверхностной (разные определения "поверхностности" могут быть, кстати). Поэтому кто-то может решить, что всё устаревает, и некий исторический взгляд таки нужен

То есть, по-моему, из твоего принципа (принципов) следует что угодно (и может ты не очень хорошо проверяешь, действительно ли соответствует то на что ты указываешь чему-то плохому по принципу: а ведь могут быть запутанные "сложные случаи")

> Думаю да, я люблю повторять, что история начинается и заканчивается в пределах одной статьи.

Но ведь любую идею можно абстрагировать от исторического опыта, на котором она основана! Просто презентовать иначе, без упоминания истории. "История/не история" по-моему оффтоп относительно утверждающих что-то идей (где в самих утверждениях теории какая-то "история"?) и так отличать одни идеи от других просто невозможно. Я же говорю, могу разбить идею на просто заметки (как ТК можно разбить на что она абстрагирует и на исходные заметки — но а можно и не разбивать вообще! Это просто фантик же, способ презентации)... Может исторические примеры это просто примеры того, на чём основаны принципы? Ведь и твои должны были быть вдохновлены чьими-то удачами или неудачами

Тут по-моему какая-то очень плохая вещь прокралась, тонкость, но сформулировать её трудно. Ты иногда вообще абстрагируешь от того, что плохого я делаю

Например, обвиняешь меня, что я держу что-то в голове. Но это надо же как-то переформулировать, иначе получается полный ad hominem какой-то (где проверка, что моё поведение и мои слова влияют на собственно представленную идею? Что это, опять же, не "оффтопик"? Что не произошла подмена формулировок?)
https://ru.wikipedia.org/wiki/Ad_hominem (хотя я вообще не строг к A.H. и тому что там названо в преамбуле)

Твою идею про "не думать о механизме" же можно применить и в споре — важны не убеждения, а действия. К каким плохим действиям (пусть даже "виртуальным") ведут мои убеждения? Если мы абсолютно забудем этот вопрос, то будет mindlessly карать всех без разбора кто просто заикнулся об истории Это же Кафка какой-то

>>4870954, >>4870956 Вы понимаете, что таким же способом (и с таким же уровнем разбирательства) слагаются легенды и о вашем ресурсе, вашем доме? Недавно же только картинку в треде о рисовании постили, каким образом так происходит гниение (мне ещё нравится, что один из вас поправляет другого, однако результат от глухого телефона не отличается)

А вообще просто странно, как с Реддитом: пошла какая-то волна агрессии, от которой уже нигде не скрыться. Если не заглядывать на свой Реддит, то тут тред про "Наше поколение не имеет смысла" (толстый котик), или вот эта жесть в треде рисования, и здесь уже поговорить невозможно... под новой записью Аввы (!!) самый настоящий махровый сексизм

Всё зависит лишь от вас и ваших (не)постов, хватит искать виноватых и ненавидеть

"Ему мало своего" это "ненавижу потому что существуешь"

>> No.4871047  

>>4871040
С тобой действительно не хотят играть. И не захотят играть до тех пор, пока ты что-то в себе не изменишь. Наверное, это не зависит от площадки, на которую ты придёшь, пока ты не поменяешься, в любом месте и в любом "доме" так будет.

>> No.4871110  
Файл: kuri242.jpeg -(12 KB, 185x272, kuri242.jpeg)
12

>>4870954
>>4870956
>>4871047
Полно вам обзываться! В нашем замечательном треде мы не держим никакого уровня изначально, чтобы куда-то скатываться, а если хотите математики - так лучше бы вбросили математику, а я бы поддержал!
>>4870903
>>4871040
Главное не думать, что я хочу тебя какими-то аргументами в чём-то убедить, а всё остальное - позволительно и уж тем более не думать, что я в чём-то тебя обвиняю, я просто указываю на места которые вызывают у меня подозрение или кажутся странными, и на самом деле не очень хочу слышать, что мои подозрения необоснованы (хотя ты можешь об этом говорить, если тебе так того хочется), потому что я его обосновывать ничем и не собираюсь, только в таком режиме я и умею разговаривать. Если ты будешь меньше беспокоится о том как я читаю твой текст, и как правильно следует мне его читать или тебе со мной общаться, а будешь просто общаться, я тебя стану уважать только больше! Например мне твоя тревога по-поводу того, происходит или не происходит обмен идей кажется странной - даже в голову бы не пришло такие вопросы задавать, кому какое дело происходит он или нет?

>> No.4871155  
Файл: 1509355834_img_7602.jpg -(724 KB, 1500x1500, 1509355834_img_7602.jpg)
724

>>4871047 Понятно. Просто думал что и не обращать внимание на чужие треды эгоистично

>>4871110

> Главное не думать, что я хочу тебя какими-то аргументами в чём-то убедить, а всё остальное - позволительно и уж тем более не думать, что я в чём-то тебя обвиняю, я просто указываю на места которые вызывают у меня подозрение или кажутся странными, и на самом деле не очень хочу слышать, что мои подозрения необоснованы (хотя ты можешь об этом говорить, если тебе так того хочется), потому что я его обосновывать ничем и не собираюсь, только в таком режиме я и умею разговаривать.

Но наш диалог вроде ещё и останавливатся на этом. То есть когда "пометили" какое-то место в тексте оно для диалога перестаёт существовать (углубление не идёт). Не станет общение обменом полуслучайными триггерами или просто налюдением "мозговых поллюций" друг друга?

Идею далее вроде невозможно обсуждать без оспаривания увиденных тобой её свойств. Аргументацию тоже (я обычно по идеям и аргументации)

> Например мне твоя тревога по-поводу того, происходит или не происходит обмен идей кажется странной - даже в голову бы не пришло такие вопросы задавать, кому какое дело происходит он или нет?

У меня в общем случае всё-таки стоит цель передать свои идеи, поэтому мне важно. Тут же тоже зависит от опыта/контекста — в определённом контексте "Обмен идеями есть или нет?" это просто какая-то странная философия, а когда у тебя накапливаются заметки от общения с людьми о том как они спорят (и как сам споришь) и трактуют тезисы (чтобы их можно было оспорить; и как сам трактуешь) — это уже естественный (в контексте твоей жизни) вопрос, который сам всплыл на поверхность без усилий с твоей стороны (ты же сам вроде об этом думаешь: о непонимании где не понимаешь, я о том же может быть)

Как, действительно, тебе он мог придти в голову, если ты не носишься с идеями и оставил споры (перекрыв этот тип опыта)? Если в твоей области этот вопрос часто не имеет смысла (слихвой достаточно обмена результатами)? И всё равно, он пришёл на самом деле даже тебе — ты развил эмпатию представления желаний за мнениями, например (>>4870396)

Мне самому бы он не пришёл в голову, если бы

  • У меня в жизни не случилось "провисание". Я разрабатывал очередной "дар" людям... но тут вернулся к тем, кого оставил позади, и оказалось что каждый кулик в своём болоте (оказывается-то, вот удивление, центропупом считаю себя не только я). Я ещё замечал похожие темы у некоторых музыкантов, но воспринимал это как нечто хорошее (эгоистично считал что мир движется туда же, куда и я). А оказалось, что похожесть это не всегда хорошо (это может быть и жестокой издёвкой: как животные перед зеркалом в каком-то эксперименте)
  • Но даже этого ("депрессии") было мало. Но другие появившиеся заметки родили идеи, в которых этот вопрос естественнен
  • Было бы какое-то улучшение дел, можно было бы забыть об этом вопросе (перестал бы быть актуален).

Про это (и непонимание где не понимаешь может) даже есть песни:
https://www.youtube.com/watch?v=iuf_ojS_nCw
Мы на разных частотах кричим о своём
Мне говорить не о чем и не с кем, никто не учится моему языку
Расцарпанные пальцами стены

Это же просто по сути то же самое что "немного разные интересы" (только "немного разное" в довесок к этому вообще всё, чтение и логика например) Не хотел поднимать эгоистично тему понимания именно моих текстов: лучше чьих угодно

>> No.4871296  
Файл: kuri243.jpeg -(8 KB, 201x251, kuri243.jpeg)
8

>>4871155
Как ни странно, мне кажется твои идеи распостранялись бы эффективнее если бы ты так ревностно не заботился об их распостранении. Про три регистра лакана - мне они самому не очень нравятся, ну кроме "символического" - это действительно неплохое слово, часто его использую, когда не боюсь того, что люди подумают, что я умничаю.

>> No.4871346  
Файл: kuri245.jpeg -(11 KB, 199x253, kuri245.jpeg)
11
>> No.4871511  
Файл: kuri246.jpeg -(11 KB, 184x273, kuri246.jpeg)
11
>> No.4871525  
Файл: src.png -(16 KB, 859x385, src.png)
16

Решил почитать Гельфанд Шень "Алгебра". Вот например по описанию коммутативности:

>Проще всего равенство 5 · 3 = 3 · 5 объяснить на картинке: яблоки в прямоугольнике можно считать по рядам (три ряда из пяти яблок) или по колонкам (пять колонок по три яблока).

Я убежден, что это плохое объяснение. Почему мы должны быть уверены, что для любого прямоугольника из яблок это справедливо? Быть может, если мы выложим ряды из триллиард яблок в триллион колонок, то это будет не то же самое количество яблок, как если б мы выложим ряды из триллиона яблок в триллиард колонок? Есть ли книга, где все эти вещи доказываются более полно?

>> No.4871538  
Файл: 249733.jpg -(670 KB, 566x800, 249733.jpg)
670

>>4871525
Мне кажется хорошее, потому что учит думать как думают математики: проассоциировать семантику с синтаксисом - заметить какую-то симметрию в семантике - вывести из этого следствие для синтаксиса. Более формализованные есть во всяких учебниках мат.логики и теории типов, где процесс мат.доказательства моделируется в некоторой синтаксической машинерии: упомянутые библиотеки metamath, и кока, ещё более-менее на элементарном уровне формализованная арифметика Пеано объяснялась в "Элементарная математика" Иванов. Но только доказывая что-то в какой-то машинерии тебе нужно проверить в осмысленность этой машинерии, потому не понятно в чем тут будет выигрыш.

>> No.4871544  

>>4871525
Коммутативность умножения натуральных чисел выводится с использованием принципа математической индукции из определения умножения и пары теорем: об односторонней дистрибутивности к сложению и о коммутативности умножения на единицу. Эти теоремы тоже доказываются при помощи принципа математической индукции с использованием определения умножения и свойств операции сложения.

>> No.4871550  
Файл: kuri247.jpeg -(11 KB, 201x251, kuri247.jpeg)
11

>>4871544
Смотря как определять! Если определять в ZFC как мощность множества n x m (декартового произведения конечного множества из n элементов и конечного множества из m элементов), то доказательство ровно такое как в учебнике, заметить что f : n x m -> m x n, (a,b) \mapsto (b,a) биекция. Энивей не вижу смысла спускаться в такие формализации, если не занимаешься пруфчекерами, они не увеличивают убедительность.

>> No.4871570  
Файл: 14677537209652.jpg -(46 KB, 594x367, 14677537209652.jpg)
46

>>4871538

>Более формализованные есть во всяких учебниках мат.логики и теории типов, где процесс мат.доказательства моделируется в некоторой синтаксической машинерии

Тогда такой вопрос: что было раньше, курица или я как мы можем формально определить какие-либо синтаксисы и семантики, если мы еще ничего не знаем о том, каким образом определяются какие-либо синтаксисы и семантики?
Есть более глубокий вопрос, не знаю как этот вопрос сформулиловать вообще. Есть ли вообще какой-то фундамент познания математики, если начинать вообще абсолютно из ничего? Мы должны понять, что есть один "чего-то", видя перед собой один предмет (у меня в кордине одно яблоко), но когда рядом находится два схожих предмета (у меня в корзине два яблока), то мы говорим что их два? Но ведь яблоки не полностью аналогичны! Ведь одно яблоко может быть легче другого, или быть недозревшим. И вообще, что значи "понять"? Почему мы например должны решить, что аксиома математической индукции - правильная?

>> No.4871579  
Файл: kuri248.jpg -(311 KB, 1332x850, kuri248.jpg)
311

>>4871570

>Тогда такой вопрос: что было раньше, курица или я как мы можем формально определить какие-либо синтаксисы и семантики, если мы еще ничего не знаем о том, каким образом определяются какие-либо синтаксисы и семантики?

Уже спрашивали выше, посмотри: >>4870663 короткий ответ - никак, и приходится с этим как-то жить.

>Есть более глубокий вопрос, не знаю как этот вопрос сформулиловать вообще. Есть ли вообще какой-то фундамент познания математики, если начинать вообще абсолютно из ничего? Мы должны понять, что есть один "чего-то", видя перед собой один предмет (у меня в кордине одно яблоко), но когда рядом находится два схожих предмета (у меня в корзине два яблока), то мы говорим что их два? Но ведь яблоки не полностью аналогичны! Ведь одно яблоко может быть легче другого, или быть недозревшим. И вообще, что значи "понять"? Почему мы например должны решить, что аксиома математической индукции - правильная?

Тоже в том посте писал и тоже о таком задумывался: нет, нужно сразу принять очень много правил игры, чтобы построить машинерию в которой можно что-то выводить и определять. Никак не понять, но если ты веришь что натуральные числа они как бы есть и ты их хорошо понимаешь (а то есть те, кто не верят), то в них она выполняется, значит в неё тоже следует поверить и ввести как аксиому в свою дедуктивную систему.

>> No.4871594  
Файл: 1.jpg -(106 KB, 600x409, 1.jpg)
106

>>4871579

>Тоже в том посте писал и тоже о таком задумывался: нет, нужно сразу принять очень много правил игры, чтобы построить машинерию в которой можно что-то выводить и определять.

Является ли эта машинерия (правила игры в ней) прямым следствием законов т.н. физической реальности, в которой мы живем? (не уверен что этот вопрос имеет смысл)

>> No.4871596  
Файл: kuri249.jpg -(241 KB, 1400x1000, kuri249.jpg)
241

>>4871594
Думаю нет, скорее это мат. модель математического же доказательства и математического рассуждения, на это указывает как минимум то, что какой-то одной нету, а есть достаточно много таких вот машинерий и подходов к тому, как кодировать в них математические доказательства.

>> No.4871605  
Файл: kuri250.jpg -(38 KB, 723x1024, kuri250.jpg)
38

>>4871602
Мне кажется, что вопросы о том как fundamentals строить неинтересными, неважными и отвлекающими (по крайней мере в вопросе изучения математики). Но даже если рассмотреть твой аргумент в отрыве от этого, то мне кажется ровно наоборот - брутфорс и прямое вычисление всегда хуже высокоуровнего рассуждения, вскрывающего, так сказать, суть.

>> No.4871604  

>>4871550
Если определять как мощность декартова произведения в более выразительной теории, чем собственно арифметика, то действительно получится такое простое и ясное доказательство, но оно слишком привязано к натуральным числам. Мощность множества же не может быть отрицательной, например. Более низкоуровневое доказательство, напротив, мне кажется, позволяет понять, почему свойство выполняется, какие свойства структуры необходимы для его выполнения, а без каких можно обойтись. То есть если для свойства очень регулярной структуры есть элегантное доказательство, для которого регулярность является обязательной, — это хорошо, но сильнее доказательство, не использующее регулярность.

>> No.4871616  

>>4871605
По-моему, оба подхода "вскрывают суть", но с разных сторон. Общее низкоуровневое рассуждение раскрывает её со стороны необходимости и анализа, а высокоуровневое — со стороны достаточности и синтеза. Так сказать, диалектическое единство дополняющих друга друга методов.

>> No.4871619  
Файл: kuri250.jpeg -(10 KB, 193x262, kuri250.jpeg)
10

>>4871616
Пусть так, мне не принципиально!

>> No.4871650  
Файл: 19908259855583.jpg -(62 KB, 369x470, 19908259855583.jpg)
62

>>4871296

Я тут подумал/"вспомнил":

> Думаю важно, но не потому что другой что-то там говорит, это как раз не имеет значения, а потому что сам язык устроен так, что обращаться можно только к другому. Скажем, я помню что каждый учитель выходил в моей школе и все начинали шуметь, то шумел я не потому что никто меня не видел и поэтому я мог делать нечто безнаказанное, а как раз шумел на глазах отсутствующего места учителя, в некотором роде я считал это чуть ли не жестом флирта с учителем.

А почему ты пытаешься каким-то квадратно-гнездовым методом связывать идею с фактами реальности? Превращать её в какой-то тест, в модель по типу "выключателя" (нажал кнопку — мораль (/другой эффект) появилась, нажал — исчезла)?

Есть учитель (в твоих мыслях; не важно, где он в это время в реальности) и есть твои действия (другие мысли). Зачем "громоздить" понятие пустого места или вообще "отдалять картинку" из мозга в реальную ситуацию (т.е. от взаимодействия мыслей к тому какими тригеррами они были взываны)? Можно воспринимать идею не на причинно-следственном уровне, а на уровне связей которые она устанавливает и "важности" которую она придаёт таким-то вещам

По-моему неправильное смешивание разных уровней и приводит к "борьбе с монстрами" или вводу специфических понятий (пустое место учителя)... это же может тормозить и разработку ИИ (представление, будто машина должна думать о чём-то, что можно банально протестировать на опыте)

Каков тут математический подход? По-моему тут может быть даже связь с вашим диалогом:
>>4871538

> Мне кажется хорошее, потому что учит думать как думают математики: проассоциировать семантику с синтаксисом - заметить какую-то симметрию в семантике - вывести из этого следствие для синтаксиса.

А ты мог бы как-то провести аналогию с наукой? Если бы то же самое говорилось об исследовании мира?
>>4871570

> Есть более глубокий вопрос, не знаю как этот вопрос сформулиловать вообще. Есть ли вообще какой-то фундамент познания математики, если начинать вообще абсолютно из ничего?

Вот в познании реальности бы нельзя было начать абсолютно из ничего.
>>4871616

> По-моему, оба подхода "вскрывают суть", но с разных сторон. Общее низкоуровневое рассуждение раскрывает её со стороны необходимости и анализа, а высокоуровневое — со стороны достаточности и синтеза. Так сказать, диалектическое единство дополняющих друга друга методов.

Возможно, этот вопрос можно решить аналогией с физикой?

>> No.4871752  
Файл: kuri251.png -(1725 KB, 1802x2400, kuri251.png)
1725

>>4871650
Не знаю, мне как-то показалось такое построение естественным исходя из моих воспоминаний, в любом случае это абсолютно точно не было просто какой-то физической разрядкой или усталостью от концентрации внимания на уроке (хотя это тоже было). Я обычно как-то никогда не задумываюсь о естественности/неестественности реконструкции каких-то своих воспоминаний, просто думаю как мне хочется и всё. А математического подхода мне и в математике хватет.

>А ты мог бы как-то провести аналогию с наукой? Если бы то же самое говорилось об исследовании мира?

Думаю нет, в физике обычно противоположное происходит: подмечают какую-то симметрию в синтаксисе и затем уже стараются обнаружить её в реальности, чтобы в очередной (или в первый) раз убедится в, если не истиности, то как минимум полезности рассматриваемого синтаксиса. Обычно когда в реальности находят какой-то эффект ранее не присутствоваший в построениях, его в эти построения вводят. Всё-таки арифметическая истинность это не то же самое, что и "как в реальном мире".

>> No.4871999  
Файл: kuri253.jpeg -(6 KB, 259x194, kuri253.jpeg)
6
>> No.4872159  
Файл: F2JRQ11I5TI2KYM.LARGE.jpg -(159 KB, 1024x713, F2JRQ11I5TI2KYM.LARGE.jpg)
159

>>4870663

>https://math.stackexchange.com/questions/1334678/does-mathematics-become-circular-at-the-bottom-what-is-at-the-bottom-of-mathema

become circular at the bottom? Клеточные автоматы (вроде Conway's Game of Life) могут сами себя описывать и эмулировать, https://youtu.be/uRIHR55tyko например вот видео. И тогда можно сказать, что на самом деле нет никаких изначальных "клеток" в этом клеточном автомате нет, всё на себе само замыкается в бесконечности т.е. держится на какой-то рекурсии.

>> No.4872161  

>>4872159
Город Перестановок можешь почитать, там про клеточные автоматы есть в этом духе.

>> No.4872164  

>>4871596

>а есть достаточно много таких вот машинерий

А они все сводимы друг к другу? Т.е. верно ли то, что исходя из любой такой машинерии можно описать любую другую?

>> No.4872172  
Файл: kuri254.jpeg -(8 KB, 194x260, kuri254.jpeg)
8

>>4872159
Любая тьюринг-полная машинка эмулирует саму себя (а заодно любую другую такую машинку), это не очень интересно. Вопрос в том, что в случае твоей игры "жизнь" ты должен изначально принять/поверить правила самого низкого уровня, который должен быть.
>>4872164
Сводимы, но не каноническим способом, т.е. в то что сводимость на самом деле сводимость тоже нужно верить.

>> No.4872178  
Файл: 440px-Knights_of_the_Lambda_Calculus.svg.png -(42 KB, 440x440, 440px-Knights_of_the_Lambda_Calculus.svg.png)
42

>>4872172

>Вопрос в том, что в случае твоей игры "жизнь" ты должен изначально принять/поверить правила самого низкого уровня, который должен быть.

А что если нет никакого самого низкого уровня? Что если оно самоподобно и эмулирует само себя до бесконечности, как фрактал?
Или может быть ничего вообще нет, никаких законов, аксиом, и все вокруг - иллюзия? Дзен какой-то
https://without-answer.livejournal.com/83649.html

>Считается, что основной концепцией Дзена является вера (тезис) в то, что окружающий мир не существует, а существует только сознание мира (мир существует только в сознании). В действительности так описывают только внешнюю сторону Дзен-учения, создающую нездоровый антураж: так мы не приближаемся к истине, а изгоняем из мозга устоявшиеся стереотипы, помогая себе совершить свой первый, начальный шаг. На более глубоком уровне познания Дзен ничего не отрицает, он «всего лишь» доказывает, что посредством «чистого» сознания невозможно ни обнаружить, ни опровергнуть существование «внешнего» мира (материального, физического). Может быть, все мы «спим» и являемся частью чьего-то сна (прямо как в Матрице), а может, и нет. Кто знает?
>> No.4872181  
Файл: kuri255.jpg -(61 KB, 300x255, kuri255.jpg)
61

>>4872178
Ну можно формально рассмотреть такую коиндуктивную цепочку бесконечно вглубь уходящих синтаксисов, конечно, но в чём профит непонятно, обычно основания строят чтобы свести какие-то рассуждения о бесконечностях к конечному натуральному выводу из конечных строк.

>Или может быть ничего вообще нет, никаких законов, аксиом, и все вокруг - иллюзия? Дзен какой-то

Может!

>> No.4872195  
Файл: 5b6b3e792000009f00379402.jpeg -(285 KB, 720x890, 5b6b3e792000009f00379402.jpeg)
285

>>4872181

>Ну можно формально рассмотреть такую коиндуктивную цепочку бесконечно вглубь уходящих синтаксисов, конечно, но в чём профит непонятно, обычно основания строят чтобы свести какие-то рассуждения о бесконечностях к конечному натуральному выводу из конечных строк.

Дело не в профите. Получается, что для формального рассмотрения какой-либо кондуктивной цепочки, нужно иметь саму машинерию для построения формальной модели этой цепочки, а эта машинерия допустим описана в какой-то книге по матлогике, в книге по матлогике это описано на каком-то человекочитаемом языке с использованием неких слов, состоящих из неких букв, и для понимания написанного человеку тоже какая-то "машинерия" нужна т.е. знание смысла слов и предложений из них построенных, понимание их значений, а чтобы понимать те слова и предложения нужно еще что-то (ну допустим связи между нейронами в мозгах человека, которые позволяют выстраивать какие-то ассоциации и что-то "понимать", причинно-следственные связи выстраивать, распознавать буквы) и все это просто в какой-то момент должно уйти вот в такую рекурсию, находиться в таком самоподвешенном состоянии. Как Мюнхаузен, вытаскивающий сам себя из болота за волосы. По-моему это неизбежно. Иначе тогда просто ничего б не существовало.
Но это уже не математика, это какая-то метафизика и философия пошла.

>> No.4872204  
Файл: Al-RaziInGerardusCremonensis1250.JPG -(730 KB, 1067x1211, Al-RaziInGerardusCremonensis1250.JPG)
730

>>4872195

> Как Мюнхаузен, вытаскивающий сам себя из болота за волосы. По-моему это неизбежно. Иначе тогда просто ничего б не существовало. Но это уже не математика, это какая-то метафизика и философия пошла.

По-моему у тебя какая-то логическая ошибка: ты подразумеваешь, что через всю названную тобой цепочку (от матана до нейронов) проходит какая-то нигде не нарушающаяся нить, либо эта нить почему-то меняет свои свойства. Если мы "принимаем" детерменизм/материализм, то в нашем мире не существует ничего "высокоуровневого" в принципе. Либо наша реальность занимает лишь часть миров и высокоуровневые понятия на ней не завязаны. Если "не принимаем" (и у нас идеализм), то мы не спускаемся на уровень нейронов и атомов и нить заканчивается (да и без этого она похоже прошла, кажется, лишь через два цикла). Но думаю это не единственное, что способно обрубить фейковую (по-моему: вообще не люблю эти каузистики) причинно-следственную нить. Эта нить по-моему бессмысленна ещё и тем, что касается вопросов намного важнее её самой (как мы понимаем детерменизм и идеализм, например). Может ещё конец ответа хозяину треда как-то связан с этим всем

>>4871752

> Не знаю, мне как-то показалось такое построение естественным исходя из моих воспоминаний, в любом случае это абсолютно точно не было просто какой-то физической разрядкой или усталостью от концентрации внимания на уроке (хотя это тоже было). Я обычно как-то никогда не задумываюсь о естественности/неестественности реконструкции каких-то своих воспоминаний, просто думаю как мне хочется и всё. А математического подхода мне и в математике хватет.

Я не спорю с твоей идеей "в общем": спорю с дополнительным на неё наслоением (речь моей критики лишь о "пустом месте", ничём больше). Говорю, что вне причинно-следственных наслоений (то есть как ты сейчас написал: без них) наши идеи (заметки) не противоречат друг другу, а совпадают. Я как раз о чувствах типа флирта — и ты о том же. 100%-ой достоверности воспоминания тоже не требую (как и крепкой причинно-следственной связи концепции с реальностью, "тестируемости"). Поэтому меня и волнует, как ты читаешь

в любом случае это абсолютно точно не было просто какой-то физической разрядкой или усталостью от концентрации внимания на уроке (хотя это тоже было) — я как раз и предлагаю строить теории так, исключительно через обрубание одних вариантов (ослабление одних связей) и усилении других. Но это не обязаны быть прямые (полные) причинно-следственные связи, просто связи. Надо идти так, постепенно и определять степень (не)согласности тоже постепенно.

Не это является "подмечанием и обозначением"? Ты же попробовал как-то строго связать "семантику" и "синтаксис" (тригерры, физические условия реальной ситуации: был учитель, не было) или какие-то другие два разных уровня

> Думаю нет, в физике обычно противоположное происходит: подмечают какую-то симметрию в синтаксисе и затем уже стараются обнаружить её в реальности, чтобы в очередной (или в первый) раз убедится в, если не истиности, то как минимум полезности рассматриваемого синтаксиса. Обычно когда в реальности находят какой-то эффект ранее не присутствоваший в построениях, его в эти построения вводят. Всё-таки арифметическая истинность это не то же самое, что и "как в реальном мире".

Здесь о предсказаниях наперёд? Спасибо за ответ.

А разделение на синтаксис и семантику — оно само по себе (везде) полезное? Вот ты в первом изложении своего воспоминания по-моему смешал их, или какие-то другие два уровня абстракции смешал

Проводя аналогию с математикой (надоевшей), ты как будто считаешь что твои теории должны быть простыми функциями, устанавливающими взаимно-однозначное соответствие паре каких-то наобум выбранных событий (учитель есть/учителя нет) реальности и "событий" в твоей модели (как будто ты каждое событие должен объяснить, причём уникальным способом), в то время как твоя теория может быть (везде или лишь в ряде областей) и многозначной функцией
https://ru.wikipedia.org/wiki/Многозначная_функция
Есть учитель в классе? Нет его там? Да пофигу, это лишь тормозит [твою] идею, она не об этом
Если же тебе хочется (почему-то) строгих предсказаний от твоей теории/заметки — переложи это на дополнительные функции, которые могут превратить основную "многозначную" идею в "однозначную"

По-моему нечто такое тормозит и анализ языка (если помнишь пример, как можно не пытаться представлять языковые концепты алгоритмами), а то и всю ИИ-разработку — фокус на однозначных функциях, в то время как возможно надо идти в радикально-обратном направлении. А "однозначность" перекладывать на самые побочные вещи в алгоритме сети

>> No.4872222  
Файл: 500px-Formal_languages-ru.svg.png -(12 KB, 500x460, 500px-Formal_languages-ru.svg.png)
12

>>4872204

>По-моему у тебя какая-то логическая ошибка: ты подразумеваешь, что через всю названную тобой цепочку (от матана до нейронов) проходит какая-то нигде не нарушающаяся нить, либо эта нить почему-то меняет свои свойства. Если мы "принимаем" детерменизм/материализм, то в нашем мире не существует ничего "высокоуровневого" в принципе. Либо наша реальность занимает лишь часть миров и высокоуровневые понятия на ней не завязаны.

Я совершенно не понимаю то, что ты сейчас имеешь ввиду. Попробуй переформулировать свою мысль в более доступных мне понятиях

Я тоже попробую объяснить еще раз свою мысль, но немного другими словами.
Представим себе многослойный пирог. На верхних слоях допустим могут быть какие-то там пространства Калаби-Яу, группы Ли, топология и еще черти-что, уровнем ниже будут некие формальные языки и теории, в которых эти теории пространства Калаби-Яу описываются, еще уровнем ниже будет языки, описывающие языки для описания пространств Калаби-Яу... и так спускаемся вниз по этому многослойному пирогу, мы приходим к самому нижнему слою, некоторому формальному языку, который ни на что не опирается и описывает сам себя. И чтобы нам, людям, понять этот слой (его смысл), нам нужны какие-то книги по матлогике, описанные на уже понятном для нас естественном языке (русском, английском и так далее). Но ведь понимание смыслов естественных языков тоже магическим образом изниоткуда не возникает... Можно дальше попробовать разобраться в том, как работает думание человека, углубляться в физическое устройство мозга, ну там нейроны-аксоны всякие, можно прийти к построению каких-то моделей искуственных нейросетей и мы тут уже зациклились. Работу самого мозга мы описываем в терминах, которые были выведены (и которые понимаемы, осознаваемы) через думание мозгом.

>> No.4872233  
Файл: 4767_371977.jpg -(1330 KB, 1400x1400, 4767_371977.jpg)
1330

>>4872222

> Я совершенно не понимаю то, что ты сейчас имеешь ввиду. Попробуй переформулировать свою мысль в более доступных мне понятиях

Ты обозначил цепочку причин и следствий: Калаби-Яу <- простейшие понятия <- неформальный язык <- буквы <- нейроны ("синтаксис"). В ней самой видны какие-либо петли? Нет. Значит, петля получается в какой-то другой цепочке ("семантической"), которую ты накладываешь на первую. По каким правилам ты её накладываешь? В чём ты увидел схожесть этих причин и следствий? Ты не пишешь, ты только подразумеваешь. А мы уже можем увидеть даже наоборот, несхожести:

Я сомневаюсь, что буквы создают смысл слов так же как простейшие мат. понятия создают сложнейшие. Я даже могу сразу не согласиться, что идея пространства Калаби-Яу имеет хоть что-либо общее с построением его в какой-то мат. системе (и твоя цепочка, возможно, даже не начнётся)

Ты смешал факты и идеи, "физические" причины и следствия с "аргументационными" причинами и следствиями. Беспорядочно мешаешь всё это вместе чтобы сделать нужный тебе вывод/парадокс

> Но ведь понимание смыслов естественных языков тоже магическим образом изниоткуда не возникает... Можно дальше попробовать разобраться в том, как работает думание человека, углубляться в физическое устройство мозга, ну там нейроны-аксоны всякие, можно прийти к построению каких-то моделей искуственных нейросетей и мы тут уже зациклились. Работу самого мозга мы описываем в терминах, которые были выведены (и которые понимаемы, осознаваемы) через думание мозгом.

А тут ты уже переходишь вообще к другому типу зацикленности, и подменяешь "должны понять мозгом" на "должны сами воссоздать модель мозга, которой поймём" (ну подумай: какое отношение познаваемость мозга имеет к твоему аргументу? никакое, а ты уже в это соскользнул)

То есть ты сейчас сам описал модель, которая опровергает твой собственный аргумент, т.к. в ней нет никакой зацикленности ("пирог"): это же (не в обиду) феерия абсурда и лицемерия (лишь бы в чужую голову пустить своё семяизвержение)

То есть ты всего лишь рассказал анекдот, основанный на двусмысленности (это мне помогли понять собственные глупые слова про извержение в чью-то голову)

>> No.4872275  

>>4872233

>Ты обозначил цепочку причин и следствий: Калаби-Яу <- простейшие понятия <- неформальный язык <- буквы <- нейроны ("синтаксис"). В ней самой видны какие-либо петли?

Петля возникает на этапе, когда нам надо сделать язык для описания языков. Этот язык должен будет сам себя описывать, но как нам его начать описывать на нем самом, если его еще нет в нашей голове?
Напрямую - никак. Нам надо описывать его словами из естественных языков. Понимание смыслов слов из естественных языков тоже само собой не возникает. Спускаясь всё ниже, мы приходим к нейронам мозга и физико/химическим процессам в этих самых мозгах, которые являются процессом думания. Для описания этих процессов нам тоже нужны какие-то формальные языки...

>Значит, петля получается в какой-то другой цепочке ("семантической"), которую ты накладываешь на первую.

Я тебя не понимаю. Твои фразы для меня лишены смысла, увы!

>По каким правилам ты её накладываешь?

Есть X. Чтобы понять как работает X, нам нужен Y. Чтобы понять как работает Y нам нужен Z...
Для понимания формального языка описания формальных языков нам нужна книга по матлогике, в которой это написано человеческим естественным языком, но мы всё еще не понимаем то, каким образом мы понимаем этот человеческий язык, и нам надо изучать устройства человеческого мышления, работу нейронов. Вот так и накладываю, что тут непонятного?

>Я сомневаюсь, что буквы создают смысл слов так же как простейшие мат. понятия создают сложнейшие.

Смысл слов создает наша интерпретация букв в голове (нейронами мозга).

>Ты смешал факты и идеи, "физические" причины и следствия с "аргументационными" причинами и следствиями.

Смысл твоей фразы неясен. По-моему тут всё вообще физично. Наши мысли и рассуждения о чем-либо являются вполне себе физическим процессом, нервным импульсом между нейронами. Все наши мысли и идеи, все наши рассуждения и модели это просто физический процесс, так что всё едино. Они не могут существовать где-то "в астрале".

>А тут ты уже переходишь вообще к другому типу зацикленности, и подменяешь "должны понять мозгом" на "должны сами воссоздать модель мозга, которой поймём" (ну подумай: какое отношение познаваемость мозга имеет к твоему аргументу? никакое, а ты уже в это соскользнул)

И тут я опять ничего не понял. Мне кажется что ты пытаешься додумывать за меня, что я якобы что-то такое имел ввиду, но на самом деле я совсем о другом говорил... эхх, когда уже разработают прямую телепатию.

>То есть ты сейчас сам описал модель, которая опровергает твой собственный аргумент, т.к. в ней нет никакой зацикленности ("пирог"): это же (не в обиду) феерия абсурда и лицемерия (лишь бы в чужую голову пустить своё семяизвержение)

Где я сам себя опровергаю? Я тебя вообще не понимаю

>> No.4872276  
Файл: 2878559.jpg -(75 KB, 595x600, 2878559.jpg)
75

>>4872233
Или может быть ты хочешь перйти на рассуждения про всяких философских зомби, идеи Чалмерса? Типа "что значит что-то понимать", "что значит ощущать что-либо" или "есть ли у человека душа"?

>> No.4872288  

Давайте, пожалуйста, отложим в сторону философию, о ней можно очень долго писать и всё без толку.

Я хотел бы кое-что всё-таки уточнить по системам автоматического доказательства. Вот ОП утверждал, что они не добавляют ему уверенности. Но ведь если доказательство не было проверено машиной, то там ошибка может быть в любой строчке! При каком-то переходе мы неявно приняли предположение, при каком-то вообще просто неправильно переписали. Даже если его внимательно прочитали 5 математиков, они всё равно могли пропустить ошибку.

Может ли быть такое, что часть доказательств, которым доверяют математики, на самом деле с ошибками?

В случае же с автоматической системой нам нужно только проверить, что мы правильно закодировали саму теорему, но это же гораздо меньшая работа. Само доказательство для автоматической проверки же можно писать механически, без понимания, что означает каждый шаг. Но зато машина обеспечит нам то, что эти шаги верны в её системе.

Нужно ещё доверять компилятору, но есть ведь как минимум тесты, а как максимум формальная верификация.

В любом случае лично я бы доверял этому гораздо больше, чем тому, что 5 математиков осознали каждый шаг 50-страничного доказательства. Почему я не прав?

Если меня, например, спросят, скомпилируется ли эта достаточно длинная программа, и дадут её прочитать, то я дам ответ с очень-очень-очень маленькой уверенностью. Но достаточно попытаться её скомпилировать, и даже несмотря на то, что могут быть баги в компиляторе, я бы оценил вероятность их встретить как пренебрежимо малую по сравнению с вероятностью ошибиться при ручном ревью.

>> No.4872313  
Файл: kuri256.jpg -(67 KB, 600x900, kuri256.jpg)
67

>>4872288
Ну я немного передергнул, конечно, на самом деле меня как-то не особо тревожит правильно доказательство или нет, если оно меня убеждает. Просто люблю математику, а писать статьи не люблю, а если ещё заставят с теориями типов себе мозги насиловать - так вообще сил никаких не будет, уйду тогда сайты верстать на апворке.

Но всё же, то что в метамате незакодировали даже такой примитивный геометрический объект как многообразие, и какую-нибудь легкодоказываемую геометрическую теорему, вроде классификации компактных 2-многообразий (которая, емнип, чуть ли не старше основной теоремы алгебры, которая там закодирована) говорит о том, что далеко не всё кодируется одинаково просто. Возможно кодирование даже формулировки той же гипотезы Ходжа (ну, скажем, в варианте "функтор из гладких компактных алг.многообразий /С в чистые структуры Ходжа faithful") будет включать в себя несколько десятков страниц кода, в которых, конечно, всё равно никто разбираться не будет.

>> No.4872339  

>>4872313
А почему математики редко когда занимаются полной формализацией своих доказательств? Почему например никто до сих пор не формализовал доказательство для Великой теоремы Ферма (ни в metamath, ни в coq ни в чем-либо еще)? Или это считается неважным?

>> No.4872343  
Файл: kuri257.jpg -(16 KB, 600x337, kuri257.jpg)
16

>>4872339
Считается неважным, есть кроме этого чем заниматься. Кроме того, надо же хлеб оставлять CS аспирантам, они на кодировании какого-либо пруфа часто кандидатские защищают, вроде как.

>> No.4872356  

>>4872313

>Ну я немного передергнул, конечно, на самом деле меня как-то не особо тревожит правильно доказательство или нет, если оно меня убеждает.

А как часто было так, что кто-то что-то доказал, сообществом профессиональных математиков это доказательство было проверено и они убедились, что в этом доказательстве всё правильно, но потом спустя скажем 2-3 года оказывалось, что это доказательство было с ошибками? Есть ли вообще примеры подобного?

>> No.4872363  
Файл: kuri258.jpeg -(7 KB, 159x318, kuri258.jpeg)
7

>>4872356
Да, куча раз, начиная с Евклида который использовал некоторые неявные предположения (вроде то, что прямая пересекает окружность либо в 0, либо в 1, либо в 2 точках) при доказательстве каких-то теорем, хотя это, наверное, можно спустить на то, что раньше стандарты строгости были другими. Фреге, чьи интуитивные аксиомы оказались противоречивыми, Коши доказал в своём учебнике по анализу что любая непрерывная функция почти-всюду дифференцируема, руководствуясь неявным предположением, что прямую можно разбить на промежутки невозростания и неубывания любой непрерывной функции.

Из более современного: Понтрягин сделал ошибку при вычислении второй стабильной гом. группы, уже упомянаемая мною комплексная гипотеза Якобиана к которой было куча пруфов, пока кто-то не построил контрпример. То что lim^1 зануляется на системах Миттаг-Леффлера тоже была т.н. "теоремой 1961" позже построили контрпример. Был известный скандал с неправильной публикацией пруфа того, что на S^6 есть комплексные структуры, потом нашли ошибку, потом опубликовали правильное, но уже другой автор другим методом. И это только самые громкие истории, ошибок поменьше в более узких областях - не счесть.

>> No.4872462  
Файл: kuri259.jpeg -(8 KB, 300x168, kuri259.jpeg)
8

>>4872313

> на самом деле меня как-то не особо тревожит правильно доказательство или нет, если оно меня убеждает.

Кстати обратное тоже верно: мне не так важно насколько верно доказательство, если оно меня не убеждает. Ну, то есть, референцы делать на доказываемый факт я буду, но если я его нутром не прочувствовал, то я для себя обычно за доказательство это не воспринимаю.

>> No.4872466  

>>4872313
Ну а как же там наука, которая устанавливает объективную истину и всё такое?..

>> No.4872468  
Файл: kuri260.jpg -(18 KB, 236x419, kuri260.jpg)
18

>>4872466
Ну те кому нужна объективная истина пусть и кодируют свои или чужие пруфы в MLTT термы, если считают что это их к этой объективной истине приблизит, а мне нужна математика - поэтому я не буду.

>> No.4872519  
Файл: Möbius_strip.jpg -(102 KB, 1328x824, Möbius_strip.jpg)
102

>>4872275

> Петля возникает на этапе, когда нам надо сделать язык для описания языков. Этот язык должен будет сам себя описывать, но как нам его начать описывать на нем самом, если его еще нет в нашей голове?
> Напрямую - никак. Нам надо описывать его словами из естественных языков. Понимание смыслов слов из естественных языков тоже само собой не возникает. Спускаясь всё ниже, мы приходим к нейронам мозга и физико/химическим процессам в этих самых мозгах, которые являются процессом думания. Для описания этих процессов нам тоже нужны какие-то формальные языки...

Я вижу тут как минимум ложное подразумевание (требование): на stackexchange ответили, что сохранение формализма до самого конца не требуется — что-то описывается на естественном языке и всё
Как максимум — где петля-то, Джонни, где? "Язык описывающий сам себя" это петля, но только на одном звене, дальше ты только подразумеваешь что-то (не говори "возникает на этапе", скажи что ты считаешь петлёй)

Мне кажется, тема "петли" могла подменяться уже несколько раз. Тут >>4872159 например вообще ничего не говорится об интерпретации математики человеком. Тут >>4872195 тоже ещё ничего не говорится об интерпретации и, кстати, вместо называния петли тоже происходит лишь "взмахивание руками"

> Есть X. Чтобы понять как работает X, нам нужен Y. Чтобы понять как работает Y нам нужен Z...
> Для понимания формального языка описания формальных языков нам нужна книга по матлогике, в которой это написано человеческим естественным языком, но мы всё еще не понимаем то, каким образом мы понимаем этот человеческий язык, и нам надо изучать устройства человеческого мышления, работу нейронов. Вот так и накладываю, что тут непонятного?

Ты назвал лишь регрессию, причём даже не бесконечную: не понятно, где заявленная петля.

> И тут я опять ничего не понял. Мне кажется что ты пытаешься додумывать за меня, что я якобы что-то такое имел ввиду, но на самом деле я совсем о другом говорил... эхх, когда уже разработают прямую телепатию.

Никогда и не надо! Это позволило бы оставаться такими же лицемерами без эмпатии и не качать логику — и вряд ли сработала бы даже телепатия

Ты понимаешь (хотя бы на уровне образов), что многослойный пирог и петля — понятия довольно неблизкие?

>> No.4872559  

>>4872468
Прости, если прозвучало обидно, обижать никого не хотел!

>> No.4872570  
Файл: kuri261.png -(2327 KB, 2100x2700, kuri261.png)
2327

>>4872559
Нет, ты что! Я это не воспринял как попытку обидеть, а воспринял как наоборот выражение тревоги и бессилия в духе "Как же так, ведь всё развалиться тогда!" и хотел показать, что у меня ровно так и ничего не развалилось, чтобы попытаться хоть чуть-чуть эту самую тревогу унять!

>> No.4872691  
Файл: kuri262.jpg -(502 KB, 1920x1875, kuri262.jpg)
502
>> No.4872694  
Файл: ESCHER_drawing_hands.jpg -(130 KB, 400x344, ESCHER_drawing_hands.jpg)
130

>>4872519

>Как максимум — где петля-то, Джонни, где?

Я тебе уже всё что можно объяснил, видимо ты не можешь это осознать. Петля возникнет в момент, когда ты начнешь рассматривать механизмы человеческого мышления, строить какие-то математические модели нейронов, связи нейронов. В этот момент и возникает, зацикливание. Ты у себя в голове (своими нейронами мозга, процессами в своих человеческих мозгах) строишь модель того, каким образом эти процессы в твоих мозгах работают. Если тут по-твоему никакого зацикливания нет, то тут уж я бессилен.

>> No.4872696  

>>4872462

>Кстати обратное тоже верно: мне не так важно насколько верно доказательство, если оно меня не убеждает.

По-моему плохо это. Убедительность - плохой критерий истинности чего-либо.

>> No.4872697  
Файл: kuri263.jpeg -(7 KB, 205x246, kuri263.jpeg)
7

>>4872696
А я не гонюсь за истинностью, впрочем как и за хорошим.

>> No.4872702  

>>4872697
А за чем ты гонишься? Что тебя привлекат в математике?

>> No.4872709  
Файл: a8BUP.jpg -(148 KB, 1200x1500, a8BUP.jpg)
148

>>4872702
Да сложно сказать. Испытываю наслаждение того же рода как и люди, которые узнают что-то новое о персонажах своей любимой вымышленной вселенной, типа звездных воин, наверное.

>> No.4872712  
Файл: SIFT features examples.png -(399 KB, 1074x689, SIFT features examples.png)
399

>>4872570

> Нет, ты что! Я это не воспринял как попытку обидеть, а воспринял как наоборот выражение тревоги и бессилия в духе "Как же так, ведь всё развалиться тогда!" и хотел показать, что у меня ровно так и ничего не развалилось, чтобы попытаться хоть чуть-чуть эту самую тревогу унять!

Не кажется ли этот ответ немного лицемерным? "Я не обиделся, ты просто бессилен, я тебя серьёзно не воспринимаю и психологию твою препарировал" (ладно, это всё как шутка, я собираюсь уходить)

Из поста Аввы
https://avva.livejournal.com/3190186.html?view=140358314

И ссылки в нём
http://www.incompleteideas.net/IncIdeas/BitterLesson.html

Узнал о SIFT features (видимо это должно быть самым близким к тому, что "уже было")
https://ru.wikipedia.org/wiki/Масштабно-инвариантная_трансформация_признаков
https://habr.com/ru/post/106302/

Но это довольно-таки далеко от >>4870165, эта штука ничего не делает (не сравнивает, не образует из мелких штук большие штуки, т.е. пытается распознавать по произвольныму шуму, не будет способна к абстракции... как прочитал на Хабре ей ещё и перебирать пары свойств надо т.к. их дофига и никакой иерархии нет). Углы и цилиндры не лучше, это тоже частности, которые сами по себе не важны

>>4872694

  • Ты способен осознать, что раньше не называл прямо петлю, по крайней мере в ряде постов не называл? Я тебе дал чёткую просьбу, ты её исполняешь (спасибо) при этом почему-то критикуя меня диким обобщением про "уже объяснил всё что можно" — нет, ты до этого ещё не исполнял моей просьбы например
  • Ты способен осознать, что в постах >>4872159 и >>4872195 (особенно в первом) никакого намёка на то о чём ты говоришь сейчас? Тема подменена, ну да ладно
  • Почему ты не признаёшь, что нигде не иллюстрировал петлю, иллюстрируя только регрессию и пироги? Это помогло бы тебе снизить накал твоего лицемерия с "меня никто не понимает"... проявив больше уважения ко мне, а не комментируя мои способности когда сам можешь быть жуком (прости за токсичность, но неуважение не должно прощаться легко)

Другие не должны страдать из-за твоего нетерпения и твоей паники — ты не знаешь, бессилен ты или нет, ты хочешь загадать заранее, потому что иначе было бы просто страшно. Но ты не знаешь, что я отвечу, поэтому это невозможно (наперёд загадывать) и даже неуважительно

> Петля возникнет в момент, когда ты начнешь рассматривать механизмы человеческого мышления, строить какие-то математические модели нейронов, связи нейронов.
> Ты у себя в голове (своими нейронами мозга, процессами в своих человеческих мозгах) строишь модель того, каким образом эти процессы в твоих мозгах работают.

Как писал уже с >>4872233, требование описать всё формально идёт лишь от тебя самого... ну причём тут познаваемость мозга/алгоритма мышления? Ты назвал петлю, но потерял контекст

Почему петля по-твоему возникает только здесь, а не уже при попытке понять законы физической реальности? Почему в моих? Я могу взять/вскрыть чужую голову

Как писал там же (>>4872233) это может оказаться просто плохим анекдотом, основанным на двусмысленности. Анализировать что-то в реальности с помощью математики (самых поверхностных вещей из неё) и анализировать её саму это разные вещи (там и области другие: физика, программирование, там все концепты реальны и указываемы пальцем). Ты подразумеваешь некую монолитность всего этого... Хотя ты, чёрт возьми, подменил тему ещё раз, мы же теперь вообще о математике не говорим получается.

Мир моделей, запрограммированных моделей — это уже не совсем мир полностью абстрактных концепций и истин

Вот смотри: в >>4872159 никакой речи о людях и 100% о матане. Теперь 0% о матане и 100% о возможности теории интеллекта (/основной ответ)

Ну блин, ты просто посмотри на других в треде — всем в голову что-то ударило — и все хотят насадить своё мнение (и паникуют, если не гладят по головке). А ОП высказывает мнение, а потом уходит в полное отрицалово (>>4872697 например), противоречащее порой даже его собственным словам (мне особенно обидно от воспоминаний о таком же друге, который умён но таким макаром никакой диалог просто невозможен)... Тред об эмуляции мышления людьми с 0-ой выносливостью (могут потратиться на разрядку своих идей в кого-то, но не на адаптацию к "неожиданным поворотам", к взаимодействию)

Это как-то жалко (и безысходно: у меня самого идеи, которые "не могут осознать" и здоровье давно упало)

>> No.4872714  

Хороший и интересный тред. Но к тому моменту как почитаешь ничего писать в него уже не хочется. Не с таким настроем обычно в /b/ захожу. Не чтобы думать. Тем более на уровне трансляции формул и прочих научно-терминологических абстракций.

>> No.4872715  

>>4872709

> Да сложно сказать. Испытываю наслаждение того же рода как и люди, которые узнают что-то новое о персонажах своей любимой вымышленной вселенной, типа звездных воин, наверное.

>>4870591

> Понятно, я вот скорее стараюсь наоборот избавиться от своего этого желания удерживать какие-то знания.

Я тогда как раз хотел сказать именно о подобного рода информации.. классификация же, помнить любимых людей и прочее (любимое)...

(Why are we still here?)

>> No.4872716  

>>4872712
Возможно немного не в тему. Касательно картинки прикреплённой к твоему посту. Как раз недавно думал, что подавляющее большинство растровых изображений, при желании, чисто теоретически, возможно перекодировать в вектор с наложением определённых растровых эффектов. С весьма высокой степенью точности воспроизведения конечного результата.

>> No.4872718  
Файл: kuri265.jpeg -(7 KB, 209x241, kuri265.jpeg)
7

>>4872712
Да остовайся, я просто очень поверхностно разбираюсь в машин лёнинге и не могу накидать тебе каких-то интересных алгоритмов, за которые ты бы мог зацепиться, а очень хотел бы!

>Не кажется ли этот ответ немного лицемерным? "Я не обиделся, ты просто бессилен, я тебя серьёзно не воспринимаю и психологию твою препарировал" (ладно, это всё как шутка, я собираюсь уходить)

Бессилие я воспринимаю очень серьезно, так как нахожусь в нём перманентно и там даже специально было "я воспринял" чтобы никто не дай бог не подумал, что я разоблачаю кого-то! (хоть это и была шутка (не уходи))

>>4872715
Ну у меня именно желания удерживать знания нет, вернее есть, но оно сугубо капиталистическое в духе "я должен быть хорошим математиком, чтобы мне платили".

>>4872714
У нас тут не было формул вроде! Одни бака-толки, так что в духе /b/ очень как раз (как я того и хотел)!

>> No.4872730  
Файл: Скриншот сделанный 2019-03-19 в 07.22.28(...).png -(398 KB, 1349x1661, Скриншот сделанный 2019-03-19 в 07.22.28(...).png)
398

>>4872718

Так со мной итак все диалоги были закончены. (Ну ладно)

> Ну у меня именно желания удерживать знания нет, вернее есть, но оно сугубо капиталистическое в духе "я должен быть хорошим математиком, чтобы мне платили".

Но это уже не про удовольствие, которое ты описал до того! Может тут тоже какая-то наша или моя ошибка в "максимализме": мышлении только полными причинами?

Для меня ценность информации это как развитие морали: для меня невозможно ценить лишь саму по себе "жизнь" людей. Вот тут кто-то запостил картинку с фиолетовой девочкой, сборник разных поз — это для меня боль, но что остаётся делать? Хочется отбросить, но это информация, и другой про неё информации нет — надо усиленно схоронять. И чем больше знаешь людей, которые могли бы оказаться в такой же ситуации, тем сильнее усиленней боль (как будто всех котов потянули за хвост сразу, все струны)

Цена информации это ещё как попытка выбраться из своих низменных желаний, сублимировать их, перерасти
>>4872716

Для меня прикреплённая картинка это пример "как не надо делать"
>>4872694

То есть я хочу сказать, что любые "петли" получаются лишь за счёт каких-то [семантических] обобщений, они (петли и обобщения) лишь в нашей голове, с любым из обобщений можно не согласиться (и "сломать" петлю).

Физическая реализация матана не уходит бесконечно вглубь/в циклы, поэтому взяв банальный калькулятор ты можешь изучать любые штуки без зацикливания... тут уже зависит от выводов, которые ты хочешь сделать (элемент зацикленности остаётся, но важен ли он?)

В >>4872195 ты говоришь "ничего бы не существовало"... Это о чём?

>> No.4872878  
Файл: kuri266.jpg -(111 KB, 700x1244, kuri266.jpg)
111
>> No.4873125  

>>4872878

Хотел ещё что-то сказать, но понял, что ересь, поэтому просто философский бамп!

>> No.4873504  
Файл: kuri267.jpeg -(10 KB, 189x267, kuri267.jpeg)
10
>> No.4873942  
Файл: kuri268.jpeg -(5 KB, 224x224, kuri268.jpeg)
5

.

>> No.4874324  

Правильно ли преподают математику школьникам? Например, зачем там по 1000 раз их заставляют решать сначала тупые примеры вида 5+7=?, потом по 1000 раз заставляют решать всякие уравнения вида 5*x^2+7*x+4=0 (считать дискриминанты всякие), а потом в вузах студентов долбят какой-то фигней типа посчитайте 100500 интегралов... зачем это вообще нужно? Как часто умения решать всякие интегралы пригождается кому-то в реальной жизни? Ведь проще всю эту муть загрузить в какой-нибудь Maple и пусть оно там его символьно интегрирует, если это так надо. Мы живем в век цифровых технологий, автоматизация и все такое, зачем нужны навыки подобные этим?

>> No.4874329  

>>4874324
Чтобы ты знал, какую лампочку тебе вкручивать в светильник заместо перегоревшей и вышедшей из производства, юзернейм.

>> No.4874332  
Файл: gggg.jpg -(374 KB, 2048x1365, gggg.jpg)
374

>>4874324
Математику преподают школьникам правильно.
Другое дело что школьники игнорируют основную часть урока (Объяснение) и от того страдают на закреплении/проверке, то есть тупых примерах, уравнениях и интегралах.
Тут проблема не с математикой, а с педагогикой в целом, когда правильный в общем-то урок пихают в человеков, которых никто не учил учиться.

>> No.4874440  
Файл: LogSpyr_1.png -(15 KB, 305x324, LogSpyr_1.png)
15

Какая фрактальная размерность у логарифмической спирали?

>> No.4874533  
Файл: kuri269.jpg -(15 KB, 512x288, kuri269.jpg)
15

>>4874324
Думаю неправильно, но не потому что это никому не нужно в реальной жизни, а потому что тупо скучно, у тебя был вуз плохой наверное, меня вот не заставляли 100500 интегралов брать. Заставили взять штук 20-30 по одному на каждый трюк, я их взял, потратив на это один вечер, а потом положил на место и больше не брал!
>>4874440
1, как и у любого гладкого погружения гладкой кривой!

>> No.4874548  

>>4874533

> 1, как и у любого гладкого погружения гладкой кривой!

Но почему тогда у неё может быть бесконечная длина (я может быть не очень разбираюсь в спиралях, но мне нужна бесконечная, такие явно есть) в конечной площади?
У набора концентрических окружностей с радиусом, приближающимся к нулю, тоже 1? Длина ведь тоже бесконечная.

Я почему-то сделал, видимо, неправильный вывод, что если размерность 1, то бесконечную длину не должно быть можно вписать в конечную площадь...

>> No.4874556  
Файл: kuri270.jpg -(129 KB, 1000x1000, kuri270.jpg)
129

>>4874548
У окружностей тоже 1. Видимо неправильно, правильно наоборот: если у образа непрерывной кривой f : [0..1] -> R^2 размерность >1, то длина обязательно бесконечная.

>> No.4874640  
Файл: sad anime (2).jpg -(174 KB, 1083x720, sad anime (2).jpg)
174

>>4874324
Математика как язык формального, но абстактного мышления это то, что отделяет человека от обезьяны. Математика в физике это вообще магия -- вместо того, чтобы лезть на дерево с рулеткой, можно по его тени узнать его высоту... Если владеть заклинанием подобных треугольниоков и иметь при себе эталонную палку. На более высоких уровнях появляется возможность предсказывать будущее и строить всякие коллайдеры, ракеты и прочие жпсы.
К сожалению именно этот, практический аспект как-то пролетает мимо учеников. И у них математика ассоциируется исключительно с перекладыванием символов туда-сюда чтобы препод отстал.

>> No.4874739  

>>4874640
Ну не встаёт перед современными школьниками задачи регулярно измерять высоту дерева рулеткой.
Поработав пару лет по специальности я не раз ловил себя на мысли, что вот сейчас с удовольствием прослушал бы курс какой-то из своих университетских дисциплин ещё раз и вероятно даже вынес бы оттуда что-то полезное, что раньше вынести не удалось.

>> No.4874781  
Файл: kuri271.jpeg -(8 KB, 204x247, kuri271.jpeg)
8
>> No.4875527  

Как проще всего доказать, что e^(i*pi*x)=(-1)^x ?

>> No.4875718  
Файл: kuri272.jpeg -(12 KB, 201x251, kuri272.jpeg)
12

>>4875527
Ну это как минимум неаккуратно, а как максимум неправда. (-1)^x := exp(x Ln(-1)) не well-defined функция на C, она определена на римановой поверхности логарифма, т.е. на "винтовой лестнице" универсальном накрытии плоскости с выколотой точкой и ветвлением в нуле. Без заумностей и грубо: непонятно какой из трёх кубических корней мы должны выбрать при x=1/3, скажем.

Правильно exp(i pi t) = cos(t) + i sin(t), ну зависит от твоих определений exp, sin и cos, обычно их определяют через соответствующие ряды тейлора в нуле и там дальше прямое вычисление lhs и rhs в терминах рядов тейлора и покоэффициентное сравнение.

>> No.4875719  

>>4875718
exp(i t) = cos(t) + i sin(t), я это убрал а глупая вакаба всё равно так отправила!

>> No.4875734  

>>4875718
А почему https://www.wolframalpha.com/input/?i=(-1)%5Ex WolframAlpha считает что (-1)^x это то же самое, что и cos(pi x) + i sin(pi x)? Как вообще считать возведение в нецелую степень для отрицательного числа (где про это написано)?

>> No.4875742  
Файл: formula.png -(5 KB, 462x142, formula.png)
5

Есть ли обобщения гипероператора для нецелых значений гипероперации?
Ну т.е.
Hyper(a, b, 0) = b + 1
Hyper(a, b, 1) = a + b
Hyper(a, b, 2) = a * b
Hyper(a, b, 3) = a ^ b
Hyper(a, b, 1.5) = ???

>> No.4875765  
Файл: kuri273.jpg -(430 KB, 1105x1631, kuri273.jpg)
430

>>4875734
Взяли ветвь в которой ln(-1) = pi i, а не ln(-1) = - pi i например, почему именно эту - не знаю, захотели так. Где почитать - не знаю, напиши "сomplex logarithm pdf" в гугле, наверняка кучу гайдов для совсем новичков найдешь. Но в принципе я всё совсем основное уже рассказал (-1)^z это по определению exp (z Ln(-1)) проблема в том что Ln(-1) это не число, а число с точностью до 2 pi i n, поэтому у тебя есть свобода в выборе функции: ты можешь скажем выбрать pi i, а можешь выбрать -pi i и это будут разные функции (скажем в точке z=1/2 просто разные числа выйдут).
>>4875742
Не задавался таким вопросом, погуглил https://math.stackexchange.com/questions/1227761/example-x-y-and-z-values-for-x-uparrow-alpha-y-z-where-alpha-in-bbb выглядит как-будто общепринятых нет, а необщепринятые какие-то сильно ограниченные. Например в https://math.eretrandre.org/tetrationforum/attachment.php?aid=1187 Hyper(b,z,4) определена только для 1<b<e^1/e и для всех z \in C. Но вопрос милый.

>> No.4876054  
Файл: kuri274.jpg -(53 KB, 700x466, kuri274.jpg)
53
>> No.4876090  

>>4874739

>Ну не встаёт перед современными школьниками задачи регулярно измерять высоту дерева рулеткой.

Значит нужно найти пример ближе к обделенном воображением ученикам. Или взять их в "поле" для наглядного эксперимента.

>> No.4876248  
Файл: kuri275.jpg -(50 KB, 1360x800, kuri275.jpg)
50
>> No.4876445  

>>4875718

> Без заумностей и грубо: непонятно какой из трёх кубических корней мы должны выбрать при x=1/3, скажем.

Ну понятно, у (-1)^(1/3) три корня, у (-1)^(1/2) два корня (-i и i), а сколько корней будет например у (-1)^(1/pi) или (-1)^(1/(pi+e))?
Кстати, оказывается что для pi+e даже не доказано, рационально оно или нет. И если оно вдруг окажется рационально, то там очень много корней будет.

>> No.4876471  
Файл: kuri276.jpeg -(5 KB, 276x183, kuri276.jpeg)
5

>>4876445
Ну опять же (-1)^x = exp(x Ln(-1) = exp(i (pi + 2 pi n) x) так что у тебя по одной "функции вида (-1)^x" на каждое целое число n так что вопрос о "количестве значений" (-1)^x это вопрос о мощности множества {exp(i (pi + 2 pi n) x), n \in Z}. Как ты правильно заметил этот вопрос сильно зависит от рациональности x, если x рационально то множество выше всегда будет конечно (и мощность будет равна знаменателю в представлении x в виде несократимой дроби), если иррациональное - то все числа будут попарно различны - и будет по одному значению для каждого целого числа. И, как ты правильно заметил, вопрос о рациональности pi+e открытый, в то время как факт иррациональности pi хорошо известный.

Ну и кстати, именно из-за этой неоднозначности взрослые люди (-1)^z обычно не пишут, а пишут сразу ту ветку которая им нужна: типа exp(3 i pi z).

>> No.4876516  

>>4876471
Тут вообще какая-то ерунда с этой (-1)^x
Ну вот есть такое правило, что a^b * a^c = a^(b+c). И оно вроде бы правильное, но это неточно.
Возьмем (-1)^1
(-1)^1 = (-1)^(0.5 + 0.5)
(-1)^(1/2 + 1/2) = (-1)^(1/2) * (-1)^(1/2)
тут уже какая-то ерунда получается. Когда у нас (-1)^1 то мы точно можем сказать, что это равно -1.
Когда у нас (-1)^(1/2) * (-1)^(1/2) то левую часть мы можем решить как i, правую мы можем решить как -i, и получим что i*(-i) = 1? Или так нельзя? Если нельзя - почему?

>> No.4876539  
Файл: kuri278.jpg -(808 KB, 1291x1500, kuri278.jpg)
808

>>4876516
Ну потому что это

>a^b * a^c = a^(b+c)

неверно, это верно только если мы заранее зафиксировали какое-то одно число Ln(a) (или, если рассуждать глобально: выбрали ветвь римановой поверхности ln), и используем одно для всех.

Тут особо никакой специфики комплексных чисел нету, такой же "парадокс" можно и в вещественных получить:
1^1 = 1^1/2 1^1/2 = (-1) 1 = 1
(так как (-1)^2 = 1 то (-1) - это один из двух квадратных корней 1).

Однако если мы заранее выбрали Ln(1) = 0, а не 2 pi i, то есть выбрали что 1^1/2 это именно 1, а не -1, то только после этого, во-первых, 1^1/2 как и 1^pi вообще cтанет числом, а не множеством чисел, а во-вторых начнёт выполняться закон степеней.

Ну, в общем, в комплексных числах нотация a^z только путает, а не помогает (по крайней мере для неположительных a), поэтому её и не особо используют.

>> No.4876541  

1^1 = 1^1/2 1^1/2 = (-1) 1 = -1, опечатался

>> No.4876634  
Файл: kuri279.jpg -(13 KB, 296x190, kuri279.jpg)
13
>> No.4876981  
Файл: kuri280.jpeg -(13 KB, 225x225, kuri280.jpeg)
13
>> No.4877068  
Файл: ringchain.jpg -(97 KB, 655x600, ringchain.jpg)
97

Какой раздел математики рассматривает объекты с топологической связью, типа кольцо в кольце, цепи, цепь соединенная в кольцо, на которую надели цепь соединенная в кольцо и так далее?

>> No.4877069  
Файл: 07-palz1.jpg -(177 KB, 558x761, 07-palz1.jpg)
177

Мне кажется что это должна быть теория узлов, но я не уверен. В частности, такое "расцепление" как на картинке там должно быть невозможно. А в обычной топологии (где крушка это как бублик) такое вполне происходит

>> No.4877085  
Файл: kuri281.jpeg -(7 KB, 166x304, kuri281.jpeg)
7

>>4877068
>>4877069
Таки да, теория узлов, теория зацеплений, маломерная топология. Там сделано не очень много, но выглядит так, как-будто ещё куча всего будет сделано, потому как такие штуки как словарь Мазура довольно впечатляют.

Ну в топологии довольно аккуратно отличают ситуацию когда кольца "объемные" и когда они "как ниточки", мне кажется очень разумно считать расцепление на твоей картинке легитимным, но при этом оно не будет работать когда кольца и соединяющих их шарнир будут "как ниточки".

>> No.4877110  
Файл: graph.png -(4 KB, 936x758, graph.png)
4

>>4877085

>Ну в топологии довольно аккуратно отличают ситуацию когда кольца "объемные" и когда они "как ниточки", мне кажется очень разумно считать расцепление на твоей картинке легитимным, но при этом оно не будет работать когда кольца и соединяющих их шарнир будут "как ниточки".

Ну тут спорный момент. Можно и для ниточек и для объемных фигур считать такие преобразования как справедливыми, так и несправедливыми (зависит от аксиоматики). Не вижу в этом какой-то "разумности". Мне кажется что просто "так договорились". Для меня напрмиер неочевидна легитимность таких преобразований, будь то нитки (графы) или объемные фигуры.

>> No.4877120  
Файл: kuri282.jpeg -(7 KB, 299x168, kuri282.jpeg)
7

>>4877110
Ну это конечно да, но желательно чтобы "справедливые преобразования" были разумным классом преобразований, для теории узлов/зацеплений обычно берут изотопии объемляющего пространства и я говорил именно для них. Не думаю что можно придумать некоторый разумный класс преобразований, запрещающий то распутывание на картинке с человеком, ну просто потому что оно очень физично, его можно провернуть в реальности, если материал из которого изготовлены фигуры умеет достаточно сильно растягиваться. Ну и я соврал в предыдущий раз, и для тонких и для толстых фигур можно провести это распутывание, тут это роли не играет.

Наверное тебе кажутся преобразование объемных фигур неестественным потому что "материал пропадает", ну это цена которую приходится платить в топологии, чтобы хоть какую-то теорию построить. И так по сути ничего слишком уж умного не сделано, а если ещё метрические препятствия учитывать, то вообще умереть можно. А преобразование графа ниже не того типа, который обычно имеют в виду говоря об узлах или зацеплениях

>> No.4877122  
Файл: graph.png -(2 KB, 659x406, graph.png)
2

И да, похоже что топологию можно описывать как правила замены неких узлов графа.
Типа правила:
(R1 - x - R2) AND (R1 != R2) <-> R1 - R2 // если между двумя какими угодно разными узлами графа есть узел, соединяющий их, то это то же самое, что и просто два этих узла, соединенных напрямую
Хотя кажется что это не совсем граф, ведь в графах вроде бы не предусмотрено ситуаций, чтоб один и тот же узел графа соединялся с другим узлом два. Или есть и такие графы?

>> No.4877129  
Файл: iichan.hk.gif -(1 KB, 52x21, iichan.hk.gif)
1

>>4877122
Хотя, блин, не очень очевидно, как описывать зацепления (продевания) одного в другое. Капча со мной согласна

>> No.4877131  

>>4877122
И такие и такие рассматривают, ты правильно заметил что при стягивании ребра который не является петлёй мы не меняем гомотопический тип графа, но это немного очевидно вообще говоря :з, но что заметил - молодец!
>>4877129
Ты наверное хотел движения рейдемейстера для диаграм узлов придумать.

>> No.4877132  
Файл: kuri283.jpeg -(8 KB, 168x300, kuri283.jpeg)
8
>> No.4877146  
Файл: graph.png -(1 KB, 450x138, graph.png)
1

>>4877131

>Ты наверное хотел движения рейдемейстера для диаграм узлов придумать.

Ну, я думал над "размыканием рук" в картинке из >>4877069 и думал над тем, из-за каких правил оно оказывается справедливым, и мне кажется что вот это правило (что на картинке) - в нем какая-то загвоздка есть. Но чтоб более точное построение сделать, нужен способ описывать закольцованность этих "рук" (топологическая связь) на графах, и тут наверно надо описывать как некие связи между ребрами. Где про это почитать?

>> No.4877153  
Файл: kuri284.jpeg -(10 KB, 191x264, kuri284.jpeg)
10

>>4877146
Уфф, ну и запросы! Пример с разязыванием поверхности рода 2 ("наручников") он как бы вообще игрушечный и, насколько я себе это представляю, за ним никакой абстрактной мощной теории не разработано. Обычно рассматривают просто узлы/зацепления - запутанные "окружности" или несколько штук запутанных друг с другом окружностей. В этом случае плоские диаграммы узлов и движения рейдеймейстера дают полное их комбинаторное описание. Где почитать - везде где о узлах написано, можешь тут: https://web.northeastern.edu/beasley/MATH7375/Lecture2.pdf

>> No.4877168  
Файл: graph.png -(2 KB, 299x367, graph.png)
2

>>4877153

>Пример с разязыванием поверхности рода 2 ("наручников") он как бы вообще игрушечный и, насколько я себе это представляю, за ним никакой абстрактной мощной теории не разработано.

А в каких-нибудь теориях это развянывание наручников является запрещенным?

>Где почитать - везде где о узлах написано, можешь тут: https://web.northeastern.edu/beasley/MATH7375/Lecture2.pdf

Нет, я думал над тем, как это описать через графы, а там это описывается не через графы, а через что-то совсем другое, какие-то гомеоморфизмы... я таких слов не знаю! Например для описывания закольцованных наручников через теорию графов, можно ввести особый узел с двумя и двумя входами, и одни "двавхода" будут зацеплены за другие "двавхода", вот такую теорию я ищу.

>> No.4877173  
Файл: kuri285.jpeg -(8 KB, 197x255, kuri285.jpeg)
8

>>4877168

>А в каких-нибудь теориях это развянывание наручников является запрещенным?

Ну как я уже говорил - в топологических ни в каких, разве что где-то кто-то ещё изучал метрические препятствия (= не хватает длины верёвки) или каких-либо других ограничений, типа введение упругости материала в модель и тд, но это уже мат. моделирование какое-то, а не математика, то есть вряд ли из этого получится выудить какие-то глубокие теоретические результаты, но я могу чего-то и не знать.

>Нет, я думал над тем, как это описать через графы, а там это описывается не через графы, а через что-то совсем другое, какие-то гомеоморфизмы... я таких слов не знаю! Например для описывания закольцованных наручников через теорию графов, можно ввести особый узел с двумя и двумя входами, и одни "двавхода" будут зацеплены за другие "двавхода", вот такую теорию я ищу.

Ну там не гомеоморфизмы, там написано доказательство того, что два узла эквивалентны в кусочно-линейной категории титтк их соответствующие диаграммы переводятся одна в другую движениями рейдеймейстера, что даёт комбинаторное описание "узел = плоская диаграмма узла с точностью до движений рейдеймейстера".

Про графы: ну так в чём проблема закодировать плоскую диаграмму узла в виде графа - там же конечное количество информации, нужно просто аккуратно помнить какие вершинки это пересечения рёбер, а какие вершинки - это просто точки на узле и в вершинках которые пересечения рёбер помнить какое входящее ребро соответствует исходящему и какая пара входящего-исходящего ребра лежит сверху, а какая снизу. Теоретического интереса такие перекодирования из одного формата в другой не представляют, но если интересно то вот https://arxiv.org/pdf/1309.3288.pdf скажем, как это в вольфрам математике делается.

>> No.4877183  
Файл: kuri287.jpg -(99 KB, 1181x1181, kuri287.jpg)
99

>>4877120
Извини ещё раз, тонкую верёвку-наручники всё-таки нельзя развязать указанным тобой способом на пикче, дважды наврал. Не очень сосредоточен на треде просто.

>> No.4877188  
Файл: graph.png -(7 KB, 591x468, graph.png)
7

>>4877173

>разве что где-то кто-то ещё изучал метрические препятствия (= не хватает длины верёвки) или каких-либо других ограничений, типа введение упругости материала в модель и тд, но это уже мат. моделирование какое-то, а не математика

По-моему нет. Мне кажется тут что-то не так. Тут не важна длина веревки. То, что обведено в зеленом прямоугольнике - его нельзя расцепить. А то, что в красном - можно. Если не веришь - можешь из нитки или веревки связать и то и другое. Очевидно, что если из одного узла торчат две петли, их можно как угодно вертеть. А когда петли торчат из двух разных узлов, такое уже не получается. Тут нужны ограничения другого рода

>> No.4877190  
Файл: kuri288.jpg -(236 KB, 1200x628, kuri288.jpg)
236

>>4877188
Да-да-да, ты прав.

>> No.4877191  

>>4877183
Извини, не видел твое сообщение, пока писал ответ. Но я понял, что ты ошибаешься. Даже картинку нарисовал

>> No.4877237  
Файл: boundary.jpg -(117 KB, 1600x864, boundary.jpg)
117

И еще по поводу топологии. Есть такая штука во всяких моделированиях - периодические граничные условия (periodic boundary condition). Для трехмерного случая это будет так: если ты смотришь вперед, ты видишь свою спину, если смотршь налево - видишь себя в профиль с повернутой влево головой. А если посмотреть через прозранчный пол - увидишь себя сверху, смотрящего на пол. Это получается как какой-то четырехмерный тор, ну т.е. когда пространство замыкается подобным образом? Или это четырехмерная сфера? Мне кажется что все же тор. Но как бы это выглядело, будь это четырехмерной сферой?

>> No.4877245  

>>4877237
Возможно будет со всех сторон твоё растянутое изображение. Можно попробовать с обычной сферой, любой луч (который на сфере будет идти по большой окружности) будет упираться в противоположную точку наблюдателя.
Страшновато.

>> No.4877263  
Файл: kuri289.jpg -(105 KB, 1006x708, kuri289.jpg)
105

>>4877237
Трёхмерный тор, размерность по числу свободных параметров "внутри" пространства, обычный тор - двумерный. Чтобы трёхмерную сферу построить нужно всю границу стянуть в одну точку. По-поводу того, как будет всё выглядеть "внутри неё" - как ты уже заметил любая линия будет упираться в точку взгляда поэтому если там находится некоторый материальный объект - будет выглядеть как чёрный фон на некотором расстоянии, если же в твоём поле зрения находится какой-то ещё объект он будет казаться выпуклым и тем более выпуклым чем ближе к горизонту (это потому что в отличии от плоского тора на сфере плоскую метрику постоянной кривизны ввести нельзя, только положительной) .

>> No.4877264  

>>4877263
А почему фон чёрный?

>> No.4877273  
Файл: kuri290.jpg -(101 KB, 600x839, kuri290.jpg)
101

>>4877264
Ну это я дофантозировал, цвета той точки откуда взгляд :з

>> No.4877421  

В этом треде много писали про какие-то программы для доказывания теорем. Почему теоремы до сих пор доказываются людьми-математиками, а не какими-то компьютерными программами, ну там нейросетями всякими? ИИ уже успешно обыгрывает человека во игры типа Го и всяких там шахмат, неужели доказывание теорем исходя из конечного числа аксиом сильно сложнее этого?

>> No.4877423  

>>4877421
Есть алгоритм Тарского. Он:

  1. Работает только на вещественных числах (не больше - не меньше)
  2. Только в логике первого порядка
  3. Дважды экспоненциален по времени от длины формулы.

В общем случае ML невозможен, потому что:

  1. Некоторые доказательства строятся через новые теоремы а то и новый мат. апаррат
  2. Никто не в курсе, как оценивать эвристику, а значит и решать задачу на оптимизацию

Полный перебор недееспособен по очевидным причинам.

>> No.4877424  
Файл: kuri291.jpeg -(77 KB, 600x692, kuri291.jpeg)
77

>>4877421
Нет ещё даже нормального языка программирования, чтобы записывать достаточно сложные (особено геометрические) теоремы, не говоря уже об автоматическом доказательстве.

Ну и видимо всё же сложные теоремы доказывать сложнее, чем на прикидывать хорошую позицию в шахматах или ГО, несмотря даже на комбинаторную сущность проблемы.

Но автопруверы существует и какие-то разработки в этом направлении есть, только вроде как ничего интересного пока нет.

>> No.4877432  

>>4877423

>В общем случае ML невозможен, потому что:
>Некоторые доказательства строятся через новые теоремы а то и новый мат. апаррат

А что именно мешает нейросети придумать и доказать эти теоремы и изобрести новый мат. аппарат?

>Никто не в курсе, как оценивать эвристику, а значит и решать задачу на оптимизацию

Но человек же каким-то образом своей органической нейросетью в мозгах может это делать, верно? Наверняка этот процесс рассуждений в мозгу у человека можно перенести и в ML. Разве что у человека есть какая-то волшебная интуиция или что-то подобное (сознание, душа может какая-то), что позволяет ему делать это эффективней, чем какой угодно "бездушной" искуственной нейросети.

>> No.4877437  

>>4877432
Скорее всего самая главная проблема просто в недостаточной вычислительной мощности. Программы писать явно проще, чем теоремы доказывать, а ты попробуй автоматизируй (хотя у теорем, конечно, обычно, гораздо чётче формулировка, чем спецификация программы, если та в принципе есть).

>> No.4877439  

>>4877432

>А что именно мешает нейросети придумать и доказать эти теоремы и изобрести новый мат. аппарат?

Проблема остановки и комбинаторная сложность.

>Наверняка этот процесс рассуждений в мозгу у человека можно перенести и в ML.

В ML не переносят, в ML берут пяток моделей, два года крутят параметры а потом говорят "этот классификатор лучше". А потом собак в людях находят. ML не про формулирование принципов, а про подбор.

>> No.4877631  

как доказать равенство функций x/abs(x), x^3/abs(x)^3, x^5/abs(x)^5 x^7/abs(x)^7 ... ?
да и вообще, как доказать что если любые функции f(x), которые определены для всех x, и для x < 0 они всегда отрицательны, а для x > 0 всегда положительны, а в x=0 они 0, то f(x)/abs(f(x)) это то же самое?
Это как функция Хевисайда получается, только надо на 2 поделить и прибавить 0.5. Или signum-функция

>> No.4877668  

>>4877631
x^2 = |x|^2

>то f(x)/abs(f(x)) это то же самое

То есть, единица. Раскрываешь модуль как составную функцию, делишь на каждом отрезке, получаешь составную функцию из -1 и 1. Конец.




[d | an-b-bro-fr-gf-hr-l-m-maid-med-mi-mu-ne-o-old_o-p-ph-r-s-sci-sp-t-tran-tv-w-x | bg-vg | au-mo-tr | a-aa-abe-azu-c-dn-fi-hau-jp-ls-ma-me-rm-sos-tan-to-vn | misc-tenma-vndev | dev-stat]
[Burichan] [Futaba] [Gurochan] [Tomorrow] [Архив-Каталог-RSS] [Главная]